OB & peds class quizzes

Pataasin ang iyong marka sa homework at exams ngayon gamit ang Quizwiz!

Which of the following signs is an EARLY indicator of post op bleeding after tonsillectomy? a) decreased blood pressure b) restlessness c) frequent swallowing d) all of the above

c) frequent swallowing

A postpartum client tells the nurse that she wishes to breast-feed. When the nurse brings her newborn to be breast-fed, the client asks whether she may drink a small glass of wine to help her relax. How should the nurse respond? 1 "I think drinking one glass of wine won't be a problem. Go ahead." 2 "You seem a little tense. Tell me how you feel about breast-feeding." 3 "You seem to find it relaxing, but you should try to find another way to relax." 4 "I think drinking one glass of wine is alright, but you had better check with your doctor first."

2 "You seem a little tense. Tell me how you feel about breast-feeding."

A 10-year-old child is found to have hemophilia. The nurse is explaining how hemophilia is inherited. What is the best explanation of the genetic factor that is involved? 1 It follows the Mendelian law of inherited disorders. 2 The mother is a carrier of the disorder but usually is not affected by it. 3 It is an autosomal dominant disorder in which the woman carries the trait. 4 A carrier may be male or female, but the disease occurs in the sex opposite that of the carrier

2 The mother is a carrier of the disorder but usually is not affected by it.

On the third day of a 2-year-old toddler's hospitalization the nurse notes that the child, who had been screaming and crying inconsolably, has begun to regress and is now lying quietly in the crib with a blanket. What stage of separation anxiety has developed? 1 Denial 2 Despair 3 Mistrust 4 Rejection

2 Despair The second stage of separation anxiety is despair, in which the child is depressed, lonely, and uninterested in the surroundings.

A nurse teaches a pregnant woman about the need to increase her intake of complete proteins. Which foods identified by the client indicate that the teaching is effective? Select all that apply. 1 Nuts 2 Milk 3 Eggs 4 Bread 5 Beans 6 Cheese

2 Milk 3 Eggs 6 Cheese

What does a nurse who is assessing a newborn 3 minutes after birth remember is the range of heart rate for a healthy, alert neonate? 1 120 and 180 beats/min 2 130 and 170 beats/min 3 110 and 160 beats/min 4 100 and 130 beats/min

3 110 and 160 beats/min

The nurse teaches a high school sex education class that herpes genitalis infection cannot be cured, but the disease is marked by remissions and exacerbations. What else should the students be taught about this infection? 1 A healthy lifestyle will prevent exacerbations. 2 Once the infection is effectively treated, exacerbations are rare. 3 Although exacerbations occur, they are not as severe as the initial episode. 4 The most effective way to prevent exacerbations is to abstain from sexual activity.

3 Although exacerbations occur, they are not as severe as the initial episode.

A nurse is caring for an infant with Down syndrome. What does the nurse recall as the most common serious anomaly associated with this disorder? 1 Renal disease 2 Hepatic defects 3 Congenital heart disease 4 Endocrine gland malfunction

3 Congenital heart disease

What is the cause of milk anemia in toddlers? 1 Drinking skim milk 2 Drinking whole milk 3 Increased milk intake 4 Increased intake of fruits

Increased milk intake

What is the most appropriate nursing diagnosis for a child diagnosed with moderate anemia? a. Activity intolerance related to generalized weakness b. Risk for injury related to depressed sensorium c. Risk for Injury related to dehydration and abnormal hemoglobin d. Decreased cardiac output related to abnormal hemoglobin

a. Activity intolerance related to generalized weakness

José is 4 year old. Preoperative teaching for a 4-year-old child scheduled for a cardiac catheterization should be done with what primary consideration in mind? Select one: a. Adapted to his level of development so that he can understand. b. Done several days before the procedure so that he will be prepared. c. Directed at his parents because he is too young to understand. d. Detailed in regard to the actual procedures so he will know what to expect.

a. Adapted to his level of development so that he can understand.

Postpartal overdistention of the bladder and urinary retention can lead to which complications? a. After birth hemorrhage and urinary tract infection b. Urinary tract infection and uterine rupture c. Fever and increased blood pressure d. After birth hemorrhage and eclampsia

a. After birth hemorrhage and urinary tract infection

A young child has just injured an ankle at school. In addition to calling the child's parents, what is the most appropriate immediate action by the school nurse? Select one: a. Apply ice b. Obtain parental permission for administration of acetaminophen or aspirin c. Encourage child to assume a comfortable position d. Observe for edema and discoloration

a. Apply ice

Which interaction is part of the discharge plan for a school-age child with osteomyelitis who is receiving home antibiotic therapy? Select one: a. Arrangements for tutoring and schoolwork b. Instructions for a low-calorie diet c. Instructions for the parent to return the child to team sports immediately d. Instructions for a high-fat, low-protein diet

a. Arrangements for tutoring and schoolwork

Which clinical manifestations would suggest hydrocephalus in a neonate? Select one: a. Bulging fontanel and dilated scalp veins b. Constant low-pitched cry and restlessness c. Depressed fontanel and decreased blood pressure d. Closed fontanel and high-pitched cry

a. Bulging fontanel and dilated scalp veins

The nurse providing care in a women's health care setting must be aware regarding which sexually transmitted infection that can be successfully treated and cured? Select one: a. Chlamydia b. Venereal warts c. Acquired immunodeficiency syndrome (AIDS) d. Herpes

a. Chlamydia

What is the primary nursing intervention necessary to prevent bacterial endocarditis? Select one: a. Counsel parents of high risk children about prophylactic antibiotics. b. Encourage restricted mobility in susceptible children. c. Institute measures to prevent dental procedures. d. Observe children for complications such as embolism and heart failure.

a. Counsel parents of high risk children about prophylactic antibiotics.

What is the primary result of anemia? a. Decreased oxygen-carrying capacity of blood. b. Increased blood viscosity. c. Depressed hematopoietic system. d. Presence of abnormal hemoglobin.

a. Decreased oxygen-carrying capacity of blood.

An infant is brought to the emergency department with poor skin turgor, sunken fontanel, lethargy, and tachycardia. This is suggestive of which condition? a. Dehydration b. Overhydration c. Calcium excess d. Sodium excess

a. Dehydration

A young child with human immunodeficiency virus is receiving several antiretroviral drugs. What is the expected outcome of these drug therapies? a. Delay disease progression b. Cure the disease c. Treat Pneumocystis jiroveci pneumonia d. Prevent spread of disease

a. Delay disease progression

Which nursing intervention is appropriate when caring for a child who has experienced a seizure? a. Describe and record the seizure activity observed. b. Place a tongue blade between the teeth if they become clenched. c. Restrain the child when seizure occurs to prevent bodily harm. d. Suction the child during a seizure to prevent aspiration.

a. Describe and record the seizure activity observed.

β-Adrenergic agonists, such as albuterol (Ventolin), are often prescribed for a child with an asthma attack for what resulting action? a. Dilation of the bronchioles b. Liquefaction of secretions c. Reduction of existing infection d. Reduction of inflammation of the lungs

a. Dilation of the bronchioles

A newborn assessment shows separated sagittal suture, oblique palpebral fissures, depressed nasal bridge, protruding tongue, and transverse palmar creases. These findings are most suggestive of: Select one: a. Down syndrome. b. cerebral palsy. c. microcephaly. d. fragile X syndrome.

a. Down syndrome.

The nurse, caring for a neonate with a suspected tracheoesophageal fistula, should include what intervention into the plan of care? a. Elevating the head to facilitate secrete drainage. b. Elevating the head for feedings only. c. Feeding glucose water only. d. Avoiding suctioning unless the infant is cyanotic.

a. Elevating the head to facilitate secrete drainage.

Chelation therapy is begun on a child with β-thalassemia major with what expected result? Select one: a. Elimination of excess iron. b. Managing nausea and vomiting. c. Treatment of the disease. d. Decreasing the risk of hypoxia.

a. Elimination of excess iron.

A mother asks a nurse why it is necessary to have her baby immunized if she disinfects her house daily. The nurse's best response is: a. Even if you clean every day, your baby is susceptible to infections because a baby's immune system is immature b. Why don't you discuss your concerns with your baby's doctor c. Is is a state law that your child received immunizations d. You don't have to immunize your child if you are sure you disinfect everything in your house

a. Even if you clean every day, your baby is susceptible to infections because a baby's immune system is immature

What is the most appropriate intervention for the parents of a 6-year-old girl with precocious puberty? Select one: a. Explain the importance of having the child foster relationships with same-age peers. b. Counsel parents that there is no treatment currently available for this disorder. c. Assure the child's parents that there is no increased risk for sexual abuse because of her appearance. d. Advise the parents to consider birth control for their daughter.

a. Explain the importance of having the child foster relationships with same-age peers.

Nurses providing nutritional instruction should be cognizant of the uniqueness of human milk. Which statement is correct? Select one: a. Frequent feedings during predictable growth spurts stimulate increased milk production. b. The milk of preterm mothers is the same as the milk of mothers who gave birth at term. c. The milk at the beginning of the feeding is the same as the milk at the end of the feeding. d. Colostrum is an early, less concentrated, less rich version of mature milk.

a. Frequent feedings during predictable growth spurts stimulate increased milk production.

Which documentation on a woman's chart on after birth day 14 indicates a normal involution process? Select one: a. Fundus below the symphysis and not palpable b. Episiotomy slightly red and puffy c. Moderate bright red lochial flow d. Breasts firm and tender

a. Fundus below the symphysis and not palpable

Which topic should be a priority to teach to daycare facility staff to help control the spread of infections? a. Hand washing procedures b. Immunization schedules for infants and children c. Immune system development in childhood d. Etiology of common infectious diseases

a. Hand washing procedures

The nurse is caring for an 11-year-old boy who has recently been diagnosed with diabetes. What should be included in the teaching plan for daily injections? Select one: a. He is old enough to give most of his own injections. b. Self-injections will be possible when he is closer to adolescence. c. The parents do not need to learn the procedure. d. He can learn about self-injections when he is able to reach all injection sites.

a. He is old enough to give most of his own injections.

What immunizations are due at a 2-month-old well child visit? [select all that apply] a. IPV b. HBV c. Hib d. MMR e. Rota

a. IPV b. HBV c. Hib e. Rota

Which statement expresses accurately the genetic implications of cystic fibrosis (CF)? a. If it is present in a child, both parents are carriers of this defective gene. b. It is inherited as an autosomal dominant trait. c. It is a genetic defect found primarily in non-Caucasian population groups. d. There is a 50% chance that siblings of an affected child also will be affected.

a. If it is present in a child, both parents are carriers of this defective gene.

What is a common side effect of corticosteroid therapy? Select one: a. Increased appetite b. Fever c. Weight loss d. Hypertension

a. Increased appetite

Which statement most accurately describes the pathologic changes of sickle cell anemia? a. Increased red blood cell destruction occurs b. Sickle-shaped cells decrease blood viscosity c. Sickle-shaped cells carry excess oxygen d. Decreased red blood cell destruction occurs

a. Increased red blood cell destruction occurs

A parent whose child has been diagnosed with a cognitive deficit should be counseled about what fact related to intellectual impairment? Select one: a. Is likely caused by a variety of factors. b. Is usually caused by parental intellectual impairment. c. Is usually due to a genetic defect. d. Is rarely due to first-trimester events.

a. Is likely caused by a variety of factors.

Which statement is most descriptive of pediatric family-centered care? a. It recognizes that the family is the constant in a child's life. b. It reduces the effect of cultural diversity on the family. c. It avoids expecting families to be part of the decision-making process. d. It encourages family dependence on the health care system.

a. It recognizes that the family is the constant in a child's life.

Which of the followng statements is true regarding care of the child with Kawasaki disease? Select one: a. Management includes administration of intravenous immune globulin and aspirin. b. The principal area of involvement is the joints. c. The child's fever is usually responsive to antibiotics within 48 hours. d. Aspirin is contraindicated.

a. Management includes administration of intravenous immune globulin and aspirin.

A woman has a thick, white, lumpy, cottage cheese-like discharge, with patches on her labia and in her vagina. She complains of intense pruritus. The nurse practitioner would order which preparation for treatment? Select one: a. Miconazole b. Tetracycline c. Acyclovir d. Clindamycin

a. Miconazole

Which statement best explains why iron deficiency anemia is common during toddlerhood? Select one: a. Milk is a poor source of iron. b. Fetal iron stores are depleted by age 1 month. c. Iron cannot be stored during fetal development. d. Dietary iron cannot be started until age 12 months.

a. Milk is a poor source of iron.

Which of the following are recommended nursing interventions when caring for a child who is receiving opioid medications? (select all that apply) a. Monitor for respiratory rate and oxygen saturation b. Administer loperamide to control diarrhea c. Ensure that nalaxone (Narcan) is readily available d. Combine with use of non-pharmacologic and non-opioid methods to achieve pain control e. Administer by oral or intravenous routes rather than intramuscular routes

a. Monitor for respiratory rate and oxygen saturation c. Ensure that nalaxone (Narcan) is readily available d. Combine with use of non-pharmacologic and non-opioid methods to achieve pain control e. Administer by oral or intravenous routes rather than intramuscular routes

What is the most common problem for children born with a myelomeningocele? a. Neurogenic bladder b. Intellectual impairment c. Cranioschisis d. Respiratory compromise

a. Neurogenic bladder

Which instruction should be included in the discharge teaching plan to assist the patient in recognizing early signs of complications? Select one: a. Notify the physician of any increase in the amount of lochia or a return to bright red bleeding. b. Palpate the fundus daily to ensure that it is soft. c. Report any decrease in the amount of brownish red lochia. d. The passage of clots as large as an orange can be expected.

a. Notify the physician of any increase in the amount of lochia or a return to bright red bleeding.

Which pain assessment tool is most appropriate for assessing an adolescent? Select one or more: a. Numeric Scale b. Wong-Bake Faces Scale c. NIPS d. rFLACC

a. Numeric Scale

The nurse, talking with the tearful parent of a child newly diagnosed with a chronic illness, asks, "Who do you talk with when something is worrying you?" What is the purpose of this statement? Select one: a. Part of assessing parent's available support system. b. Inappropriate, because parent is so upset. c. A diversion of the present crisis to similar situations with which parent has dealt. d. An intervention to find someone to help parent.

a. Part of assessing parent's available support system.

A nurse is preparing to complete an admission assessment on a 2-year-old child who is sitting on the parent's lap. Which technique should the nurse implement to complete the physical examination? a. Perform the examination while the child is on the parent's lap. b. Ask the child to stand by the parent while completing the examination. c. Ask the parent to place the child in the hospital crib. d. Take the child and parent to the examination room.

a. Perform the examination while the child is on the parent's lap.

What is the most common mode of transmission of human immunodeficiency virus (HIV) in the pediatric population? Select one: a. Perinatal transmission b. Poor hand washing c. Sexual abuse d. Blood transfusions

a. Perinatal transmission

A child is admitted to the hospital with a diagnosis of "rule out pertussis". The physician's orders include: nasopharyngeal (NP) culture, Erythromycin 250 mg po q 6 hours, and respiratory isolation. The nurse would institute these orders in what sequence? a. Place in isolation, culture NP, administer Erythromycin b. Place in isolation, administer Erythromycin, culture NP c. Culture NP, place in isolation, administer Erythromycin d. Culture NP, administer Erythromycin, place in isolation

a. Place in isolation, culture NP, administer Erythromycin

A pregnant woman's amniotic membranes rupture. Prolapsed umbilical cord is suspected. What intervention would be the top priority? a. Placing the woman in the knee-chest position. b. Covering the cord in sterile gauze soaked in saline. c. Preparing the woman for a cesarean birth. d. Starting oxygen by face mask.

a. Placing the woman in the knee-chest position.

The feeling of guilt that the child "caused" the disability or illness is especially critical in which child? Select one: a. Preschooler b. Adolescent c. School-age child d. Toddler

a. Preschooler

Which function of play is a major component of play at all ages? a. Sensorimotor activity b. Socialization c. Creativity d. Intellectual development

a. Sensorimotor activity

What represents the major stressor of hospitalization for children from middle infancy throughout the preschool years? a. Separation anxiety b. Fear of pain c. Loss of control d. Fear of bodily injury

a. Separation anxiety

Which factor predisposes a child to urinary tract infections? a. Short urethra in young girls b. Frequent emptying of the bladder c. Increased fluid intake d. Prostatic secretions in males

a. Short urethra in young girls

When developing a plan of care for a 1-month-old infant with sepsis, which of the following instructions would the nurse expect to include? a. Strictly monitor input and output b. Weigh the infant every other day c. Use the intravenous line for antibiotics only d. Monitor vital signs every 6 hours

a. Strictly monitor input and output

A parent asks the nurse why a developmental assessment is being conducted for a child during a routine well-child visit. The nurse answers based on what knowledge about such routine developmental assessments? Select one: a. The best method for early detection of cognitive disorders. b. Not necessary unless the parents request them. c. Valuable in measuring intelligence in children. d. Frightening to parents and children and should be avoided.

a. The best method for early detection of cognitive disorders.

A Chinese toddler has pneumonia. The nurse notices that the parent consistently feeds the child only the broth that comes on the clear liquid tray. Food items such as Jell-O, Popsicles, and juices are left. What would best explain this? a. The parent is trying to restore normal balance through appropriate "hot" remedies. b. Hispanics believe that an innate energy called chi is strengthened by eating soup. c. The parent is trying to feed child only what child likes most. d. Hispanics believe that the "evil eye" enters when a person gets cold.

a. The parent is trying to restore normal balance through appropriate "hot" remedies.

When a child diagnosed with chronic renal failure, the progressive deterioration produces a variety of clinical and biochemical disturbances that eventually are manifested in the clinical syndrome known as what? Select one: a. Uremia b. Proteinuria c. Pyelonephritis d. Oliguria

a. Uremia

When a child is diagnosed with chronic renal failure, the progressive deterioration produces a variety of clinical and biochemical disturbances that eventually are manifested in the clinical syndrome known as what? a. Uremia b. Proteinuria c. Oliguria d. Pyelonephritis

a. Uremia

Which of the following are recommended nursing actions for treating pain in a child? (select all that apply) Select one or more: a. Use round the clock scheduling if continued pain is anticipated b. Give IM injections of pain medicines before trying PO and IV routes c. Withhold opioids to reduce the risk of addiction d. Use an appropriate assessment tool based on the child's developmental level e. Choose a treatment method based on the child, pain history, and the situation

a. Use round the clock scheduling if continued pain is anticipated d. Use an appropriate assessment tool based on the child's developmental level e. Choose a treatment method based on the child, pain history, and the situation

What is an important consideration for the nurse who is communicating with a very young child? a. Use transition objects such as a doll. b. Disguise own feelings, attitudes, and anxiety. c. Speak loudly, clearly, and directly. d. Initiate contact with the child when the parent is not present.

a. Use transition objects such as a doll.

Which immunization should be given with caution to children infected with human immunodeficiency virus? a. Varicella b. Inactivated poliovirus c. Influenza d. Pneumococcus

a. Varicella

What immunizations are due at a 4-year-old well child check-up? [Select all that apply] a. Varicella b. MMR c. Tdap d. DTaP e. HPV

a. Varicella b. MMR d. DTaP

As related to inherited disorders, which statement is descriptive of most cases of hemophilia? a. X-linked recessive inherited disorder in which a blood-clotting factor is deficient b. Autosomal dominant disorder causing deficiency in a factor involved in the blood-clotting reaction c. Y-linked recessive inherited disorder in which the red blood cells become moon shaped d. X-linked recessive inherited disorder causing deficiency of platelets and prolonged bleeding

a. X-linked recessive inherited disorder in which a blood-clotting factor is deficient

As related to inherited disorders, which statement is descriptive of most cases of hemophilia? Select one: a. X-linked recessive inherited disorder in which a blood-clotting factor is deficient b. Y-linked recessive inherited disorder in which the red blood cells become moon shaped c. Autosomal dominant disorder causing deficiency in a factor involved in the blood-clotting reaction d. X-linked recessive inherited disorder causing deficiency of platelets and prolonged bleeding

a. X-linked recessive inherited disorder in which a blood-clotting factor is deficient

A new mother states that her infant must be cold because the baby's hands and feet are blue. The nurse explains that this is a common and temporary condition called: Select one: a. acrocyanosis. b. harlequin color. c. vernix caseosa. d. erythema neonatorum.

a. acrocyanosis.

An examiner who discovers unequal movement or uneven gluteal skinfolds during the Ortolani maneuver would then: Select one: a. alert the physician that the infant has a dislocated hip. b. tell the parents that one leg may be longer than the other, but they will equal out by the time the infant is walking. c. suggest that, if the condition does not change, surgery to correct vision problems may be needed. d. inform the parents and physician that molding has not taken place.

a. alert the physician that the infant has a dislocated hip.

The priority nursing intervention after an amniotomy should be to: a. assess the fetal heart rate. b. change the patient's gown. c. estimate the amount of amniotic fluid. d. assess the color of the amniotic fluid.

a. assess the fetal heart rate.

An infant boy was born just a few minutes ago. The nurse is conducting the initial assessment. Part of the assessment includes the Apgar score. The Apgar assessment is performed: Select one: a. at least twice, 1 minute and 5 minutes after birth. b. only if the newborn is in obvious distress. c. every 15 minutes during the newborn's first hour after birth. d. once by the obstetrician, just after the birth.

a. at least twice, 1 minute and 5 minutes after birth.

A new mother wants to be sure that she is meeting her daughter's needs while feeding her commercially prepared infant formula. The nurse should evaluate the mother's knowledge about appropriate infant care. The mother meets her child's needs when she: Select one: a. burps her infant during and after the feeding as needed. b. refrigerates any leftover formula for the next feeding. c. warms the bottles using a microwave oven. d. adds rice cereal to her formula at 2 weeks of age to ensure adequate nutrition.

a. burps her infant during and after the feeding as needed.

A newborn is placed under a radiant heat warmer, and the nurse evaluates the infant's body temperature every hour. Maintaining the newborn's body temperature is important for preventing: Select one: a. cold stress. b. respiratory depression. c. tachycardia. d. vasoconstriction.

a. cold stress.

The nurse caring for the after birth woman understands that breast engorgement is caused by: a. congestion of veins and lymphatics. b. hyperplasia of mammary tissue. c. overproduction of colostrum. d. accumulation of milk in the lactiferous ducts.

a. congestion of veins and lymphatics.

A woman is experiencing back labor and complains of intense pain in her lower back. An effective relief measure would be to use: a. counterpressure against the sacrum. b. pant-blow (breaths and puffs) breathing techniques. c. conscious relaxation or guided imagery. d. effleurage.

a. counterpressure against the sacrum.

A woman is 16 weeks pregnant and has elected to terminate her pregnancy. The nurse knows that the most common technique used for medical termination of a pregnancy in the second trimester is: a. dilation and evacuation (D&E). b. instillation of hypertonic saline into the uterine cavity. c. vacuum aspiration. d. intravenous administration of Pitocin.

a. dilation and evacuation (D&E).

While assessing the integument of a 24-hour-old newborn, the nurse notes a pink, papular rash with vesicles superimposed on the thorax, back, and abdomen. The nurse should: Select one: a. document the finding as erythema toxicum. b. notify the physician immediately. c. move the newborn to an isolation nursery. d. take the newborn's temperature and obtain a culture of one of the vesicles.

a. document the finding as erythema toxicum.

A woman in preterm labor at 30 weeks of gestation receives two 12-mg doses of betamethasone intramuscularly. The purpose of this pharmacologic treatment is to: a. stimulate fetal surfactant production. b. reduce maternal and fetal tachycardia associated with ritodrine administration. c. suppress uterine contractions. d. maintain adequate maternal respiratory effort and ventilation during magnesium sulfate therapy.

a. stimulate fetal surfactant production.

With regard to the noncontraceptive medical effects of combined oral contraceptive pills (COCs), nurses should be aware that: a. the effectiveness of COCs can be altered by some over-the-counter medications and herbal supplements. b. COCs increase the risk of endometrial and ovarian cancer. c. hormonal withdrawal bleeding usually is a bit more profuse than in normal menstruation and lasts a week. d. COCs can cause toxic shock syndrome if the prescription is wrong.

a. the effectiveness of COCs can be altered by some over-the-counter medications and herbal supplements.

A 25-year-old gravida 2, para 2-0-0-2 gave birth 4 hours ago to a 9-lb, 7-ounce boy after augmentation of labor with Pitocin. She puts on her call light and asks for her nurse right away, stating, "I'm bleeding a lot." The most likely cause of after birth hemorrhage in this woman is: Select one: a. uterine atony. b. unrepaired vaginal lacerations. c. retained placental fragments. d. puerperal infection.

a. uterine atony.

The perinatal nurse is caring for a woman in the immediate postbirth period. Assessment reveals that the woman is experiencing profuse bleeding. The most likely etiology for the bleeding is: a. uterine atony. b. vaginal hematoma. c. uterine inversion. d. vaginal laceration.

a. uterine atony.

A woman in labor hears the primary healthcare provider tell the nurse that the fetal lie is longitudinal. The mother asks the nurse what this means in relation to her labor and birth of the baby. How should the nurse respond? 1 "A vaginal birth is possible." 2 "We're anticipating a cesarean delivery." 3 "It has no relevance to the labor and birth." 4 "Labor probably will be long, and you might have back pain."

1 "A vaginal birth is possible."

The nurse is teaching participants in a prenatal class regarding breastfeeding versus formula feeding. A client asks, "What is the primary advantage of breastfeeding?" Which response is most appropriate? 1 "Breastfed infants have fewer infections." 2 "Breastfeeding inhibits ovulation in the mother." 3 "Breastfed infants adhere more easily to a feeding schedule." 4 "Breastfeeding provides more protein than cow's milk formula does."

1 "Breastfed infants have fewer infections."

An infant has been admitted with failure to thrive (FTT). The nurse knows that more education is needed when one of the parents makes what statement? 1 "I can double the amount of water in the formula to save money." 2 "I need to hold her head up a little higher than her stomach when I feed her." 3 "I need to burp the baby when the feeding is done to get rid of swallowed air." 4 "I need to make sure that the formula is in the nipple so she doesn't swallow so much air."

1 "I can double the amount of water in the formula to save money."

The nurse is talking to the parent of a 13-month-old child. The mother states, "My child does not make noises like 'da' or 'na' like my sister's baby, who is only 9 months old." Which statement by the nurse would be most appropriate to make? a. "You should ask other parents what noises their children made at this age." b. "I am going to request a referral to a hearing specialist." c. "You should not compare your child to your sister's child." d. "I think your child is fine, but we will check again in 3 months."

b. "I am going to request a referral to a hearing specialist."

A male patient asks the nurse why it is better to purchase condoms that are not lubricated with nonoxynol-9 (a common spermicide). The nurse's most appropriate response is: a. "The additional lubrication improves sex." b. "It has also been linked to an increase in the transmission of human immunodeficiency virus." c. "The lubricant prevents vaginal irritation." d. "Nonoxynol-9 improves penile sensitivity."

b. "It has also been linked to an increase in the transmission of human immunodeficiency virus."

Which statement by a parent about a child's conjunctivitis indicates that further teaching is needed? a. "I'll notify my doctor if the eye gets redder or the drainage increases." b. "When the eye drainage improves, we'll stop giving the antibiotic ointment." c. "After taking the antibiotic for 24 hours, my child can return to school." d. "I'll have separate towels and washcloths for each family member."

b. "When the eye drainage improves, we'll stop giving the antibiotic ointment."

While assessing the newborn, the nurse should be aware that the average expected apical pulse range of a full-term, quiet, alert newborn is: Select one: a. 80 to 100 beats/min. b. 120 to 160 beats/min. c. 100 to 120 beats/min. d. 150 to 180 beats/min.

b. 120 to 160 beats/min.

A nurse is educating a group of mothers about the nutritional needs of toddlers. Which of these statements made by a mother indicate the need for further teaching? Select all that apply. 1 "I need to give more than 24 ounces of milk per day." 2 "I should provide more calories from fats than proteins." 3 "I should refrain from giving grapes, nuts, and raw vegetables." 4 "I should give foods rich in calcium and phosphorus for healthy bone growth." 5 "I should try to give small, frequent meals consisting of breakfast, lunch, and dinner."

1 "I need to give more than 24 ounces of milk per day." 2 "I should provide more calories from fats than proteins." Toddlers who consume more than 24 ounces of milk daily in place of other foods may develop milk anemia, because milk is a poor source of iron. A toddler needs more protein than fats in the diet.

Which condition is caused by a virus that primarily infects a specific subset of T lymphocytes, the CD4+ T-cells? a. Wiskott-Aldrich syndrome b. Acquired immunodeficiency syndrome (AIDS) c. Severe combined immunodeficiency disease d. Idiopathic thrombocytopenic purpura (ITP)

b. Acquired immunodeficiency syndrome (AIDS)

Which questions should the nurse include when conducting a health history interview with the parents of a 4-year-old client to assess fine motor skills? Select all that apply. 1 "Is your child able to use scissors?" 2 "Is your child able to ride a tricycle?" 3 "Is your child able to tie shoe laces?" 4 "Is your child able to climb stairs using alternate feet?" 5 "Is your child able to build a tower using 9 or 10 blocks?"

1 "Is your child able to use scissors?" 3 "Is your child able to tie shoe laces?"

Glucocorticoids, mineralocorticoids, and sex steroids are secreted by which organ? Select one: a. Anterior pituitary b. Adrenal cortex c. Parathyroid glands d. Thyroid gland

b. Adrenal cortex

A pregnant client asks the nurse for information regarding toxoplasmosis exposure during pregnancy. What information should the nurse teach this client? 1 "Pork and beef should be cooked thoroughly." 2 "Toxoplasmosis is a disease that is most prevalent in foreign countries." 3 "Cooked shellfish are intermediary hosts and should be avoided during pregnancy" 4 "Salad dressings made with mayonnaise should be avoided during the summer months."

1 "Pork and beef should be cooked thoroughly."

What important instruction should a nurse include when teaching about the correct use of a female condom? 1 "Remove the condom before standing up." 2 "Insert the condom within 1 hour before intercourse." 3 "Have your partner wear a male condom at the same time." 4 "Cleanse the condom with warm water when preparing it for future use."

1 "Remove the condom before standing up."

How should a nurse respond to parents who are concerned about separation anxiety in their 15-month-old toddler? 1 "This is an expected developmental reaction." 2 "You may be spending too much time with your child." 3 "It might be helpful to leave your child with someone once in a while." 4 "Toddlers who have separation anxiety may have difficulty when they start school."

1 "This is an expected developmental reaction." Understanding that separation anxiety is an expected developmental occurrence will be reassuring to the parents

The nurse educates the parents of a toddler-age client regarding play and toys appropriate for this stage of development. Which parental responses indicate correct understanding of the information presented? Select all that apply. 1 "We should expect our child to participate in parallel play." 2 "We should provide our child with finger paints to foster creativity." 3 "We should allow our child to watch as much television as she wants." 4 "We should provide our child with toys that foster her imagination, such as a doll." 5 "We should provide our child with toys so that we are able to finish household chores."

1 "We should expect our child to participate in parallel play." 2 "We should provide our child with finger paints to foster creativity." 4 "We should provide our child with toys that foster her imagination, such as a doll."

The nurse discusses the recommended weight gain during pregnancy with a newly pregnant client who is 5 ft 3 in (160 centimeters) tall and weighs 130 lb (57 kilograms). The nurse explains that with the recommended weight gain, at term the client should weigh how much? 1 155 lb (70 kg) 2 140 lb (63.5 kg) 3 135 lb (61 kg) 4 130 lb (57 kg)

1 155 lb (70 kg) A weight of 155 lb (70 kg) would put the client within the recommended weight gain of at least 25 lb (11 kg) for a woman who was of average weight for her height before pregnancy.

A nurse in a family planning clinic determines that a client understands the discussion regarding use of a cervical cap with a spermicide when the client states that after intercourse, a cervical cap must be left in place for at least how long? 1 6 hours 2 5 hours 3 3 hours 4 2 hours

1 6 hours The cervical cap, used in conjunction with a spermicide that remains active for 6 hours, provides the most effective contraceptive result.

A woman in active labor arrives at the birthing unit. She tells the nurse that she was told that she had a chlamydial infection the last time she visited the clinic; however, she stopped taking the antibiotic after 3 days because she "felt better." In light of this history what would the nurse anticipate as part of the plan of care? 1 Administration of antibiotics before delivery 2 Oxytocin infusion to augment labor 3 Epidural anesthesia to relieve difficult labor discomfort 4 Magnesium sulfate infusion to prevent a precipitous birth

1 Administration of antibiotics before delivery

A client in her tenth week of pregnancy exhibits presumptive signs of pregnancy. Which clinical findings may the nurse determine upon assessment? Select all that apply. 1 Amenorrhea 2 Breast changes 3 Urinary frequency 4 Abdominal enlargement 5 Positive urine pregnancy test

1 Amenorrhea 2 Breast changes 3 Urinary frequency The key to answering this question is understanding the difference between presumptive versus probable signs of pregnancy. Presumptive signs of pregnancy are less specific subjective changes that are reported by the client during an assessment interview. Probable signs of pregnancy are more objective changes that can be measured in the reproductive organs during a physical assessment.

The nurse is caring for a postpartum client who has chosen formula feeding. What should the nurse teach her regarding minimizing breast discomfort? 1 Apply covered ice packs to the breasts. 2 Gently apply cocoa butter to the nipples. 3 Place warm, wet washcloths on the nipples. 4 Manually express colostrum from the breasts.

1 Apply covered ice packs to the breasts.

A pregnant woman presents in labor at term, having had no prenatal care. After birth her infant is noted to be small for gestational age with small eyes and a thin upper lip. The infant also is microcephalic. On the basis of her infant's physical findings, this woman should be questioned about her use of which substance during pregnancy? Select one: a. Heroin b. Alcohol c. Cocaine d. Marijuana

b. Alcohol

At her first prenatal clinic visit a primigravida has blood drawn for a rubella antibody screening test, and the results are positive. Which intervention is important when the nurse discusses this finding with the client? 1 Asking her whether she has ever had German measles and when she had the disease 2 Arranging for her to receive the rubella booster vaccine after the birth 3 Planning for her to receive the rubella booster vaccine at her next visit 4 Informing her that the result was expected and that treatment will not be needed

1 Asking her whether she has ever had German measles and when she had the disease The positive result indicates that the client has had rubella or was vaccinated. The nurse should determine whether she has had the disease, because it is important to know whether it was before or after she became pregnant; if she had rubella at the start of her pregnancy, the fetus is at risk.

A nurse is admitting a 4-year-old child to the pediatric unit and making plans to help the child adjust to the unit. The nurse understands that a 4-year-old child's greatest fear related to hospitalization is a fear of what? 1 Bodily harm 2 Lack of control 3 Loss of independence 4 Separation from mother

1 Bodily harm The psychosexual development of a preschooler is focused on the fear of invasive procedures.

How does the nurse determine when true labor and not false labor is present? 1 Cervical dilation is evident. 2 Contractions stop when the client walks around. 3 The client's contractions progress only when she is in a side-lying position. 4 Contractions occur immediately after the membranes rupture.

1 Cervical dilation is evident.

As part of the physical assessment, a nurse inspects a newborn for the presence of an umbilical hernia. Which infant behavior will assist the nurse in identifying this problem? 1 Crying 2 Inhaling 3 Suckling 4 Sleeping

1 Crying Increased intraabdominal pressure associated with crying, coughing, or straining will cause protrusion of the hernia.

The laboratory blood tests of a client at 10 weeks' gestation reveal that she has anemia. The client refuses iron supplements. The nurse teaches her that the best source of iron is liver. What other foods does the nurse encourage the client to eat? Select all that apply. 1 Dark leafy green vegetables 2 Legumes 3 Dried fruits 4 Broiled halibut 5 Ground beef patty

1 Dark leafy green vegetables 2 Legumes 3 Dried fruits 5 Ground beef patty Excellent food sources of iron include liver, meats, whole grain or enriched breads, dark green leafy vegetables, legumes, and dried fruits. Halibut is a good source of protein, not iron.

An adolescent with osteosarcoma is scheduled for a leg amputation in 2 days. The nurse's approach should include which action? Select one: a. Explaining that, although the amputation is difficult, it will cure the cancer b. Answering questions with straightforward honesty c. Avoiding discussing the seriousness of the condition d. Assisting the adolescent in accepting the amputation as better than a long course of chemotherapy

b. Answering questions with straightforward honesty

What intervention should the nurse implement when a 5 year old tells the nurse, "I need a Band-Aid" after having an injection. a. Ask why he/she wants a Band-Aid. b. Apply a Band-Aid. c. Explain why a Band-Aid is not needed. d. Show he/her that the bleeding has already stopped.

b. Apply a Band-Aid.

Which contraceptive method best protects against sexually transmitted infections (STIs) and human immunodeficiency virus (HIV)? a. Periodic abstinence b. Barrier methods c. Hormonal methods d. They all offer about the same protection.

b. Barrier methods

What is the most common cause of hearing impairment in children? Select one: a. Congenital ear defects b. Chronic otitis media c. Auditory nerve damage d. Congenital rubella

b. Chronic otitis media

Which interventions should the nurse include in the plan of care for the infant awaiting surgical closure of a myelomeningocele sac? Select one: a. Reinforcement of the original dressing if drainage noted b. Covered with a sterile, moist, nonadherent dressing c. A diaper secured over the dressing d. Open to air

b. Covered with a sterile, moist, nonadherent dressing

Which of the following are consequences of unrelieved pain in children? (Select all that apply) Select one or more: a. Pancreatitis b. Decreased pain threshold c. Increased blood glucose d. Altered sleep patterns e. Decreased oxygen saturation

b. Decreased pain threshold c. Increased blood glucose d. Altered sleep patterns e. Decreased oxygen saturation

What is an expected assessment finding in a child with coarctation of the aorta? Select one: a. Orthostatic hypotension b. Disparity in blood pressure between the upper and lower extremities c. Systolic hypertension in the lower extremities d. Blood pressure higher on the left side of the body

b. Disparity in blood pressure between the upper and lower extremities

The nurse is teaching the parents of a 7-year-old child who has just had a cast applied for a fractured arm with the wrist and elbow immobilized. Which instructions should be included in the teaching? Select one: a. Swelling of the fingers is to be expected for the next 48 hours. b. Elevate casted arm when resting and when sitting up. c. Allow the affected limb to hang down for 1 hour each day. d. Immobilize the shoulder to decrease pain in the arm.

b. Elevate casted arm when resting and when sitting up.

Which is a common clinical manifestation of Hodgkin disease? Select one: a. Bone and joint pain b. Enlarged, firm, nontender lymph nodes c. Painful, enlarged lymph nodes d. Petechiae

b. Enlarged, firm, nontender lymph nodes

Latasha, age 8 years, is being admitted to the hospital from the emergency department with an injury from falling off her bicycle. What intervention will help an 8 year old most in adjusting to a hospital admission? a. Orient her parents, because she is young, to her room and hospital facility. b. Explain hospital schedules such as mealtimes. c. Use terms such as "honey" and "dear" to show a caring attitude. d. Explain when parents can visit and why siblings cannot come to see her.

b. Explain hospital schedules such as mealtimes.

The parents of a child who has just been diagnosed with type 1 diabetes ask about exercise. The nurse should provide the parents with what information to address the child's safety needs? Select one: a. Exercise will increase blood glucose. b. Extra snacks are needed before exercise. c. Extra insulin is required during exercise. d. Exercise should be restricted.

b. Extra snacks are needed before exercise.

The Glasgow Coma Scale consists of an assessment of what functions? a. Level of consciousness and verbal response. b. Eye opening and verbal and motor responses. c. Intracranial pressure (ICP) and level of consciousness. d. Pupil reactivity and motor response.

b. Eye opening and verbal and motor responses.

The nurse assesses a 4-year-old who has a fine, pink, maculopapular rash that started on the face and then spread downward. Over the last three days, the rash has disappeared from the face, and is now on the extremities. Cervical and occipital lymph nodes are tender and enlarged. Which communicable disease would the nurse suspect? a. Roseola b. German measles (Rubella) c. Erythema infectiousum d. Varicella (chickenpox)

b. German measles (Rubella)

The parents of a child who has just died ask to be left alone so that they can rock their child one more time. In response to their request, what intervention should the nurse implement? Select one: a. Kindly explain that they need to say good-bye to their child now and leave. b. Grant their request. c. Discourage this because it will only prolong their grief. d. Assess why they feel that this is necessary.

b. Grant their request.

A 10 year old sustained a fracture in the epiphyseal plate of the right fibula when falling from a tree. When discussing this injury with the child's parents, the nurse should consider which statement? Select one: a. This type of fracture is inconsistent with a fall. b. Growth can be affected by this type of fracture. c. Healing is usually delayed in this type of fracture. d. This is an unusual fracture site in young children.

b. Growth can be affected by this type of fracture.

The nurse is assessing a pregnant client with type 1 diabetes about her understanding regarding changing insulin needs during pregnancy. The nurse determines that further teaching is needed if the client makes which statement? a. My insulin levels should return to pre-pregnant levels within 7-10 days if I bottle feed. b. I will need to increase my insulin during the first 3 months of pregnancy. c. Episodes of hypoglycemia are more likely to occur in the first 3 months of pregnancy d. My insulin will likely need to be increased during the second and third trimesters

b. I will need to increase my insulin during the first 3 months of pregnancy.

The predominant characteristic of the intellectual development of the child ages 2 to 7 years is egocentricity. What best describes this concept? a. Selfishness b. Inability to put self in another's place c. Self-centeredness d. Preferring to play alone

b. Inability to put self in another's place

Which statement correctly describes β-thalassemia major (Cooley's anemia)? a. All formed elements of the blood are depressed. b. Increased incidence occurs in families of Mediterranean extraction. c. Increased incidence occurs in persons of West African descent. d. Inadequate numbers of red blood cells are present.

b. Increased incidence occurs in families of Mediterranean extraction.

Which statement most accurately describes the pathologic changes of sickle cell anemia? Select one: a. Sickle-shaped cells carry excess oxygen b. Increased red blood cell destruction occurs c. Sickle-shaped cells decrease blood viscosity d. Decreased red blood cell destruction occurs

b. Increased red blood cell destruction occurs

The nurse is seeing an adolescent boy and his parents in the clinic for the first time. What should the nurse do first? a. Make the family comfortable. b. Introduce himself or herself. c. Explain the purpose of the interview. d. Give an assurance of privacy.

b. Introduce himself or herself.

A child is unconscious after a motor vehicle accident. The watery discharge from the nose tests positive for glucose. The nurse should recognize that this suggests what complication? Select one: a. Brainstem injury b. Leaking of cerebrospinal fluid (CSF) c. Upper respiratory tract infection d. Diabetic coma

b. Leaking of cerebrospinal fluid (CSF)

A woman gave birth to a healthy infant boy 5 days ago. What type of lochia would the nurse expect to find when assessing this woman? Select one: a. Lochia sangra b. Lochia serosa c. Lochia alba d. Lochia rubra

b. Lochia serosa

A child will start treatment for precocious puberty. This involves injections of which synthetic medication? Select one: a. Gonadotropins b. Luteinizing hormone-releasing hormone c. Somatotropic hormone d. Thyrotropin

b. Luteinizing hormone-releasing hormone

A child with autism spectrum disorder (ASD) is admitted to the hospital with pneumonia. The nurse should plan which priority intervention when caring for the child? Select one: a. Maintain frequent touch and eye contact with the child. b. Maintain a structured routine and keep stimulation to a minimum. c. Take the child frequently to the playroom to play with other children. d. Place the child in a room with a roommate of the same age.

b. Maintain a structured routine and keep stimulation to a minimum.

Why do infants and young children quickly have respiratory distress in acute and chronic alterations of the respiratory system? a. The gag reflex increases mucus production. b. Mucus and edema obstruct small airways. c. They have a widened, shorter airway. d. There is a defect in their sucking ability.

b. Mucus and edema obstruct small airways.

Therapeutic management of the child with acute diarrhea and dehydration usually begins with what intervention? a. Antidiarrheal medications such as paregoric b. Oral rehydration solution (ORS) c. Adsorbents such as kaolin and pectin d. Clear liquids

b. Oral rehydration solution (ORS)

A pregnant woman's amniotic membranes rupture. Prolapsed umbilical cord is suspected. What intervention would be the top priority? a. Starting oxygen by face mask. b. Placing the woman in the knee-chest position. c. Covering the cord in sterile gauze soaked in saline. d. Preparing the woman for a cesarean birth.

b. Placing the woman in the knee-chest position.

Surgical closure of the ductus arteriosus would bring about what desired effect? Select one: a. Decrease the edema in legs and feet. b. Prevent the return of oxygenated blood to the lungs. c. Stop the loss of unoxygenated blood to the systemic circulation. d. Increase the oxygenation of blood.

b. Prevent the return of oxygenated blood to the lungs.

When a preschool child is hospitalized without adequate preparation, what is the child may likely see hospitalization as? a. Threat to child's self-image b. Punishment c. An opportunity for regression d. Loss of companionship with friends

b. Punishment

The nurse is planning activity for a 4-year-old child with anemia. Which activity should the nurse plan for this child? Select one: a. Game of "hide and seek" in the children's outdoor play area b. Puppet play in the child's room c. Participation in dance activities in the playroom d. A walk down to the hospital lobby

b. Puppet play in the child's room

Because of their striving for independence and productivity, which age-group of children is particularly vulnerable to events that may lessen their feeling of control and power? a. Preschoolers b. School-age children c. Infants d. Toddlers

b. School-age children

A man and a woman want to use the calendar (rhythm) method of contraception but do not understand how it works. Based on an average 28-day cycle, during what time frame does the nurse explain that they should refrain from intercourse? 1 Days 10 to 17 of the menstrual cycle 2 7 days before the end of the menstrual cycle 3 7 days after completion of the menstrual period 4 14 days after completion of the menstrual period

1 Days 10 to 17 of the menstrual cycle Ovulation is anticipated approximately 14 days before menstruation; however, it is more reliable to avoid using a specific number of days and instead base calculations on the individual's cycle rather than the average 28-day cycle. Before relying on this method, a woman records the number of days in each menstrual cycle for at least 6 months. The first day of monthly bleeding is always counted as day 1. The woman subtracts 18 from the length of her shortest recorded cycle. This tells her the estimated first day of her fertile time. Then she subtracts 11 days from the length of her longest recorded cycle. This tells her the estimated last day of her fertile time. The couple avoids vaginal sex, or uses condoms or a diaphragm, during the fertile time. They can also use withdrawal or spermicides, but these are less effective. She updates these calculations each month, always using the 6 most recent cycles.

A nurse is assessing a toddler with vesicoureteral reflux. What clinical finding does the nurse expect to identify? 1 Dysuria 2 Oliguria 3 Glycosuria 4 Proteinuria

1 Dysuria Discomfort during urination (dysuria) is a symptom of a urinary tract infection (UTI), which is common with vesicoureteral reflux. During voiding, urine is swept up the ureters and then flows back to the bladder, resulting in a residual volume that provides a medium for the development of a UTI.

The nurse is differentiating between cephalhematoma and caput succedaneum. What finding is unique to caput succedaneum? 1 Edema that crosses the suture line 2 Scalp tenderness over the affected area 3 Edema that increases during the first day 4 Scalp over the area becomes ecchymosed

1 Edema that crosses the suture line

An adolescent girl is concerned about her body image after amputation of a leg for bone cancer. After the nurse has obtained the girl's consent, what nursing action is most therapeutic? 1 Encouraging her peers to visit 2 Keeping her lower body covered 3 Placing her in a room by herself 4 Limiting her visitors to the family

1 Encouraging her peers to visit

What should be included in a plan of care to limit the development of hyperbilirubinemia in the breastfed neonate? 1 Encouraging more frequent breastfeeding during the first 2 days 2 Instituting phototherapy for 30 minutes every 6 hours for 3 days 3 Substituting formula feeding for breastfeeding on the second day 4 Supplementing breastfeeding with glucose water during the first day

1 Encouraging more frequent breastfeeding during the first 2 days

A client asks the nurse about the use of an intrauterine device (IUD) for contraception. Which information should the nurse include in the response? Select all that apply. 1 Expulsion of the device 2 Occasional dyspareunia 3 Delay of return to fertility 4 Risk for perforation of the uterus 5 Increased number of vaginal infections

1 Expulsion of the device 2 Occasional dyspareunia 4 Risk for perforation of the uterus The presence of the IUD thread should be verified before coitus, because the device may be expelled during menses; if the IUD has been expelled, pregnancy can occur. Although dyspareunia is not common, if it does occur, it should be reported. Perforation may occur during insertion of the IUD.

What pain scale is used to measure the intensity of pain in preschoolers? 1 FACES scale 2 Visual analogue scale 3 Numerical rating scale 4 Verbal descriptor scale

1 FACES scale

Which pain scale should a nurse use to measure the intensity of pain in toddlers? 1 FACES scale 2 Visual analogue scale 3 Numerical rating scale 4 Verbal descriptor scale

1 FACES scale

The nurse is observing parents playing with their 10-month-old daughter. What should the nurse recognize as evidence that the child is developing object permanence? a. She recognizes that a ball of clay is the same when flattened out. b. She looks for the toy the parents hide under the blanket. c. She returns the blocks to the same spot on the table. d. She bangs two cubes held in her hands.

b. She looks for the toy the parents hide under the blanket.

Which term is used to describe a type of fracture that does not produce a break in the skin? Select one: a. Compound b. Simple c. Comminuted d. Complicated

b. Simple

Cystic fibrosis (CF) is suspected in a toddler. Which test is essential in establishing this diagnosis? a. Bronchoscopy b. Sweat chloride test c. Urine creatinine d. Serum calcium

b. Sweat chloride test

The nurse is caring for a client in active labor at a birthing center. She is 100% effaced, dilated 3 cm, and at +1 station. In which stage of labor is this client? 1 First 2 Latent 3 Second 4 Transitional

1 First The client is in the first stage of labor because she is fully effaced but not yet completely dilated. The first stage lasts from the onset of contractions until full cervical effacement and dilation. The second stage of labor lasts from complete dilation to birth. Latent and transition are phases and not stages of labor. Latent is the first phase of the first and second stages of labor. Transition is the last of three phases occurring in the first stage of labor.

During the assessment of a client in labor, the cervix is determined to be dilated 4 cm. What stage of labor does the nurse record? 1 First 2 Second 3 Prodromal 4 Transitional

1 First The first stage of labor is from zero cervical dilation to full cervical dilation (10 cm). The second stage is from full cervical dilation to delivery. The prodromal stage is before cervical dilation begins. The transitional phase is the first stage of labor, from 8 cm of dilation to 10 cm of dilation.

Neonates have difficulty maintaining their body temperature; however, their bodies have several mechanisms to help them do so. Which ones should a nurse remember when caring for the newborn? Select all that apply. 1 Flexed fetal position 2 Hepatic insulin stores 3 Brown fat metabolism 4 Peripheral vasoconstriction 5 Parasympathetic nervous system

1 Flexed fetal position 3 Brown fat metabolism 4 Peripheral vasoconstriction

The parent of a newborn asks, "Why do I have to scrub my baby's formula bottles?" Which information regarding the normal newborn should the nurse consider before replying in language that the parent will understand? 1 Gastric acidity is low and does not provide bacteriostatic protection 2 Absence of hydrochloric acid renders the stomach vulnerable to infection 3 Infants are almost completely lacking in immunity and require sterile fluids 4 Escherichia coli, a bacterium that is found in the stomach, does not act on milk

1 Gastric acidity is low and does not provide bacteriostatic protection

What nursing consideration is especially important when caring for a child diagnosed with juvenile idiopathic arthritis (JIA)? Select one: a. Encourage range-of-motion exercises during periods of inflammation. b. Teach child and family the correct administration of medications. c. Administer acetaminophen to reduce inflammation. d. Apply ice packs to relieve stiffness and pain.

b. Teach child and family the correct administration of medications.

A woman at 26 weeks of gestation is being assessed to determine whether she is experiencing preterm labor. What finding indicates that preterm labor is occurring? a. Irregular, mild uterine contractions are occurring every 12 to 15 minutes. b. The cervix is effacing and dilated to 2 cm. c. Estriol is not found in maternal saliva. d. Fetal fibronectin is present in vaginal secretions.

b. The cervix is effacing and dilated to 2 cm.

A woman at 26 weeks of gestation is being assessed to determine whether she is experiencing preterm labor. What finding indicates that preterm labor is occurring? a. Fetal fibronectin is present in vaginal secretions. b. The cervix is effacing and dilated to 2 cm. c. Irregular, mild uterine contractions are occurring every 12 to 15 minutes. d. Estriol is not found in maternal saliva.

b. The cervix is effacing and dilated to 2 cm.

A client is admitted to the birthing suite with a blood pressure of 150/90 mm Hg, 3+ proteinuria, and edema of the hands and face. A diagnosis of severe preeclampsia is made. What other clinical findings support this diagnosis? Select all that apply. 1 Headache 2 Constipation 3 Abdominal pain 4 Vaginal bleeding 5 Visual disturbances

1 Headache 3 Abdominal pain 5 Visual disturbances Headache in severe preeclampsia is related to cerebral edema. Abdominal pain in severe preeclampsia is related to decreased circulating blood volume and generalized edema. Visual disturbances in severe preeclampsia are related to retinal edema. Constipation and vaginal bleeding are not related to preeclampsia.

A nurse is reviewing the laboratory report of an adolescent child with nephrotic syndrome. What does the nurse expect analysis of the child's urine to reveal? 1 High protein level 2 Low specific gravity 3 Numerous red blood cells 4 Several crystalline particles

1 High protein level

What should the nurse keep in mind when planning to communicate with a child who is diagnosed with an autism spectrum disorder (ASD)? Select one: a. The child is expected to use sign language. b. The child may exhibit monotone speech and echolalia. c. The child is not listening if he/she is not looking at the nurse. d. The child has normal verbal communication.

b. The child may exhibit monotone speech and echolalia.

Which of the following are complimentary therapy methods that can be used to treat pain in an infant? (select all that apply) Select one or more: a. Magic glove b. Touch/massage c. Patterned breathing d. Sucrose e. Kangaroo care

b. Touch/massage d. Sucrose e. Kangaroo care

Methotrexate is recommended as part of the treatment plan for which obstetric complication? a. Complete hydatidiform mole b. Unruptured ectopic pregnancy c. Abruptio placentae d. Missed abortion

b. Unruptured ectopic pregnancy

The drug of choice for treatment of gonorrhea is: Select one: a. tetracycline. b. ceftriaxone. c. acyclovir. d. penicillin G.

b. ceftriaxone.

A woman is experiencing back labor and complains of intense pain in her lower back. An effective relief measure would be to use: a. conscious relaxation or guided imagery. b. counterpressure against the sacrum. c. effleurage. d. pant-blow (breaths and puffs) breathing techniques.

b. counterpressure against the sacrum.

Physiologic anemia often occurs during pregnancy as a result of: a. the fetus establishing iron stores. b. dilution of hemoglobin concentration. c. inadequate intake of iron. d. decreased production of erythrocytes.

b. dilution of hemoglobin concentration.

In the classification of newborns by gestational age and birth weight, the appropriate for gestational age (AGA) weight would: Select one: a. be modified to consider intrauterine growth restriction (IUGR). b. fall between the 10th and 90th percentiles for the infant's age. c. depend on the infant's length and the size of the head. d. fall between the 25th and 75th percentiles for the infant's age.

b. fall between the 10th and 90th percentiles for the infant's age.

A primary healthcare provider suspects ectopic pregnancy in an adolescent and conducts further evaluation. Which signs and symptoms have led the provider to suspect ectopic pregnancy? Select all that apply. 1 Hypotension 2 Abdominal pain 3 Vaginal bleeding 4 Cervical abnormalities 5 Maternal systemic illness

1 Hypotension 2 Abdominal pain 3 Vaginal bleeding An adolescent girl with hypotension and abdominal pain may have an ectopic pregnancy that has ruptured, and emergency surgery may be indicated after prompt evaluation. When an adolescent girl experiences vaginal bleeding and abdominal pain, ectopic pregnancy must be ruled out. Cervical abnormalities and systemic maternal illness may lead to spontaneous abortion, but they do not signal ectopic pregnancy.

A toddler has frequent temper tantrums. The parents ask a nurse how to limit this acting-out behavior. What should the nurse recommend? 1 Ignore the tantrum whenever possible. 2 Restrain the child whenever a tantrum begins. 3 Move the child to a quiet area as soon as a tantrum begins. 4 Visit the clinic to request medication to control the tantrum.

1 Ignore the tantrum whenever possible. Ignoring the temper tantrum as long as the child is not causing self-harm avoids reinforcement of the behavior.

An infant was born 2 hours ago at 37 weeks of gestation and weighing 4.1 kg. The infant appears chubby with a flushed complexion and is very tremulous. The tremors are most likely the result of: Select one: a. hypocalcemia. b. hypoglycemia. c. birth injury. d. seizures.

b. hypoglycemia.

Three days after birth, a breast-feeding newborn becomes jaundiced. The parents bring the infant to the clinic, and blood is drawn for an indirect serum bilirubin determination, which reveals a concentration of 12 mg/dL (100 mcmol/L). The nurse explains that the infant has physiologic jaundice. What is the cause of this benign condition? 1 Immature liver function 2 An inability to synthesize bile 3 An increased maternal hemoglobin level 4 A high hemoglobin and low hematocrit level

1 Immature liver function Jaundice occurs because of the expected physiologic breakdown of fetal red blood cells and the inability of the newborn's immature liver to conjugate the resulting bilirubin.

According to Erikson, by the time children reach their 12 birthday, they should have learned to trust others and should have developed a sense of: a. intimacy. b. industry. c. integrity. d. identity.

b. industry.

While reviewing the health history of a newborn with suspected jaundice, the nurse recalls that some risk factors place infants at a higher risk for developing jaundice. Which conditions are risk factors for jaundice? Select all that apply. 1 Infection 2 Female sex 3 Prematurity 4 Breast-feeding 5 Formula feeding 6 Maternal diabetes

1 Infection 3 Prematurity 4 Breast-feeding 6 Maternal diabetes

The most prevalent clinical manifestation of abruptio placentae (as opposed to placenta previa) is: a. cramping. b. intense abdominal pain. c. uterine activity. d. bleeding.

b. intense abdominal pain.

The parents of a 14-month-old boy with bilateral cryptorchidism ask the nurse in the pediatric clinic why it is important for him to have surgery before he is 2 years old. Before responding, the nurse takes into consideration the fact that uncorrected cryptorchidism can result in what? 1 Infertility 2 Hydrocele 3 Varicocele 4 Epididymitis

1 Infertility

A client arrives at the fertility clinic for a diagnostic workup and is told by the nurse to prepare for a Papanicolaou (Pap) test. The client states, "I do not want this test. I want to speak to the person in charge." How should the nurse respond to this statement? 1 Inform the primary healthcare provider of the client's request. 2 Encourage the client to comply with clinic procedures. 3 Express some disapproval of the client's lack of cooperation. 4 Remind the client of the importance of early cancer detection.

1 Inform the primary healthcare provider of the client's request.

Which stage of Erikson's theory would the nurse explain describes the development of a preschooler? 1 Initiative versus guilt 2 Trust versus mistrust 3 Identity versus role confusion 4 Autonomy versus sense of shame and doubt

1 Initiative versus guilt According to Erikson's theory, the initiative versus guilt stage is applicable to preschoolers between the ages of 3 to 6 years.

The nurse is caring for a client who has just had an amniotomy performed by the primary healthcare provider. The fetal heart rate immediately decreases from 140 to 80 beats/min. What is the priority nursing action? 1 Inspecting the vagina 2 Administering oxygen 3 Increasing the intravenous fluids 4 Placing the client in the knee-chest position

1 Inspecting the vagina Follow the nursing process and begin with an assessment to determine possible cause for the deceleration

The nurse is explaining insulin needs to a client with gestational diabetes who is in her second trimester of pregnancy. Which information should the nurse give to this client? 1 Insulin needs will increase during the second trimester. 2 Insulin needs will decrease during the second trimester. 3 Insulin needs will not change during the second trimester. 4 Insulin will be switched to an oral antidiabetic medication during the second trimester.

1 Insulin needs will increase during the second trimester.

While caring for a client during labor, what does the nurse remember about the second stage of labor? 1 It ends at the time of birth. 2 It ends as the placenta is expelled. = 3 It begins with the transition phase of labor. 4 It begins with the onset of strong contractions.

1 It ends at the time of birth.

A 3-year-old child is hospitalized with nephrotic syndrome. The child has oliguria and generalized edema. What factor does the nurse identify that will have the greatest effect on the child's adjustment to hospitalization? 1 Lack of parental visits 2 Inability to select a variety of foods 3 Response of peers to the edematous appearance 4 Willingness to participate in cooperative play activities

1 Lack of parental visits

What is the priority nursing intervention for the postpartum client whose fundus is three fingerbreadths above the umbilicus, boggy, and midline? 1 Massaging the uterine fundus 2 Helping the client to the bathroom 3 Assessing the peripad for the amount of lochia 4 Administering intramuscular methylergonovine (Methergine) 0.2 mg

1 Massaging the uterine fundus A uterus that is displaced and above the fundus indicates relaxation of the uterine muscle. Fundal massage is necessary to stimulate uterine contractions.

A nurse is caring for a preterm infant with necrotizing enterocolitis (NEC). Which nursing intervention is most important for this infant? 1 Measuring abdominal girth frequently 2 Diluting the formula mixture as prescribed 3 Administering oxygen before the gastric feeding 4 Using half-strength formula for gavage feeding

1 Measuring abdominal girth frequently NEC is marked by prolonged gastric emptying; an increase in abdominal girth of more than 1 cm in 4 hours is significant and requires immediate intervention. Formula feeding is stopped and parenteral fluids, usually total parenteral nutrition (TPN), are started instead

The nurse is caring for four clients on the postpartum unit. Which client will most likely state that she is having difficulty sleeping due to afterbirth pains? 1 Multipara who has vaginally delivered three children 2 Primipara whose newborn weighed 7 lb 3 Multipara with effectively controlled diabetes 4 Multipara whose second child was small for gestational age

1 Multipara who has vaginally delivered three children A multipara's uterus tends to contract and relax spasmodically, even if uterine tone is effective, resulting in pain that may require an analgesic for relief. A primipara's uterus usually remains in the contracted state unless the newborn is large for gestational age. However, she is less likely to have afterbirth pains requiring an analgesic than a multipara is

When calculating the Apgar score for a newborn, what does the nurse assess in addition to the heart rate? 1 Muscle tone 2 Amount of mucus 3 Degree of head lag 4 Depth of respirations

1 Muscle tone The five areas that are assessed when the Apgar score is calculated are heart rate, respiratory effort, muscle tone, reflex irritability, and color. The rate of respirations, not the depth, is assessed for an Apgar score. Amount of mucus, degree of head lag, and depth of respirations are not tested for an Apgar score.

An infant with a diagnosis of failure to thrive has been receiving enteral feedings for 3 days. All feedings have been retained, but the skin and mucous membranes are dry, and the infant has lost weight. What should the nurse do first in light of these findings? 1 Notify the practitioner 2 Document the assessment findings 3 Increase the fluid component in the feeding 4 Increase the calorie component of the feeding

1 Notify the practitioner Dry mucous membranes and weight loss are classic signs of dehydration. The nurse should calculate the infant's fluid requirements, then obtain a prescription from the practitioner to increase either free water or the amount of the feedings as needed.

While caring for the patient who requires an induction of labor, the nurse should be cognizant that: a. ripening the cervix usually results in a decreased success rate for induction. b. labor sometimes can be induced with balloon catheters or laminaria tents. c. amniotomy can be used to make the cervix more favorable for labor. d. oxytocin is less expensive than prostaglandins and more effective but creates greater health risks.

b. labor sometimes can be induced with balloon catheters or laminaria tents.

A community health nurse visits an infant who was born at home 24 hours ago. While assessing the infant the nurse identifies slight jaundice of the face and trunk. What should the nurse do next? 1 Obtain a stat order for a bilirubin level. 2 Plan for immediate admission to the hospital. 3 Document this expected finding in the infant's record. 4 Arrange for the infant to have phototherapy in the home.

1 Obtain a stat order for a bilirubin level. Jaundice that appears within 24 hours of birth may be indicative of a pathological process; if the bilirubin level is high, intervention is required. Physiologic jaundice does not appear until 72 hours after birth; this observation in the 24 hours after birth indicates pathologic hyperbilirubinemia.

Which hypothalamic hormone helps to treat postpartum uterine atony and hemorrhage? 1 Oxytocin 2 Indomethacin 3 Dinoprostone 4 Methylergonovine

1 Oxytocin **key word HORMONE**

A client with a diagnosis of severe preeclampsia is admitted to the hospital from the emergency department. Which precaution should the nurse institute? 1 Padding the side rails on the bed 2 Placing the call button next to the client 3 Having oxygen and a facemask available 4 Assigning a nursing assistant to stay with the client

1 Padding the side rails on the bed seizure precautions

A woman with preeclampsia has a seizure. The nurse's primary duty during the seizure is to: a. insert an oral airway. b. stay with the patient and call for help. c. administer oxygen by mask. d. suction the mouth to prevent aspiration.

b. stay with the patient and call for help.

A woman in preterm labor at 30 weeks of gestation receives two 12-mg doses of betamethasone intramuscularly. The purpose of this pharmacologic treatment is to: a. reduce maternal and fetal tachycardia associated with ritodrine administration. b. stimulate fetal surfactant production. c. suppress uterine contractions. d. maintain adequate maternal respiratory effort and ventilation during magnesium sulfate therapy.

b. stimulate fetal surfactant production

The nurse should immediately alert the physician when: Select one: a. the infant's blood glucose level is 45 mg/dL. b. the infant is dusky and turns cyanotic when crying. c. acrocyanosis is present at age 1 hour. d. the infant goes into a deep sleep at age 1 hour.

b. the infant is dusky and turns cyanotic when crying.

Spontaneous termination of a pregnancy is considered to be an abortion if: a. the products of conception are passed intact. b. the pregnancy is less than 20 weeks. c. the fetus weighs less than 1000 g. d. no evidence exists of intrauterine infection.

b. the pregnancy is less than 20 weeks

The major cause of death for children older than 1 year is: a. congenital abnormalities. b. unintentional injuries. c. infection. d. cancer.

b. unintentional injuries.

A after birth woman overhears the nurse tell the obstetrics clinician that she has a positive Homans' sign and asks what it means. The nurse's best response is: Select one: a. "You have pitting edema in your ankles." b. "You have deep tendon reflexes rated 2+." c. "You have calf pain when the nurse flexes your foot." d. "You have a 'fleshy' odor to your vaginal drainage."

c. "You have calf pain when the nurse flexes your foot."

Which interventions are included in the care plan of a postpartum client with a fourth-degree laceration? Select all that apply. 1 Pain management with oral analgesics 2 Continuous application of a warm pack 3 Assessment of the site every 15 minutes 4 Gentle cleansing with antibacterial cleanser 5 Application of an ice pack for 20-minute intervals 6 Instructing the client in how to promote normal bowel function

1 Pain management with oral analgesics 3 Assessment of the site every 15 minutes 5 Application of an ice pack for 20-minute intervals Providing pain management will prevent the client's pain from reaching an unmanageable level. Assessment of the site will identify any abnormal changes. Application of ice will decrease pain and edema. Warmth applied to newly traumatized tissue will increase pain and edema. Antibacterial cleanser would be caustic and painful to the laceration. Teaching regarding bowel function would be more appropriately presented after the client has completed the fourth stage and resumed normal intake.

The nurse is caring for the newborn of a mother with diabetes. For which signs of hypoglycemia should the nurse assess the newborn? Select all that apply. 1 Pallor 2 Irritability 3 Hypotonia 4 Ineffective sucking 5 Excessive birth weight

1 Pallor 2 Irritability 3 Hypotonia 4 Ineffective sucking

A nurse plans to talk to the parents of a toddler about toilet training. What should the nurse tell the parents is the most important factor in the process of toilet training? 1 Parents' attitude about it 2 Child's desire to remain dry 3 Child's ability to sit still on the toilet 4 Parents' willingness to work at the toilet training

1 Parents' attitude about it The parents' attitude, approach, and understanding of the child's physical and psychologic readiness are essential to letting the child proceed at his or her own pace with appropriate parental intervention.

The nurse is testing newborns' heel blood for the level of glucose. Which newborn does the nurse anticipate will experience hypoglycemia? Select all that apply. 1 Preterm infant 2 Infant with Down syndrome 3 Small-for-gestational-age infant 4 Large-for-gestational-age infant 5 Appropriate-for-gestational-age infant

1 Preterm infant 3 Small-for-gestational-age infant 4 Large-for-gestational-age infant

What should a nurse include in the plan of care for a 9-year-old child with nephrotic syndrome? 1 Providing meticulous skin care 2 Restricting fluids to 4 oz (120 mL) each shift 3 Offering a diet low in carbohydrates and protein 4 Sending blood to the laboratory for typing and crossmatching

1 Providing meticulous skin care

Which between-meal snack should a nurse tell the parents of a preschooler with a urinary tract infection to offer their child? 1 Skim milk 2 Fresh fruit 3 Hard candy 4 Cream soup

1 Skim milk A high-protein, high-carbohydrate snack provides additional nutrients to combat an infection and a fever. Also, fluid helps flush the urinary tract. Fruit does not provide the protein needed for the healing process. Candy provides empty calories. A cream soup is too heavy for a between-meal snack and does not provide the needed protein.

A woman visits the prenatal clinic because an over-the-counter pregnancy test has rendered a positive result. After the initial examination verifies the pregnancy, the nurse explains some of the metabolic changes that occur during the first trimester of pregnancy. What should the nurse include? Select all that apply. 1 Sleep needs increase 2 Urinary frequency 3 Body temperature decreases 4 Calcium requirements remain the same 5 The need for carbohydrates decreases

1 Sleep needs increase 2 Urinary frequency 4 Calcium requirements remain the same

The nurse is talking to a parent of an infant with heart failure about feeding the infant. Which statement about feeding the child is correct? Select one: a. "You should feed your baby every 2 hours." b. "You should place a nasal oxygen cannula on your infant during and after each feeding." c. "You may need to increase the caloric density of your infant's formula." d. "You may need to increase the amount of formula your infant eats with each feeding."

c. "You may need to increase the caloric density of your infant's formula."

Which information should the nurse give to a child who is to have magnetic resonance imaging (MRI) of the brain? Select one: a. "You will have to lie flat after the test is finished." b. "You will have to drink a special fluid before the test." c. "Your head will be kept from moving during the procedure." d. "You will have electrodes placed on your head with glue."

c. "Your head will be kept from moving during the procedure."

A school-age child diagnosed with leukemia experienced severe nausea and vomiting when receiving chemotherapy for the first time. Which is the most appropriate nursing action to prevent or minimize these reactions with subsequent treatments? Select one: a. Administer an antiemetic as soon as child has nausea. b. Encourage drinking large amounts of favorite fluids. c. Administer an antiemetic at least 30 minutes before chemotherapy begins. d. Encourage child to take nothing by mouth (remain NPO) until nausea and vomiting subside.

c. Administer an antiemetic at least 30 minutes before chemotherapy begins.

A child with a rhinorrhea and a low-grade fever presents at an immunization clinic to receive vaccinations. The nurse should: a. Ask the mother to reschedule with appointment for a time when the child is symptom free b. Hold the immunization and ask the mother to return when the child is well c. Administer the vaccines the child is scheduled to receive d. Administer inactivated vaccines and hold live vaccines for another date

c. Administer the vaccines the child is scheduled to receive

What is a common cause of acute diarrhea? a. Hypothyroidism b. Hirschsprung's disease c. Antibiotic therapy d. Meconium ileus

c. Antibiotic therapy

In which condition are all the formed elements of the blood simultaneously depressed? a. Thalassemia major b. Sickle cell anemia c. Aplastic anemia d. Iron deficiency anemia

c. Aplastic anemia

When infants are seen for fractures, which nursing intervention is a priority? Select one: a. No intervention is necessary. It is not uncommon for infants to fracture bones. b. Assess for genetic factors. c. Assess for child abuse. Fractures in infants are often nonaccidental. d. Assess the family's safety practices. Fractures in infants usually result from falls.

c. Assess for child abuse. Fractures in infants are often nonaccidental.

Instructions for decongestant nose drops should include what recommendation? a. Keeping drops to use again for nasal congestion. b. Administering drops after feedings and at bedtime. c. Avoiding use for more than 3 days. d. Administering drops until nasal congestion subsides.

c. Avoiding use for more than 3 days.

Which newborn reflex is elicited by stroking the lateral sole of the infant's foot from the heel to the ball of the foot? Select one: a. Plantar grasp b. Tonic neck c. Babinski d. Stepping

c. Babinski

An adolescent boy is brought to the emergency department after a motorcycle accident. His respirations are deep, periodic, and gasping. There are extreme fluctuations in blood pressure. Pupils are dilated and fixed. What type of head injury should the nurse suspect? a. Subdural hemorrhage b. Skull fracture c. Brainstem d. Epidural hemorrhage

c. Brainstem

The nurse observes some children in the playroom. Which play situation exhibits the characteristics of parallel play? a. Danielle playing with a music box on her mother's lap. b. Kimberly and Amanda sharing clay to each make things. c. Brian playing with his truck next to Kristina playing with her truck. d. Adam playing a board game with Kyle, Steven, and Erich.

c. Brian playing with his truck next to Kristina playing with her truck.

The nurse, caring for a child with acute renal failure, should recognize event as a sign of hyperkalemia? Select one: a. Dyspnea b. Seizure c. Cardiac arrhythmia d. Oliguria

c. Cardiac arrhythmia

The nurse, caring for a child with acute renal failure, should recognize which event as a sign of hyperkalemia? a. Seizure b. Oliguria c. Cardiac arrhythmia d. Dyspnea

c. Cardiac arrhythmia

A 9 year old diagnosed with Down syndrome is mainstreamed into a regular third-grade class for part of the school day. His mother asks the school nurse about programs such as Cub Scouts that he might join. The nurse's recommendation should be based on what knowledge? a. Programs such as Cub Scouts are inappropriate for children who are cognitively impaired. b. Parents of children with Down syndrome encourage programs such as scouting because they deny that their children have disabilities. c. Children with Down syndrome have the same need for socialization as other children. d. Children with Down syndrome socialize better with children who have similar disabilities.

c. Children with Down syndrome have the same need for socialization as other children.

The nurse is assessing a child with acute epiglottitis. Examining the child's throat by using a tongue depressor might precipitate which symptom or condition? a. Respiratory tract infection b. Inspiratory stridor c. Complete obstruction d. Sore throat

c. Complete obstruction

The nurse is assessing a child with acute epiglottitis. Examining the child's throat by using a tongue depressor might precipitate which symptom or condition? Select one: a. Respiratory tract infection b. Sore throat c. Complete obstruction d. Inspiratory stridor

c. Complete obstruction

The nurse is admitting a toddler with the diagnosis of juvenile hypothyroidism. Which is a common clinical manifestation of this disorder? Select one: a. Insomnia b. Accelerated growth c. Dry skin d. Diarrhea

c. Dry skin

A nurse is charting that a hospitalized child has labored breathing. Which medical term describes labored breathing? a. Tachypnea b. Hypopnea c. Dyspnea d. Orthopnea

c. Dyspnea

Chelation therapy is begun on a child with β-thalassemia major with what expected result? a. Decreasing the risk of hypoxia. b. Managing nausea and vomiting. c. Elimination of excess iron. d. Treatment of the disease.

c. Elimination of excess iron.

Which strategy would be the least appropriate for a child to use to cope? a. Learning problem solving b. Using relaxation techniques c. Having parents solve problems d. Listening to music

c. Having parents solve problems

What is best described as the inability of the heart to pump an adequate amount of blood to the systemic circulation at normal filling pressures? Select one: a. Systemic venous congestion b. Congenital heart defect c. Heart failure d. Pulmonary congestion

c. Heart failure

Which complication should the nurse asses for when caring for a child post cardiac catheterization? Select one: a. Pneumonia b. Rapidly increasing blood pressure c. Hemorrhage d. Congestive heart failure

c. Hemorrhage

Which is the most descriptive of a school-age child's reaction to death? Select one: a. Imagines the deceased person to be still alive b. Has little understanding of words such as forever c. Is very interested in funerals and burials d. Has an idealistic view of the world and criticizes funerals as barbaric

c. Is very interested in funerals and burials

Which statement best explains why iron deficiency anemia is common during toddlerhood? a. Fetal iron stores are depleted by age 1 month. b. Dietary iron cannot be started until age 12 months. c. Milk is a poor source of iron. d. Iron cannot be stored during fetal development.

c. Milk is a poor source of iron.

To help the adolescent deal with diabetes, the nurse must consider which characteristic of adolescence? Select one: a. Need to make peers aware of the seriousness of hypoglycemic reactions b. Preoccupation with the future c. Need to be perfect and similar to peers d. Desire to be unique

c. Need to be perfect and similar to peers

Four year old, placed in Buck's extension traction for Legg-Calvé-Perthes disease, is crying with pain as the nurse assesses that the skin of the right foot is pale with an absence of pulse. What should the nurse do first? Select one: a. Give the child medication to relieve the pain. b. Reposition the child and notify the physician. c. Notify the practitioner of the changes noted. d. Chart the observations and check the extremity again in 15 minutes.

c. Notify the practitioner of the changes noted.

What marks on a baby's skin may indicate an underlying problem that requires notification of a physician? Select one: a. Erythema toxicum anywhere on the body b. Telangiectatic nevi on the nose or nape of the neck c. Petechiae scattered over the infant's body d. Mongolian spots on the back

c. Petechiae scattered over the infant's body

In planning for home care of a woman with preterm labor, which concern must the nurse address? a. Restricted activity and medications will be necessary to prevent recurrence of preterm labor. b. Nursing assessments will be different from those done in the hospital setting. c. Prolonged bed rest may cause negative physiologic effects. d. Home health care providers will be necessary.

c. Prolonged bed rest may cause negative physiologic effects.

Which statement best describes why children have fewer respiratory tract infections as they grow older? Select one: a. Secondary infections rarely occur after viral illnesses. b. The amount of lymphoid tissue decreases. c. Repeated exposure to organisms causes increased immunity. d. Viral organisms are less prevalent in the population.

c. Repeated exposure to organisms causes increased immunity.

What term is used to identify the condition in which the normal adult hemoglobin is partly or completely replaced by abnormal hemoglobin? a. Aplastic anemia b. Thalassemia major c. Sickle cell anemia d. Iron deficiency anemia

c. Sickle cell anemia

What term is used to identify the condition in which the normal adult hemoglobin is partly or completely replaced by abnormal hemoglobin? Select one: a. Aplastic anemia b. Iron deficiency anemia c. Sickle cell anemia d. Thalassemia major

c. Sickle cell anemia

A woman is at 14 weeks of gestation. The nurse would expect to palpate the fundus at which level? a. Slightly above the umbilicus b. Not palpable above the symphysis at this time c. Slightly above the symphysis pubis d. At the level of the umbilicus

c. Slightly above the symphysis pubis

The nurse is administering an IV chemotherapeutic agent to a child diagnosed with leukemia. The child suddenly begins to wheeze and have severe urticaria. Which is the most appropriate nursing action? Select one: a. Explain to child that this is an expected side effect. b. Observe child closely for next 10 minutes. c. Stop drug infusion immediately. d. Recheck rate of drug infusion.

c. Stop drug infusion immediately.

When caring for a child with probable appendicitis, the nurse should be alert to recognize what sign of perforation? a. Decreased abdominal distention b. Bradycardia c. Sudden relief from pain d. Anorexia

c. Sudden relief from pain

What does the term equianalgesia refer to for a patient receiving morphine sulfate? a. The dosage of fetanyl to achieve the same effect as a dose of morphine sulfate. b. The maximum dose of morphine sulfate that can be given to achieve the desired effect. c. The amount of morphine sulfate needed to get the same effect with different routes of administration d. The combination of a non-opioid drug given along with morphine sulfate.

c. The amount of morphine sulfate needed to get the same effect with different routes of administration

What is the single most important factor to consider when communicating with children? Select one: a. The child's physical condition b. The presence or absence of the child's parent c. The child's developmental level d. The child's nonverbal behaviors

c. The child's developmental level

Which finding 12 hours after birth requires further assessment? Select one: a. The fundus is palpable at the level of the umbilicus. b. The fundus is palpable one fingerbreadth below the umbilicus. c. The fundus is palpable two fingerbreadths above the umbilicus. d. The fundus is palpable two fingerbreadths below the umbilicus.

c. The fundus is palpable two fingerbreadths above the umbilicus.

Which immunization should be given with caution to children infected with human immunodeficiency virus? Select one: a. Pneumococcus b. Inactivated poliovirus c. Varicella d. Influenza

c. Varicella (live attinuated)

A preschool child is being admitted to the hospital with dehydration and a urinary tract infection (UTI). Which urinalysis result should the nurse expect with these conditions? a. WBC >2; specific gravity 1.016 b. WBC <1; specific gravity 1.008 c. WBC >2; specific gravity 1.030 d. WBC <2; specific gravity 1.025

c. WBC >2; specific gravity 1.030

A preschool child is being admitted to the hospital with dehydration and a urinary tract infection (UTI). Which urinalysis result should the nurse expect with these conditions? Select one: a. WBC <2; specific gravity 1.025 b. WBC <1; specific gravity 1.008 c. WBC >2; specific gravity 1.030 d. WBC >2; specific gravity 1.016

c. WBC >2; specific gravity 1.030

Which teaching guideline helps prevent eye injuries during sports and play activities? a. Restrict helmet use to those who wear eyeglasses or contact lenses. b. Discourage the use of goggles with helmets. c. Wear eye protection when participating in high risk sports such as paintball. d. Wear a face mask when playing any sport or playing roughly.

c. Wear eye protection when participating in high risk sports such as paintball.

What should the nurse recommend to prevent urinary tract infections in young girls? a. Cleansing the perineum with water after voiding b. Increasing fluids; decreasing salt intake c. Wearing cotton underpants d. Limiting bathing as much as possible

c. Wearing cotton underpants

During a funduscopic examination of a school-age child, the nurse notes a brilliant, uniform red reflex in both eyes. The nurse should recognize that this is: a. a sign of a possible visual defect; the child needs vision screening. b. a sign of small hemorrhages, which usually resolve spontaneously. c. a normal finding. d. an abnormal finding; the child needs referral to an ophthalmologist.

c. a normal finding.

With regard to the after birth uterus, nurses should be aware that: Select one: a. after 2 weeks after birth it weighs 100 g. b. it returns to its original (prepregnancy) size by 6 weeks after birth. c. after 2 weeks after birth it should not be palpable abdominally. d. at the end of the third stage of labor it weighs approximately 500 g.

c. after 2 weeks after birth it should not be palpable abdominally.

A woman at 10 weeks of gestation who is seen in the prenatal clinic with presumptive signs and symptoms of pregnancy likely will have: a. positive pregnancy test. b. Chadwick's sign. c. amenorrhea. d. Hegar's sign.

c. amenorrhea.

When evaluating a patient for sexually transmitted infections (STIs), the nurse should be aware that the most common bacterial STI is: Select one: a. gonorrhea. b. candidiasis. c. chlamydia. d. syphilis.

c. chlamydia.

A child eats some sugar cubes after experiencing symptoms of hypoglycemia. This rapid-releasing sugar should be followed by: Select one: a. fruit juice. b. saturated and unsaturated fat. c. complex carbohydrate and protein. d. several glasses of water.

c. complex carbohydrate and protein.

A woman is experiencing back labor and complains of intense pain in her lower back. An effective relief measure would be to use: a. conscious relaxation or guided imagery. b. pant-blow (breaths and puffs) breathing techniques. c. counterpressure against the sacrum. d. effleurage.

c. counterpressure against the sacrum.

In planning for the care of a 30-year-old woman with pregestational diabetes, the nurse recognizes that the most important factor affecting pregnancy outcome is the: a. mother's age. b. amount of insulin required prenatally. c. degree of glycemic control during pregnancy. d. number of years since diabetes was diagnosed.

c. degree of glycemic control during pregnancy.

Excessive blood loss after childbirth can have several causes; the most common is: Select one: a. vaginal or vulvar hematomas. b. retained placental fragments. c. failure of the uterine muscle to contract firmly. d. unrepaired lacerations of the vagina or cervix.

c. failure of the uterine muscle to contract firmly.

Women with hyperemesis gravidarum: a. are a majority because 80% of all pregnant women suffer from it at some time. b. often inspire similar, milder symptoms in their male partners and mothers. c. have vomiting severe and persistent enough to cause weight loss, dehydration, and electrolyte imbalance. d. need intravenous (IV) fluid and nutrition for most of their pregnancy.

c. have vomiting severe and persistent enough to cause weight loss, dehydration, and electrolyte imbalance.

An Apgar score of 10 at 1 minute after birth would indicate a(n): Select one: a. prediction of a future free of neurologic problems. b. infant having no difficulty adjusting to extrauterine life and needing no further testing. c. infant having no difficulty adjusting to extrauterine life but who should be assessed again at 5 minutes after birth. d. infant in severe distress who needs resuscitation.

c. infant having no difficulty adjusting to extrauterine life but who should be assessed again at 5 minutes after birth.

As relates to the use of tocolytic therapy to suppress uterine activity, nurses should be aware that: a. there are no important maternal (as opposed to fetal) contraindications. b. if the patient develops pulmonary edema while receiving tocolytics, intravenous (IV) fluids should be given. c. its most important function is to afford the opportunity to administer antenatal glucocorticoids. d. the drugs can be given efficaciously up to the designated beginning of term at 37 weeks.

c. its most important function is to afford the opportunity to administer antenatal glucocorticoids.

The priority nursing care associated with an oxytocin (Pitocin) infusion is: a. measuring urinary output. b. evaluating cervical dilation. c. monitoring uterine response. d. increasing infusion rate every 30 minutes.

c. monitoring uterine response.

In administering vitamin K to the infant shortly after birth, the nurse understands that vitamin K is: Select one: a. important in the production of red blood cells. b. responsible for the breakdown of bilirubin and prevention of jaundice. c. not initially synthesized because of a sterile bowel at birth. d. necessary in the production of platelets.

c. not initially synthesized because of a sterile bowel at birth.

Nurses should know some basic definitions concerning preterm birth, preterm labor, and low birth weight. For instance: a. the terms preterm birth and low birth weight can be used interchangeably. b. low birth weight is anything below 3.7 lbs. c. preterm labor is defined as cervical changes and uterine contractions occurring between 20 and 37 weeks of pregnancy. d. in the United States early in this century, preterm birth accounted for 18% to 20% of all births.

c. preterm labor is defined as cervical changes and uterine contractions occurring between 20 and 37 weeks of pregnancy.

With shortened hospital stays, new mothers are often discharged before they begin to experience symptoms of the baby blues or after birth depression. As part of the discharge teaching, the nurse can prepare the mother for this adjustment to her new role by instructing her regarding self-care activities to help prevent after birth depression. The most accurate statement as related to these activities is to: Select one: a. keep feelings of sadness and adjustment to your new role to yourself. b. be certain that you are the only caregiver for your baby to facilitate infant attachment. c. realize that this is a common occurrence that affects many women. d. stay home and avoid outside activities to ensure adequate rest.

c. realize that this is a common occurrence that affects many women.

A woman gave birth to a 7-lb, 6-ounce infant girl 1 hour ago. The birth was vaginal, and the estimated blood loss (EBL) was approximately 1500 mL. When assessing the woman's vital signs, the nurse would be concerned to see: Select one: a. temperature 36.8° C, heart rate 60, respirations 18, BP 140/90. b. temperature 37.4° C, heart rate 88, respirations 36, BP 126/68. c. temperature 37.9° C, heart rate 120, respirations 20, blood pressure (BP) 90/50. d. temperature 38° C, heart rate 80, respirations 16, BP 110/80.

c. temperature 37.9° C, heart rate 120, respirations 20, blood pressure (BP) 90/50.

With regard to the care management of preterm labor, nurses should be aware that: a. all women must be considered at risk for preterm labor and prediction is so hit-and-miss, teaching pregnant women the symptoms probably causes more harm through false alarms. b. preterm labor is likely to be the start of an extended labor, a woman with symptoms can wait several hours before contacting the primary caregiver. c. the diagnosis of preterm labor is based on gestational age, uterine activity, and progressive cervical change. d. Braxton Hicks contractions often signal the onset of preterm labor.

c. the diagnosis of preterm labor is based on gestational age, uterine activity, and progressive cervical change.

With regard to the care management of preterm labor, nurses should be aware that: a. all women must be considered at risk for preterm labor and prediction is so hit-and-miss, teaching pregnant women the symptoms probably causes more harm through false alarms. b. preterm labor is likely to be the start of an extended labor, a woman with symptoms can wait several hours before contacting the primary caregiver. c. the diagnosis of preterm labor is based on gestational age, uterine activity, and progressive cervical change. d. Braxton Hicks contractions often signal the onset of preterm labor.

c. the diagnosis of preterm labor is based on gestational age, uterine activity, and progressive cervical change.

During the complete physical examination 24 hours after birth: Select one: a. once often neglected, blood pressure is now routinely checked. b. the parents are excused to reduce their normal anxiety. c. the nurse can gauge the neonate's maturity level by assessing the infant's general appearance. d. when the nurse listens to the heart, the S1 and S2 sounds can be heard; the first sound is somewhat higher in pitch and sharper than the second.

c. the nurse can gauge the neonate's maturity level by assessing the infant's general appearance.

In assisting the breastfeeding mother position the baby, nurses should keep in mind that: Select one: a. while supporting the head, the mother should push gently on the occiput. b. the cradle position usually is preferred by mothers who had a cesarean birth. c. whatever the position used, the infant is "belly to belly" with the mother. d. women with perineal pain and swelling prefer the modified cradle position.

c. whatever the position used, the infant is "belly to belly" with the mother.

A toddler with croup is in a cool mist tent. He is afraid and tries to climb out. What is the most appropriate action? a) tell the mother the child must stay in the tent b) place a toy in the tent to increase comfort c) administer a mild sedative d) let his mother hold him and direct the cool mist over his face

d) let his mother hold him and direct the cool mist over his face Being with the parent is something that is calming and comforting for a toddler. If we let them be with their safe space person, that helps control their fear and still allows for the cool mist to be applied.

Which of the following should be included in discharge instructions for a child with asthma? A) Keep home humidity above 50% B) use only feather pillows C) eliminate exposure to all cats and dogs d) take long-term medication even in the absence of symptoms

d) take long-term medication even in the absence of symptoms Tip: Be careful of questions that use concrete words like always, never, only, all, etc.

The nurse is conducting teaching for an adolescent being discharged to home after a renal transplantation. The adolescent needs further teaching if which statement is made? Select one: a. "I will observe my incision for any redness or swelling." b. "I won't miss doing kidney dialysis every week." c. "I will report any fever to my primary health care provider." d. "I am glad I only have to take the immunosuppressant medication for 2 weeks."

d. "I am glad I only have to take the immunosuppressant medication for 2 weeks."

A nurse is providing a parent information regarding autism spectrum disorder (ASD). Which statement made by the parent indicates understanding of the teaching? Select one: a. "Autism can be treated effectively with medication." b. "Childern with autism have hearing problems and prefer loud sounds." c. "Autism is characterized by periods of remission and exacerbation." d. "The onset of autism usually occurs before toddler stage."

d. "The onset of autism usually occurs before toddler stage."

A married couple is discussing alternatives for pregnancy prevention and has asked about fertility awareness methods (FAMs). The nurse's most appropriate reply is: a. "They're not very effective, and it's very likely you'll get pregnant." b. "These methods have a few advantages and several health risks." c. "You would be much safer going on the pill and not having to worry." d. "They can be effective for many couples, but they require motivation."

d. "They can be effective for many couples, but they require motivation."

The nurse is teaching the parent about the diet of a child experiencing severe edema associated with acute glomerulonephritis. Which information should the nurse include in the teaching? Select one: a. "Your child's diet will need an increased amount of protein." b. "Your child's diet will consist of low-fat, low-carbohydrate foods." c. "You will need to decrease the number of calories in your child's diet." d. "You will need to avoid adding salt to your child's food."

d. "You will need to avoid adding salt to your child's food."

A patient is warm and asks for a fan in her room for her comfort. The nurse enters the room to assess the mother and her infant and finds the infant unwrapped in his crib with the fan blowing over him on "high." The nurse instructs the mother that the fan should not be directed toward the newborn and the newborn should be wrapped in a blanket. The mother asks why. The nurse's best response is: Select one: a. "Your baby may lose heat by conduction, which means that he will lose heat from his body to the cooler ambient air. You should keep him wrapped and prevent cool air from blowing on him." b. "Your baby may lose heat by evaporation, which means that he will lose heat from his body to the cooler ambient air. You should keep him wrapped and prevent cool air from blowing on him." c. "Your baby will get cold stressed easily and needs to be bundled up at all times." d. "Your baby may lose heat by convection, which means that he will lose heat from his body to the cooler ambient air. You should keep him wrapped and prevent cool air from blowing on him."

d. "Your baby may lose heat by convection, which means that he will lose heat from his body to the cooler ambient air. You should keep him wrapped and prevent cool air from blowing on him."

Binocularity, the ability to fixate on one visual field with both eyes simultaneously, is normally present by what age? a. 6 to 8 months b. 12 months c. 1 month d. 3 to 4 months

d. 3 to 4 months

The nurse has received report on four children. Which child should the nurse assess first? a. An adolescent admitted after a motor vehicle accident who is oriented to person and place. b. A toddler in a persistent vegetative state with a low-grade fever. c. A school-age child in a coma with stable vital signs. d. A preschool child with a head injury and decreasing level of consciousness.

d. A preschool child with a head injury and decreasing level of consciousness.

Which condition is caused by a virus that primarily infects a specific subset of T lymphocytes, the CD4+ T-cells? Select one: a. Idiopathic thrombocytopenic purpura (ITP) b. Wiskott-Aldrich syndrome c. Severe combined immunodeficiency disease d. Acquired immunodeficiency syndrome (AIDS)

d. Acquired immunodeficiency syndrome (AIDS)

What is the most appropriate nursing diagnosis for a child diagnosed with moderate anemia? Select one: a. Risk for injury related to depressed sensorium b. Decreased cardiac output related to abnormal hemoglobin c. Risk for Injury related to dehydration and abnormal hemoglobin d. Activity intolerance related to generalized weakness

d. Activity intolerance related to generalized weakness

When should a child diagnosed with cognitive impairment be referred for stimulation and educational programs? Select one: a. At age 3 years, when schools are required to provide services. b. At age 5 or 6 years, when schools are required to provide services. c. As soon as they have the ability to communicate in some way. d. As young as possible.

d. As young as possible.

In which cultural group is good health considered to be a balance between yin and yang? a. Australian aborigines b. African-Americans c. Native Americans d. Asians

d. Asians

What is the nurse's first action when planning to teach the parents of an infant with a congenital heart defect (CHD)? Select one: a. Discuss the plan with the nursing team b. Gather literature for the parents c. Secure a quiet place for teaching d. Assess the parents' anxiety level and readiness to learn

d. Assess the parents' anxiety level and readiness to learn

Which defect results in increased pulmonary blood flow? Select one: a. Tricuspid atresia b. Pulmonic stenosis c. Transposition of the great arteries d. Atrial septal defect

d. Atrial septal defect

Which situation poses the greatest challenge to the nurse working with a child and family? a. Rehabilitation admission b. Twenty-four-hour observation c. Outpatient admission d. Emergency hospitalization

d. Emergency hospitalization

What is the term used to identify when the meatal opening is located on the dorsal surface of the penis? Select one: a. Chordee b. Phimosis c. Hypospadias d. Epispadias

d. Epispadias

What is the priority nursing intervention for a child hospitalized with hemarthrosis resulting from hemophilia? Select one: a. Assessment of the impact of hospitalization on the family system b. Assessment of the child's response to hospitalization c. Administration of acetaminophen for pain relief d. Immobilization and elevation of the affected joint

d. Immobilization and elevation of the affected joint

Which statement is most descriptive of a concussion? a. A slight lesion develops remote from the site of trauma. b. Petechial hemorrhages cause amnesia. c. Visible bruising and tearing of cerebral tissue occur. d. It is a transient, reversible neuronal dysfunction.

d. It is a transient, reversible neuronal dysfunction.

What should the nurse consider when preparing a school-age child and the family for heart surgery? Select one: a. Not showing unfamiliar equipment b. Explaining that an endotracheal tube will not be needed if the surgery goes well c. Avoiding mentioning postoperative discomfort and interventions d. Letting child hear the sounds of an electrocardiograph monitor

d. Letting child hear the sounds of an electrocardiograph monitor

A laboring woman received an opioid agonist (meperidine) intravenously 90 minutes before she gave birth. Which medication should be available to reduce the postnatal effects of Demerol on the neonate? a. Fentanyl (Sublimaze) b. Promethazine (Phenergan) c. Nalbuphine (Nubain) d. Naloxone (Narcan)

d. Naloxone (Narcan)

The nurse, closely monitoring a child who is unconscious after a fall, notices that the child suddenly has a fixed and dilated pupil. The nurse should interpret this as the indication of what occurrence? Select one: a. Eye trauma b. Indication of brain death c. Severe brainstem damage d. Neurosurgical emergency

d. Neurosurgical emergency

A woman gave birth to an infant boy 10 hours ago. Where would the nurse expect to locate this woman's fundus? Select one: a. Midway between the umbilicus and the symphysis pubis b. Nonpalpable abdominally c. Two centimeters below the umbilicus d. One centimeter above the umbilicus

d. One centimeter above the umbilicus

The nurse is caring for an infant diagnosed with congestive heart disease (CHD). The nurse should plan which intervention to decrease cardiac demands? Select one: a. Discourage parents from rocking the infant b. Allow the infant to sleep through feedings during the night c. Wait for the infant to cry to show definite signs of hunger d. Organize nursing activities to allow for uninterrupted sleep

d. Organize nursing activities to allow for uninterrupted sleep

As part of their teaching function at discharge, nurses should educate parents regarding safe sleep. Which statement is incorrect? Select one: a. Prevent exposure to people with upper respiratory tract infections. b. Avoid loose bedding, water beds, and beanbag chairs. c. Keep the infant away from secondhand smoke. d. Place the infant on his or her abdomen to sleep.

d. Place the infant on his or her abdomen to sleep.

The nurse is planning activity for a 4-year-old child with anemia. Which activity should the nurse plan for this child? a. Game of "hide and seek" in the children's outdoor play area b. A walk down to the hospital lobby c. Participation in dance activities in the playroom d. Puppet play in the child's room

d. Puppet play in the child's room

A first-time dad is concerned that his 3-day-old daughter's skin looks "yellow." In the nurse's explanation of physiologic jaundice, what fact should be included? Select one: a. This condition is also known as "breast milk jaundice." b. Physiologic jaundice is caused by blood incompatibilities between the mother and infant blood types. c. Physiologic jaundice occurs during the first 24 hours of life. d. The bilirubin levels of physiologic jaundice peak between 72 to 96 hours of life.

d. The bilirubin levels of physiologic jaundice peak between 72 to 96 hours of life.

When nurses help their expectant mothers assess the daily fetal movement counts, they should be aware that: a. "Kick counts" should be taken every half hour and averaged every 6 hours, with every other 6-hour stretch off". b. Obese mothers familiar with their bodies can assess fetal movement as well as average-size women. c. Alcohol or cigarette smoke can irritate the fetus into greater activity. d. The fetal alarm signal should go off when fetal movements stop entirely for 12 hours.

d. The fetal alarm signal should go off when fetal movements stop entirely for 12 hours.

What should the nurse include in a teaching plan for the parents of a child with vesicoureteral reflux? a. The need for the child to hold urine for 6 to 8 hours b. The use of bubble baths as an incentive to increase bath time c. Suggestions for how to maintain fluid restrictions d. The importance of taking prophylactic antibiotics

d. The importance of taking prophylactic antibiotics

A woman gave birth vaginally to a 9-lb, 12-ounce girl yesterday. Her primary health care provider has written orders for perineal ice packs, use of a sitz bath tid, and a stool softener. What information is most closely correlated with these orders? Select one: a. The woman received epidural anesthesia. b. The woman had a vacuum-assisted birth. c. The woman is a gravida 2, para 2. d. The woman has an episiotomy.

d. The woman has an episiotomy.

Which of the following statements is true regarding factors that influence pain? a. Expectation about how painful an event will be does not affect how much pain is felt b. Anxiety affects the level of pain only if the anxiety is about the event causing the pain c. Using restraints can help decrease pain during a procedure d. Use of a magic blanket is a pain control strategy the would assist an 8-year-old

d. Use of a magic blanket is a pain control strategy the would assist an 8-year-old

Which action is most likely to encourage parents to talk about their feelings related to their child's illness? a. Use direct questions. b. Be sympathetic. c. Avoid periods of silence. d. Use open-ended questions.

d. Use open-ended questions.

The mother of a 1-month-old infant tells the nurse that she worries that her baby will get meningitis like her oldest son did when he was an infant. On what information should the nurse's response be based upon? a. Meningitis rarely occurs during infancy. b. Vaccination to prevent all types of meningitis is now available. c. Often a genetic predisposition to meningitis is found. d. Vaccination to prevent Haemophilus influenzae type B meningitis has decreased the frequency of this disease in children.

d. Vaccination to prevent Haemophilus influenzae type B meningitis has decreased the frequency of this disease in children.

Which symptom is considered a warning sign and should be reported immediately by the pregnant woman to her health care provider? a. Nausea with occasional vomiting b. Urinary frequency c. Fatigue d. Vaginal bleeding

d. Vaginal bleeding

Which statement best describes a myelomeningocele? a. Herniation of the brain and meninges through a defect in the skull. b. Fissure in the spinal column that leaves the meninges and the spinal cord exposed. c. Hernial protrusion of a sac-like cyst of meninges with spinal fluid but no neural elements. d. Visible defect with an external sac-like protrusion containing meninges, spinal fluid, and nerves.

d. Visible defect with an external sac-like protrusion containing meninges, spinal fluid, and nerves.

What major complication is noted in a child with chronic renal failure? Select one: a. Metabolic alkalosis b. Excessive excretion of blood urea nitrogen c. Hypokalemia d. Water and sodium retention

d. Water and sodium retention

A woman gave birth 48 hours ago to a healthy infant girl. She has decided to bottle-feed. During your assessment you notice that both of her breasts are swollen, warm, and tender on palpation. The woman should be advised that this condition can best be treated by: Select one: a. wearing a loose-fitting bra to prevent nipple irritation. b. expressing small amounts of milk from the breasts to relieve pressure. c. running warm water on her breasts during a shower. d. applying ice to the breasts for comfort.

d. applying ice to the breasts for comfort.

The nurse is taking a sexual history on an adolescent girl. The best way to determine whether she is sexually active is to: Select one: a. ask both the girl and her parent if she is sexually active. b. ask her, "Are you sexually active?" c. ask her, "Are you having sex with a boyfriend?" d. ask her, "Are you having sex with anyone?"

d. ask her, "Are you having sex with anyone?"

The nurse caring for the after birth woman understands that breast engorgement is caused by: Select one: a. hyperplasia of mammary tissue. b. accumulation of milk in the lactiferous ducts and glands. c. overproduction of colostrum. d. congestion of veins and lymphatics.

d. congestion of veins and lymphatics.

The process in which bilirubin is changed from a fat-soluble product to a water-soluble product is known as: Select one: a. unconjugation of bilirubin. b. enterohepatic circuit. c. albumin binding. d. conjugation of bilirubin.

d. conjugation of bilirubin.

The type of injury a child is especially susceptible to at a specific age is most closely related to: a. number of responsible adults in the home. b. educational level of the child. c. physical health of the child. d. developmental level of the child.

d. developmental level of the child.

An 8-year-old girl asks the nurse how the blood pressure apparatus works. The most appropriate nursing action is to: Select one: a. ask her why she wants to know. b. determine why she is so anxious. c. tell her she will see how it works as it is used. d. explain in simple terms how it works.

d. explain in simple terms how it works.

A pregnant woman's diet may not meet her need for folates. A good source of this nutrient is: a. chicken. b. potatoes. c. cheese. d. green leafy vegetables.

d. green leafy vegetables.

Informed consent concerning contraceptive use is important because some of the methods: a. may not be reliable. b. are invasive procedures that require hospitalization. c. require a surgical procedure to insert. d. have potentially dangerous side effects.

d. have potentially dangerous side effects.

The nurse practicing in the perinatal setting should promote kangaroo care regardless of an infant's gestational age. This intervention: Select one: a. helps infants with motor and central nervous system impairment. b. is adopted from classical British nursing traditions. c. gets infants ready for breastfeeding. d. helps infants to interact directly with their parents and enhances their temperature regulation.

d. helps infants to interact directly with their parents and enhances their temperature regulation.

As relates to the use of tocolytic therapy to suppress uterine activity, nurses should be aware that: a. if the patient develops pulmonary edema while receiving tocolytics, intravenous (IV) fluids should be given. b. there are no important maternal (as opposed to fetal) contraindications. c. the drugs can be given efficaciously up to the designated beginning of term at 37 weeks. d. its most important function is to afford the opportunity to administer antenatal glucocorticoids.

d. its most important function is to afford the opportunity to administer antenatal glucocorticoids.

While caring for the patient who requires an induction of labor, the nurse should be cognizant that: a. oxytocin is less expensive than prostaglandins and more effective but creates greater health risks. b. ripening the cervix usually results in a decreased success rate for induction. c. amniotomy can be used to make the cervix more favorable for labor. d. labor sometimes can be induced with balloon catheters or laminaria tents.

d. labor sometimes can be induced with balloon catheters or laminaria tents.

A steady trickle of bright red blood from the vagina in the presence of a firm fundus suggests: Select one: a. uterine atony. b. infection of the uterus. c. perineal hematoma. d. lacerations of the genital tract.

d. lacerations of the genital tract.

A maternal indication for the use of forceps is: a. a history of rapid deliveries. b. failure to progress past 0 station. c. a wide pelvic outlet. d. maternal exhaustion.

d. maternal exhaustion.

A macrosomic infant is born after a difficult forceps-assisted delivery. After stabilization the infant is weighed, and the birth weight is 4550 g (9 lbs, 6 ounces). The nurse's most appropriate action is to: Select one: a. leave the infant in the room with the mother. b. take the infant immediately to the nursery. c. perform a gestational age assessment to determine whether the infant is large for gestational age. d. monitor blood glucose levels frequently and observe closely for signs of hypoglycemia.

d. monitor blood glucose levels frequently and observe closely for signs of hypoglycemia.

A woman in labor has just received an epidural block. The most important nursing intervention is to: a. monitor the maternal pulse for possible bradycardia. b. monitor the fetus for possible tachycardia. c. limit parenteral fluids. d. monitor the maternal blood pressure for possible hypotension.

d. monitor the maternal blood pressure for possible hypotension.

The first and most important nursing intervention when a nurse observes profuse after birth bleeding is to: Select one: a. call the woman's primary health care provider. b. assess maternal blood pressure and pulse for signs of hypovolemic shock. c. administer the standing order for an oxytocic. d. palpate the uterus and massage it if it is boggy.

d. palpate the uterus and massage it if it is boggy.

Question text The nurse must assess a child's capillary refilling time. This can be accomplished by: Select one: a. inspecting the chest. b. auscultating the heart. c. palpating the apical pulse. d. palpating the skin to produce a slight blanching

d. palpating the skin to produce a slight blanching

When teaching parents about mandatory newborn screening, it is important for the nurse to explain that the main purpose is to: Select one: a. keep the state records updated. b. allow accurate statistical information. c. document the number of births. d. recognize and treat newborn disorders early.

d. recognize and treat newborn disorders early.

The two primary areas of risk for sexually transmitted infections (STIs) are: Select one: a. large number of sexual partners and race. b. sexual orientation and socioeconomic status. c. age and educational level. d. risky sexual behaviors and inadequate preventive health behaviors.

d. risky sexual behaviors and inadequate preventive health behaviors.

With regard to a pregnant woman's anxiety and pain experience, nurses should be aware that: a. even mild anxiety must be treated. b. women who have had a painful labor will have learned from the experience and have less anxiety the second time because of increased familiarity. c. anxiety may increase the perception of pain, but it does not affect the mechanism of labor. d. severe anxiety increases tension, which increases pain, which in turn increases fear and anxiety, and so on.

d. severe anxiety increases tension, which increases pain, which in turn increases fear and anxiety, and so on.

With regard to a pregnant woman's anxiety and pain experience, nurses should be aware that: a. even mild anxiety must be treated. b. women who have had a painful labor will have learned from the experience and have less anxiety the second time because of increased familiarity. c. anxiety may increase the perception of pain, but it does not affect the mechanism of labor. d. severe anxiety increases tension, which increases pain, which in turn increases fear and anxiety, and so on.

d. severe anxiety increases tension, which increases pain, which in turn increases fear and anxiety, and so on.

As related to the normal functioning of the renal system in newborns, nurses should be aware that: Select one: a. breastfed infants likely will void more often during the first days after birth. b. "Brick dust" or blood on a diaper is always a cause to notify the physician. c. weight loss from fluid loss and other normal factors should be made up in 4 to 7 days. d. the pediatrician should be notified if the newborn has not voided in 24 hours.

d. the pediatrician should be notified if the newborn has not voided in 24 hours.

A 25-year-old gravida 2, para 2-0-0-2 gave birth 4 hours ago to a 9-lb, 7-ounce boy after augmentation of labor with Pitocin. She puts on her call light and asks for her nurse right away, stating, "I'm bleeding a lot." The most likely cause of after birth hemorrhage in this woman is: a. unrepaired vaginal lacerations. b. puerperal infection. c. retained placental fragments. d. uterine atony.

d. uterine atony.

The perinatal nurse is caring for a woman in the immediate postbirth period. Assessment reveals that the woman is experiencing profuse bleeding. The most likely etiology for the bleeding is: Select one: a. vaginal hematoma. b. vaginal laceration. c. uterine inversion. d. uterine atony.

d. uterine atony.

A 2-year-old child is brought to the emergency department after the sudden onset of high fever, drooling, and respiratory distress. What nursing actions should the nurse perform? Select all that apply. 1 Start an intravenous line. 2 Draw a blood sample for a complete blood count and differential. 3 Examine the child's throat with a flashlight and tongue depressor for swelling. 4 Assess oxygen saturation of the blood and administer oxygen by mask if it is below 94%. 5 Ask the parents to remain in the waiting room during the examination and interventions. 6 Assess the child's temperature and administer an antipyretic if the rectal temperature is higher than 101° F (38.3° C).

1 Start an intravenous line. 2 Draw a blood sample for a complete blood count and differential. 4 Assess oxygen saturation of the blood and administer oxygen by mask if it is below 94%. 6 Assess the child's temperature and administer an antipyretic if the rectal temperature is higher than 101° F (38.3° C). The child is presenting with signs of epiglottitis. An intravenous line will prevent dehydration and provide access for emergency medications if needed. Oxygen saturation should be kept above 94% to prevent hypoxia. Determination of the cause of the epiglottis (bacterial versus viral) will be necessary to manage the illness. Children in this age range can have febrile seizures; antipyretics will control the temperature.

The electronic fetal monitor displays contractions every 2 minutes and lasting 95 seconds. What is the nurse's highest priority intervention at this time? 1 Stop the oxytocin (Pitocin) infusion. 2 Administer oxygen at 8 to 10 L/min. 3 Increase the main line fluid delivery rate to 150 mL/hr. 4 Prepare the client for insertion of an intrauterine pressure catheter.

1 Stop the oxytocin (Pitocin) infusion. The contraction pattern indicates hyperstimulation of the uterus. Stopping the oxytocin infusion permits relaxation of the uterus and perfusion of the placenta.

A 6-year-old child with acute spasmodic bronchitis who is receiving humidified air removes the mask, and while bathing the child the nurse notes increasing respiratory distress. What is the most appropriate nursing intervention? 1 Stopping the bath and replacing the mask 2 Performing postural drainage and clapping the chest 3 Placing the child in the orthopneic position and calling the practitioner 4 Suctioning the child's nasal passages and waiting for the dyspnea to subside

1 Stopping the bath and replacing the mask

A nurse explains to a nursing class that the efficiency of the basal body temperature method of contraception depends on fluctuation of the basal body temperature. Which factor can alter the effectiveness of this method? 1 Stress 2 Length of abstinence 3 Age of those involved 4 Frequency of intercourse

1 Stress

A nurse is performing a respiratory assessment of an 8-month-old child with the diagnosis of viral pneumonia. The nurse identifies bronchial breath sounds over areas of consolidation, mild substernal retractions, profuse mucus production, pallor, and a temperature of 102° F (38.9° C). What is the priority nursing action? 1 Suctioning the nasopharynx so a patent airway can be maintained 2 Starting an intravenous infusion to provide necessary fluids and electrolytes 3 Calling the respiratory therapist to start preparations for oxygen administration 4 Notifying the practitioner of the fever so a prescription for an antipyretic can be issued

1 Suctioning the nasopharynx so a patent airway can be maintained

A client is admitted to the birthing unit because fluid is leaking from her vagina. She is unsure whether her "bag of water" has broken. What should the nurse do to help determine whether the fluid is amniotic fluid? 1 Test the fluid with Nitrazine paper. 2 Inspect the fluid for its characteristics. 3 Assess the fluid for the presence of protein. 4 Send the fluid to the laboratory for analysis.

1 Test the fluid with Nitrazine paper. Amniotic fluid is slightly alkaline, and urine is acidic; when moistened with amniotic fluid, Nitrazine will turn dark blue, indicating an alkaline substance.

A young sexually active client at the family planning clinic is advised to have a Papanicolaou (Pap) smear. She has never had a Pap smear before. What information should the nurse include in the explanation of this procedure? 1 The Pap smear can detect cancer of the cervix. 2 Vaginal bleeding is expected after a Pap smear. 3 Colposcopy will be used to visualize the cervix. 4 Scraping the cervix is the most uncomfortable part.

1 The Pap smear can detect cancer of the cervix.

A woman at 40 weeks' gestation is having contractions. Wondering whether she is in true labor, she asks, "How will you know if I'm really in labor?" Which information should the nurse provide to the client at this time? 1 The cervix dilates and becomes effaced in true labor. 2 Bloody show is the first sign of true labor. 3 The membranes rupture at the beginning of true labor. 4 Fetal movements lessen and become weaker in true labor.

1 The cervix dilates and becomes effaced in true labor. The major difference between true and false labor is that true labor can be confirmed by the presence of dilation and effacement of the cervix.

The mother of a 5-year-old girl child reports to a nurse that her daughter has a genital discharge and recurrent urinary tract infections. What should the nurse suspect from the mother's statement? 1 The child may be a victim of sexual abuse. 2 The child may be a victim of physical abuse. 3 The child may be a victim of physical neglect. 4 The child may be a victim of emotional neglect.

1 The child may be a victim of sexual abuse.

A client who is visiting the prenatal clinic for the first time has a serology test for toxoplasmosis. What information about the client's activities in the history indicates to the nurse that there is a need for this test? 1 The client cares for a neighbor's cat 2 The client works as a dog trainer 3 The client uses chemical cleaners 4 The client consumes raw vegetables

1 The client cares for a neighbor's cat

A client who is visiting the prenatal clinic for the first time has a serology test for toxoplasmosis. What information about the client's activities in the history indicates to the nurse that there is a need for this test? 1 The client cares for a neighbor's cat 2 The client works as a dog trainer 3 The client uses chemical cleaners 4 The client consumes raw vegetables

1 The client cares for a neighbor's cat Toxoplasmosis is caused by a protozoal parasite; cats acquire the organism by ingesting infected mice or birds, and the cysts are found in their feces. Caring for or working with cats, not dogs, poses a potential problem with toxoplasmosis. Chemical cleaners may be teratogenic, but they do not cause toxoplasmosis.

A female client asks a nurse about using an intrauterine device (IUD) for contraception. When explaining this method, what common problem should the nurse include in the discussion? 1 The device can be expelled. 2 The uterus may be perforated. 3 Discomfort during intercourse may occur. 4 Vaginal infections are frequent consequences.

1 The device can be expelled.

Laboratory studies reveal that a pregnant client's blood type is O, and she is Rh positive. The client asks whether her newborn will have a problem with blood incompatibility. Before responding, the nurse must remember that fetal problems may develop in what circumstance? 1 The fetus has type A or B blood. 2 The fetus is born preterm. 3 The fetus has type O, Rh positive blood. 4 The mother has diabetes.

1 The fetus has type A or B blood. ABO incompatibility may develop even in a firstborn infant. The mother has antibodies against antigens of the A and B blood cells. These antibodies, which are transferred across the placenta, produce hemolysis of fetal red blood cells. If the fetus is type A, B, or AB, incompatibility may occur.

A client has delivered her infant by cesarean birth. The nurse monitors the newborn's respiration closely, because infants born via the cesarean method are prone to atelectasis. Why does this occur? 1 The ribcage is not compressed and released during birth. 2 The sudden temperature change at birth causes aspiration. 3 There is usually oxygen deprivation after a cesarean birth. 4 There is no gravity during the birth to promote drainage from the lungs.

1 The ribcage is not compressed and released during birth.

At 18 months of age a child born with a cleft lip and palate is readmitted for palate surgery. Why does the nurse teach the parents not to brush their child's teeth immediately after the surgery? 1 The suture line might be injured. 2 A toothbrush might be frightening. 3 The child will probably have no teeth. 4 A toothbrush has not been used before.

1 The suture line might be injured.

The nurse is providing care to a preschool-age client of Asian descent during a scheduled health maintenance visit. The family speaks fluent English. Which assessment strategies should the nurse implement with the child and family based on the current data? Select all that apply. 1 Using open-ended questions 2 Avoiding prolonged eye contact 3 Phrasing questions in a neutral manner 4 Asking all questions directly to the interpreter 5 Asking several questions for time management purposes

1 Using open-ended questions 2 Avoiding prolonged eye contact 3 Phrasing questions in a neutral manner

A newborn with acquired herpes simplex virus infection is being discharged. Which facet of development should the nurse instruct the parents to monitor closely? 1 Visual clarity 2 Renal function 3 Long bone growth 4 Responses to sounds

1 Visual clarity

What should a nurse incorporate into the plan of care for a school-aged child hospitalized with acute glomerulonephritis (AGN)? Select all that apply. 1 Weighing daily 2 Restricting fluids 3 Monitoring intravenous therapy 4 Instituting isolation precautions 5 Checking the blood pressure hourly

1 Weighing daily 2 Restricting fluids Comparing daily weights is an objective measure of fluid balance and response to diuretic therapy. Fluids, as well as sodium, are restricted in the presence of oliguria. Intravenous therapy is not needed unless there is an emergency. Isolation is unnecessary because the illness is not communicable. Although the blood pressure is closely monitored, it need not be taken hourly.

The nurse is preparing to discharge a 3-day-old infant who weighed 7 lb (3175 g) at birth. Which finding should be reported immediately to the healthcare provider? 1 Weight of 6 lb 4 oz (2835 g) 2 Hemoglobin of 16.2 g/dL (162 mmol/L) 3 Three wet diapers over the last 12 hours 4 Total serum bilirubin of 10 mg/dL (171 µmol/L)

1 Weight of 6 lb 4 oz (2835 g) A loss of 12 oz (340 g) since birth, or more than 10%, is higher than the acceptable figure of 5% to 6%

A nurse in the pediatric intensive care unit is assessing a 6-month-old infant with bronchiolitis. What physiologic responses to this lower respiratory tract infection does the nurse expect? Select all that apply. 1 Wheezing 2 Bradycardia 3 Sternal retractions 4 Nasal flaring 5 Prolonged expiratory phase

1 Wheezing 3 Sternal retractions 4 Nasal flaring 5 Prolonged expiratory phase Bronchiolitis in most infants is caused by respiratory syncytial virus. Wheezing occurs as the air passages narrow, resulting in the typical whistling sound. As breathing becomes more difficult, the infant must expend more energy and use accessory muscles of respiration to breathe. Nasal flaring is a predominant characteristic of bronchiolitis. The infectious and inflammatory changes narrow the bronchial passage, making it difficult for air to leave the lungs. As a result of increased respiratory effort and decreased oxygen exchange, tachycardia, not bradycardia, develops. Breath sounds are diminished because of edema of the bronchiolar mucosa and filling of the lumina with mucus and exudate.

Name three factors that put an infant or child with bronchiolitis at risk for hospital admission?

1) Someone who is more symptomatic (tachycardia, tachypneic) 2)) Underlying health issues, esp cardiac or respiratory 3) Dehydrated 4) Age- the younger they are esp if less than 6mos or if they were born premature

A primigravida at 39 weeks' gestation is admitted to the high-risk unit with an acute infection and is to have labor induced. In what sequence should the nurse implement the primary healthcare provider's prescriptions? 1. Initiate monitoring with an electronic fetal/maternal monitor. 2. Start oxytocin 30 units in 1000 mL of D5W per protocol. 3. Give the client a 2-g loading dose of ampicillin, followed by 1 g every 4 hours. 4. Call the anesthesia department to evaluate the client for an epidural.

1, 3, 2, 4

The nurse is providing care to an infant diagnosed with Down syndrome. Which parental statement related to the infant's growth indicates the need for further education? 1 "My baby will have growth deficiencies during infancy." 2 "My child will have accelerated growth during adolescence." 3 "My child will most likely be overweight by 3 years of age." 4 "My baby will have reduced growth in both height and weight."

2 "My child will have accelerated growth during adolescence."

The nurse is providing anticipatory guidance to the parents of a child who has a difficult temperament. Which statement should the nurse include in the teaching session with these parents regarding their child's temperament? Select all that apply. 1 "Your child will develop predictable habits." 2 "Your child will benefit from a bedtime routine." 3 "Your child is open to change and adapts easily." 4 "Your child is likely to adapt slowly to the new daycare setting." 5 "Your child will display intense emotions, including those associated with happiness."

2 "Your child will benefit from a bedtime routine." 4 "Your child is likely to adapt slowly to the new daycare setting." 5 "Your child will display intense emotions, including those associated with happiness."

A neonate weighing 5 lb 6 oz (2438 g) is born in a cesarean birth and admitted to the newborn nursery. What range of resting respiratory rate should the nurse anticipate? 1 20 to 40 breaths/min 2 30 to 60 breaths/min 3 60 to 80 breaths/min 4 70 to 90 breaths/min

2 30 to 60 breaths/min

A mother is breastfeeding her newborn. She asks when she may switch the baby to a cup. The nurse concludes that the mother understands the teaching about feeding when she says she will start to introduce a cup after the baby reaches what age? 1 3 months 2 6 months 3 12 months 4 16 months

2 6 months

A couple interested in delaying the start of a family discuss the various methods of family planning. Together they decide to use the basal body temperature method. The nurse explains that the fertile period surrounding ovulation lasts from when to when? 1 12 hours before to 24 hours after ovulation 2 72 hours before to 24 hours after ovulation 3 72 to 80 hours before to 72 hours after ovulation 4 24 to 48 hours before to 48 hours after ovulation

2 72 hours before to 24 hours after ovulation

While performing a newborn assessment after a vaginal birth, a student nurse observes a swelling on one side of the top of the head that does not cross the suture line. The student nurse has identified what clinical manifestation? 1 A bulging fontanel 2 A cephalhematoma 3 Caput succedaneum 4 Normal molding pattern

2 A cephalhematoma A cephalhematoma is a collection of blood between the skull bone and its periosteum that results from trauma during birth. It resolves spontaneously in 3 to 6 weeks.

A severely dehydrated infant with gastroenteritis is admitted to the pediatric unit. Nothing-by-mouth (NPO) status is prescribed. The parents ask why their baby cannot be fed. The nurse explains that it is necessary to do what? 1 Correct electrolyte imbalances 2 Allow the intestinal tract to rest 3 Determine the cause of the diarrhea 4 Prevent perianal irritation from the diarrhea

2 Allow the intestinal tract to rest

During a pelvic examination of a 24-year-old woman, the nurse suspects a vaginal infection because of the presence of a white curdlike vaginal discharge. What other finding supports a fungal vaginal infection? 1 A foul odor 2 An itchy perineum 3 An ischemic cervix 4 A forgotten tampon

2 An itchy perineum An itchy perineum usually occurs with candidiasis, a fungal infection; pruritus is the most common symptom.

While changing a newborn girl's diaper a nurse observes a brick-red stain on the diaper. How does the nurse interpret this clinical finding? 1 A sign of low iron excretion 2 An uncommon benign occurrence 3 An expected occurrence in female newborns 4 The result of a medication administered during labor

2 An uncommon benign occurrence The brick-red color in the urine is caused by albumin and urates that are found in the first week of life. Iron is eliminated by way of the gastrointestinal tract. The finding is unrelated to the sex of the infant; it is not hormonally based. No medication administered during labor will cause this discoloration.

A client tells the nurse that the first day of her last menstrual period was July 22. What is the estimated date of birth (EDB)? 1 May 7 2 April 29 3 April 22 4 March 6

2 April 29 EDB = last menstrual period + 1 year - 3 months + 7 days.

A parent tearfully tells a nurse, "They think our toddler is developmentally delayed. We're investigating a preschool program for cognitively impaired children." What is the most appropriate response by the nurse? 1 Praising the parent for the decision and encouraging the plan 2 Asking for more specific information related to the developmental delays 3 Advising the parent to have the healthcare provider help choose an appropriate program 4 Explaining that this action may be premature and that the developmental delays could disappear

2 Asking for more specific information related to the developmental delays More information is needed. The term developmental delay suggests that some milestones for age are not being met at the average time; it is not synonymous with cognitive impairment.

A primigravida is admitted to the birthing unit in early labor. A pelvic examination reveals that her cervix is 100% effaced and dilated 3 cm. The fetal head is at +1 station. In which area of the client's pelvis is the fetal occiput? 1 Not yet engaged 2 Below the ischial spines 3 Entering the pelvic inlet 4 Visible at the vaginal opening

2 Below the ischial spines A station of +1 indicates that the fetal head is 1 cm below the ischial spines. The head is now past the points of engagement, the ischial spines. When the head is entering the pelvic inlet, it is said to be at 0 station. The head must be at +3 to +4 station to be visible at the vaginal opening.

A nurse is caring for a preterm neonate with physiologic jaundice who requires phototherapy. What is the physiologic mechanism of this therapy? 1 Stimulates the liver to dispose of the bilirubin 2 Breaks down the bilirubin into a conjugated form 3 Facilitates the excretion of bilirubin by activating vitamin K 4 Dissolves the bilirubin, allowing it to be excreted by the skin

2 Breaks down the bilirubin into a conjugated form Phototherapy changes unconjugated bilirubin in the skin to conjugated bilirubin bound to protein, permitting excretion in the urine and feces.

A nurse teaches a woman who is planning to breastfeed how to relieve breast engorgement. The nurse determines that further teaching is necessary when the woman states that she will do what? 1 Manually express breast milk 2 Breastfeed the infant less frequently 3 Apply warm compresses to both breasts 4 Place cold compresses on the breasts just after breastfeeding

2 Breastfeed the infant less frequently

A nurse practitioner prescribes doxycycline for a sexually active woman with a history of mucopurulent discharge and bleeding associated with cervical dysplasia, dysuria, and dyspareunia. With which sexually transmitted infection are these clinical findings and medication therapy commonly associated? 1 Herpes simplex 2 2 Chlamydial infection 3 Treponema pallidum 4 Neisseria gonorrhoeae

2 Chlamydial infection The signs and symptoms listed and the treatment ordered (doxycycline or azithromycin) indicate that the client has a chlamydial infection.

After the removal of a hydatidiform mole, the nurse assesses the client's laboratory data during a follow-up visit. The nurse notes that a prolonged increase of the serum human chorionic gonadotropin (hCG) level is a danger sign. Which condition is this client at increased risk of developing? 1 Uterine rupture 2 Choriocarcinoma 3 Hyperemesis gravidarum 4 Disseminated intravascular coagulation (DIC)

2 Choriocarcinoma

A 3-month-old infant has been hospitalized with respiratory syncytial virus (RSV). What is the priority intervention? 1 Administering an antiviral agent 2 Clustering care to conserve energy 3 Offering oral fluids to promote hydration 4 Providing an antitussive agent whenever necessary

2 Clustering care to conserve energy Often the infant will have a decreased pulmonary reserve, and the clustering of care is essential to provide for periods of rest.

A nurse determines that a newborn is suffering from respiratory distress. Which visible signs confirm this assessment? Select all that apply. 1 Crackles 2 Cyanosis 3 Wheezing 4 Tachypnea 5 Retractions

2 Cyanosis 4 Tachypnea 5 Retractions Cyanosis occurs because of inadequate oxygenation. Tachypnea is a compensatory mechanism necessary to increase oxygenation. Retractions occur in an effort to increase lung capacity. Crackles occur in the healthy newborn. Wheezing in the newborn is benign.

What clinical finding does a nurse expect when assessing a 4-month-old infant with gastroenteritis and moderate dehydration? 1 Urine output of 50 mL/hr 2 Depressed anterior fontanel 3 History of allergies to certain formulas 4 Capillary refill time of less than 2 seconds

2 Depressed anterior fontanel

A client asks the nurse what advantage breast-feeding holds over formula feeding. What major group of substances in human milk are of special importance to the newborn and cannot be reproduced in a bottle formula? 1 Amino acids 2 Gamma globulins 3 Essential electrolytes 4 Complex carbohydrates

2 Gamma globulins

The nurse is reviewing a client's history. Which two predisposing causes of puerperal (postpartum) infection should prompt the nurse to monitor this client closely? 1 Malnutrition and anemia 2 Hemorrhage and trauma during labor 3 Preeclampsia and retention of placental fragments 4 Organisms in the birth canal and trauma during labor

2 Hemorrhage and trauma during labor

How can a nurse best soothe a hospitalized infant who appears to be in pain? 1 Feeding the infant 2 Holding the infant 3 Playing soft music in the room 4 Providing a quiet environment

2 Holding the infant

While conducting prenatal teaching, the nurse should explain to clients that there is an increase in vaginal secretions during pregnancy called leukorrhea. What causes this increase? 1 Decreased metabolic rate 2 Increased production of estrogen 3 Secretion from the Bartholin glands 4 Supply of sodium chloride to the vaginal cells

2 Increased production of estrogen Increased estrogen production during pregnancy causes hyperplasia of the vaginal mucosa, which leads to increased production of mucus by the endocervical glands. The mucus contains exfoliated epithelial cells.

A nurse is discussing diet with a pregnant client who is 5 feet 4 inches tall (163 cm) and whose pre-pregnancy weight was 120 lb (54 kg). What should the nurse include about the changes in calories and nutrients, compared with the pre-pregnancy diet, during the second trimester? 1 Decreasing daily fat consumption by 220 calories 2 Increasing total daily caloric intake by 340 calories 3 Increasing total daily caloric intake by 460 calories 4 Decreasing daily carbohydrate consumption by 130 calories

2 Increasing total daily caloric intake by 340 calories A daily increase of 340 calories is recommended for adult women during the second trimester of pregnancy.

An infant is admitted to the pediatric unit with bronchiolitis caused by respiratory syncytial virus (RSV). What interventions are appropriate nursing care for the infant? Select all that apply. 1 Limiting fluid intake 2 Instilling saline nose drops 3 Maintaining contact precautions 4 Suctioning mucus with a bulb syringe 5 Administering warm humidified oxygen

2 Instilling saline nose drops 3 Maintaining contact precautions 4 Suctioning mucus with a bulb syringe Saline nose drops help clear the nasal passage, which improves breathing and aids the intake of fluids. RSV is contagious; infants with RSV should be isolated from other children, and the number of people visiting or caring for the infant should be limited. Infants with RSV produce copious amounts of mucus, which hinders breathing and feeding; suctioning before meals and at naptime and bedtime provides relief.

When teaching a client about using a diaphragm as a form of contraception, what instructions should the nurse provide about the diaphragm? 1 It may or may not be used with a spermicidal lubricant. 2 It should remain in place for at least 6 hours after intercourse. 3 It must be inserted with the dome facing down to be maximally effective. 4 It often appears puckered but this will not interfere with its effectiveness.

2 It should remain in place for at least 6 hours after intercourse.

A nurse is caring for a 6-year-old child who is admitted to the pediatric unit with recently diagnosed nephrotic syndrome. The parents ask the nurse why their child is retaining so much fluid. What should the nurse consider before telling the parents about the changes in body fluid distribution in language that they can understand? 1 Loss of sodium and water through an impaired basement membrane of the glomerulus results in hypovolemia. 2 Loss of body protein reduces oncotic pressure, and fluid moves from the intravascular to the interstitial space. 3 Hyperproteinemia results in increased oncotic pressure, and fluid moves from the intravascular to interstitial space. 4 Basement membranes of the glomeruli become selectively impermeable to water, and fluid is retained in the tissues.

2 Loss of body protein reduces oncotic pressure, and fluid moves from the intravascular to the interstitial space.

The primary healthcare provider diagnoses placenta previa in a primiparous client. What does this indicate to the nurse regarding the condition of the placenta? 1 Infarcted 2 Low-lying 3 Immaturely developed 4 Separating prematurely

2 Low-lying Implantation should occur in the upper third of the uterus; a low-lying placenta is termed placenta previa. Placenta previa indicates where the placenta is implanted and has no relationship to placental aging.

A 6-month-old infant is brought to the emergency department in severe respiratory distress. A diagnosis of respiratory syncytial virus (RSV) infection is made, and the infant is admitted to the pediatric unit. What should be included in the nursing plan of care? 1 Place in a warm, dry environment. 2 Maintain standard and contact precautions. 3 Administer prescribed antibiotic immediately. 4 Allow parents and siblings to room in with the infant.

2 Maintain standard and contact precautions.

A newborn is admitted to the nursery and classified as small for gestational age (SGA). What is the priority nursing intervention for this infant? 1 Testing the infant's stools for occult blood 2 Monitoring the infant's blood glucose level 3 Placing the infant in the Trendelenburg position 4 Comparing the infant's head circumference and chest circumference

2 Monitoring the infant's blood glucose level

A client at 6 weeks' gestation who has type 1 diabetes is attending the prenatal clinic for the first time. The nurse explains that during the first trimester insulin requirements may decrease for what reason? 1 Body metabolism is sluggish in the first trimester. 2 Morning sickness may result in decreased food intake. 3 Fetal requirements of glucose in this period are minimal. 4 Hormones of pregnancy increase the body's need for insulin.

2 Morning sickness may result in decreased food intake. Morning sickness, a common occurrence during pregnancy, contributes to decreased food intake; the insulin dosage must be reduced to prevent hypoglycemia. The body's metabolism increases during pregnancy, because the needs of the fetus, as well as those of the mother, must be met. Rapid organogenesis requires large amounts of glucose. During the first trimester the blood glucose level is reduced and glycemic control is enhanced; glycemic control is more difficult to maintain later in the pregnancy.

A 2-year-old toddler has hearing loss caused by recurrent otitis media. What treatment does the nurse anticipate that the practitioner will recommend? 1 Ear drops 2 Myringotomy 3 Mastoidectomy 4 Steroid therapy

2 Myringotomy Myringotomy is a surgical opening into the eardrum to permit drainage of accumulated fluid associated with otitis media.

Two 3-year-old clients are playing together in a hospital playroom. One is working on a puzzle, and the other is stacking blocks. Which type of play are these children participating in based on this scenario? 1 Solitary play 2 Parallel play 3 Associative play 4 Cooperative play

2 Parallel play

A nurse is discussing the care of an infant with colic. What should the nurse explain to the parents is the cause of colicky behavior? 1 Inadequate peristalsis 2 Paroxysmal abdominal pain 3 An allergic response to certain proteins in milk 4 A protective mechanism designed to eliminate foreign proteins

2 Paroxysmal abdominal pain The traditional efforts to explain and treat colic center on the paroxysmal abdominal pain; multiple factors appear to be involved, including immaturity of the intestinal nervous system and lack of normal intestinal flora.

A 15-year-old with cystic fibrosis (CF) is admitted with a respiratory infection. The nurse determines that the adolescent is cyanotic, has a barrel-shaped chest, and is in the 10th percentile for both height and weight. What is the priority nursing intervention? 1 Increasing physical activities 2 Performing postural drainage 3 Maintaining dietary restrictions 4 Administering prescribed pancreatic enzymes

2 Performing postural drainage Postural drainage, including percussion and vibration, aids removal of respiratory secretions that provide a medium for further bacterial growth.

A nurse is assessing the condition of a school-aged child with acute glomerulonephritis. What clinical finding does the nurse anticipate? 1 Ketonuria 2 Periorbital edema 3 Increased appetite 4 Decreased blood pressure

2 Periorbital edema

A nurse is teaching parents about the nutritional needs of their 15-year-old child. What information should the nurse provide? Select all that apply. 1 Increase the child's fat intake 2 Provide iron-rich foods to the child 3 Increase the child's daily protein intake 4 Curb the child's diet to help prevent obesity 5 Provide adequate vitamin supplementation to your child

2 Provide iron-rich foods to the child 3 Increase the child's daily protein intake The nurse should instruct the parents to provide an iron-rich diet to the child. This will help prevent anemia. The nurse should instruct the parents to increase the child's protein intake. Vitamin and mineral supplements are not required, so the nurse should instruct the parents to avoid them.

A 6-week-old infant has just been found to have gastroesophageal reflux. What teaching is most important to discuss with the parents at this time? 1 Feeding cereal with a spoon 2 Providing formula thickened with cereal 3 Placing the infant on its back immediately after feedings 4 Explaining changes in care after surgical repair of the esophageal defect

2 Providing formula thickened with cereal

A school-aged child with cystic fibrosis has been admitted with a respiratory infection. The child has been very disruptive and angry with staff and parents. What does the nurse suspect is the reason for the child's uncooperative behavior? 1 Spoiled and needs to be adequately disciplined 2 Resentful of the restriction of the hospitalizations 3 Having a reaction to the new respiratory medications 4 Angry about dietary restrictions related to the disease

2 Resentful of the restriction of the hospitalizations Children with cystic fibrosis often become resentful of repeated hospitalizations, the disease itself, and restrictions on their activities.

A 2-month-old infant is admitted to the pediatric unit with a diagnosis of respiratory syncytial virus infection. The nurse plans to position the infant to improve the respiratory effort. What positions are best? Select all that apply. 1 Prone 2 Semi-Fowler 3 Trendelenburg 4 Hyper-extended head 5 Head in sniffing position

2 Semi-Fowler 5 Head in sniffing position

A pregnant client has class II cardiac disease. To best plan the client's care, what does the nurse anticipate for the client? 1 May participate in as much activity as she desires 2 Should be hospitalized if there is evidence of cardiac decompensation 3 Will have to maintain bed rest for most of the day throughout her pregnancy 4 May have to consider a therapeutic abortion if there is evidence of cardiac decompensation

2 Should be hospitalized if there is evidence of cardiac decompensation Clients with cardiac disease should be taught the signs and symptoms of cardiac decompensation; if they occur, the client should stop the activity that precipitated them and notify the primary healthcare provider. Participating in as much activity as she desires is acceptable behavior for a client with class I cardiac disease. Maintaining bed rest is the treatment for a client with class III cardiac disease. Considering a therapeutic abortion is the recommendation for a client with class IV cardiac disease.

A client asks a nurse about the most common problem associated with the use of an intrauterine device (IUD). What answer should the nurse provide? 1 Perforation of the uterus 2 Spontaneous device expulsion 3 Discomfort associated with coitus 4 Development of vaginal infections

2 Spontaneous device expulsion The IUD may cause irritability of the myometrium, inducing contraction of the uterus and expulsion of the device.

The nurse observes a laboring client's amniotic fluid and decides that it is the expected color and consistency. Which finding supports this conclusion? 1 Clear, dark amber colored, and containing shreds of mucus 2 Straw-colored, clear, and containing little white specks 3 Milky, greenish yellow, and containing shreds of mucus 4 Greenish yellow, cloudy, and containing little white specks

2 Straw-colored, clear, and containing little white specks Dark amber-colored fluid suggests the presence of bilirubin, an ominous sign. Greenish-yellow fluid may indicate the presence of meconium and suggests fetal compromise. Cloudy fluid suggests the presence of purulent material.

A 9-year-old child is found to have acute glomerulonephritis after a recent infection. What microorganism should the nurse suspect as the cause of the child's current health problem? 1 Haemophilus 2 Streptococcus 3 Pseudomonas 4 Staphylococcus

2 Streptococcus

A client arrives at the clinic with swollen, tender breasts and flulike symptoms. A diagnosis of mastitis is made. What does the nurse plan to do? 1 Help her wean the infant gradually. 2 Teach her to empty her breasts frequently. 3 Review breastfeeding techniques with her. 4 Send a sample of her milk to the laboratory for testing.

2 Teach her to empty her breasts frequently. Emptying the breasts limits engorgement because engorgement causes pressure and tenderness in an already tender area. Breastfeeding should be continued; it is not only unnecessary but also unwise to remove the infant from breastfeeding. Suckling keeps the breasts empty, limits engorgement, and reduces pain. The milk culture may be negative because the infection may be limited to the connective tissue of the breast.

Which data collected during the nursing assessment for a 24-month-old client indicates the need for further evaluation for delayed language? Select all that apply. 1 The child uses two-word sentences. 2 The child cries and points at an object he wants. 3 The child states, "Me do it" when asked to stack blocks. 4 The child understands the meaning of as many as 50 words. 5 The child asks, "What's that?" when the nurse uses the stethoscope to assess lung sounds.

2 The child cries and points at an object he wants 4 The child understands the meaning of as many as 50 words. Observations made during the nursing assessment for a 24-month-old client that would require further evaluation for a language delay include: the child crying and pointing at an object that is wanted and the child who understands the meaning of only 50 words (300 is expected by this stage of development). The use of two-word sentences, "Me do it," and asking "What's that?" are all expected findings for the 24-month-old client in regards to language development.

The nurse teaches a postpartum client how to care for her episiotomy in order to prevent infection. Which behavior indicates that the teaching has been effective? 1 The perineal pad is changed twice daily. 2 The client washes her hands before and after she changes a perineal pad. 3 The client rinses her perineum with water after using an analgesic spray. 4 The client cleanses the perineum from the anus toward the symphysis pubis.

2 The client washes her hands before and after she changes a perineal pad.

The nurse is providing care to a 6-week-old infant who is hospitalized for poor growth. The infant is currently being breastfed and is diagnosed with failure to thrive (FTT). Which is the priority nursing assessment for this infant? 1 Family financial difficulties 2 Uncoordinated suck and swallow 3 Neglect and abuse by the parents 4 Knowledge deficit related to nutritional intake

2 Uncoordinated suck and swallow Most cases of poor growth and FTT in the first two months of life occur due to an uncoordinated suck and swallow during feedings (formula or breast); therefore, this is the priority nursing assessment. Assessing for financial difficulties, neglect and abuse, and a knowledge deficit are appropriate but not the priority in this situation.

At 38 weeks' gestation a client is admitted to the birthing unit in active labor, and an external fetal monitor is applied. Late fetal heart rate decelerations begin to appear when her cervix is dilated 6 cm, and her contractions are occurring every 4 minutes and lasting 45 seconds. What is the likely cause of these late decelerations? 1 Imminent vaginal birth 2 Uteroplacental insufficiency 3 Pattern of nonprogressive labor 4 Reassuring response to contractions

2 Uteroplacental insufficiency

The nurse assures a breast-feeding mother that one way she will know that her infant is getting an adequate supply of breast milk is if the infant gains weight. What behavior does the infant exhibit if an adequate amount of milk is being ingested? 1 Has several firm stools daily 2 Voids six or more times a day 3 Spits out a pacifier when offered 4 Awakens to feed about every 4 hours

2 Voids six or more times a day

What is the correct order of the steps in which the nurse should administer eardrops to a toddler? 1. Place the toddler in the side-lying position. 2. Clean the outer ear using a washcloth. 3. Keep the toddler in the side-lying position for 2 to 3 minutes. 4. Pull the auricle down and back to straighten the ear canal. 5. Using a finger, apply gentle massage or pressure to the tragus. 6. Hold the dropper 1 cm above the ear canal to instill the prescribed drops.

2, 1, 4, 6, 3, 5

According to Erikson's theory of psychosocial development, what is the correct order of a child's behavior as they age? 1. The child concentrates on work and play. 2. The child develops autonomy by making choices. 3. The child is concerned about appearance and body image. 4. The child develops feelings of superego or conscience.

2, 4, 1, 3

The nurse plans to perform an abdominal assessment of a 10-year-old child with suspected appendicitis. List in order of priority the techniques the nurse should use when assessing this child's abdomen. 1. Auscultating for bowel sounds 2. Asking where it hurts 3. Assessing the abdomen by touch 4. Visually examining the abdomen 5. Warming the stethoscope's diaphragm

2, 4, 5, 1, 3

A nurse is teaching breast-feeding to a newly delivered client. Which statement by the client indicates the need for further instruction? 1 "I'll try to empty my breasts at each feeding." 2 "I'll alternate between breasts to start feedings." 3 "I need to wash my breasts with soapy water before I breast-feed." 4 "I need to stroke my baby's cheek gently when I'm ready to breast-feed."

3 "I need to wash my breasts with soapy water before I breast-feed." Soap irritates, cracks, and dries breasts and nipples, making it painful for the mother when the baby sucks; it also increases the risk for mastitis. The client should empty the breasts at each feeding to keep milk flowing.

A nurse provides instructions to a group of adolescents about ways to prevent obesity. Which statements made by an adolescent indicates a need for further learning? Select all that apply. 1 "I should avoid trans fats." 2 "I should limit portion sizes." 3 "I should consume a high-fat diet." 4 "I should take highly refined starch food." 5 "I should watch television for four hours only."

3 "I should consume a high-fat diet." 4 "I should take highly refined starch food." 5 "I should watch television for four hours only." A high-fat diet should be avoided by adolescents. Highly refined starches and sugars should be avoided because they are rich in calories. Adolescents should be advised to watch less than two hours of television per day. Most dieticians and nutrition experts recommend a diet with no trans fats. Adolescents should limit portion sizes to improve body weight.

The nurse educates an obese adolescent about healthy dietary habits and risk associated with obesity. Which statement by the adolescent indicates the need for further counseling? 1 "I should do exercise." 2 "I should play more outdoor games." 3 "I should watch more TV to reduce the stress." 4 "I should modify my diet and have lots of vegetables and water."

3 "I should watch more TV to reduce the stress."

The nurse determines that a young female client who is being treated for a sexually transmitted infection (STI) understands instructions regarding future sexual contacts. Which client statement confirms the nurse's conclusion? 1 "If I have sex, nothing I do will really prevent me from getting another STI." 2 "If I get another STI, I can take any antibiotic, because I'm not allergic to any of them." 3 "I won't have unprotected sex again, and I'll tell my partners to be tested for STIs." 4 "I have to ask my partners if they have an STI, and if they say no I'll know that I can have sex."

3 "I won't have unprotected sex again, and I'll tell my partners to be tested for STIs."

Which statements by a client with hyperemesis gravidarum would confirm that the client requires further teaching? Select all that apply. 1 "I'll start drinking protein shakes." 2 "I'll start drinking plenty of fluids." 3 "I'll start limiting my carbohydrates." 4 "I'll lie down for at least 2 hours after I eat." 5 "I'll be sure to schedule rest periods throughout the day so I won't get tired."

3 "I'll start limiting my carbohydrates." 4 "I'll lie down for at least 2 hours after I eat." During pregnancy the cardiac sphincter may relax, which allows food to come back up into esophagus when supine. Not lying down for up to 2 hours after eating should provide time for digestion so that food is not regurgitated. The client should not decrease carbohydrate intake.

A primigravida is admitted to the birthing suite at term with contractions occurring every 5 to 8 minutes and a bloody show present. She and her partner attended childbirth preparation classes. Vaginal examination reveals that the cervix is dilated 3 cm and 75% effaced. The fetus is at +1 station in occiput anterior position, and the membranes are intact. The client is cheerful and relaxed and asks the nurse whether it is all right for her to walk around. In light of the nurse's observations regarding the contractions and the client's knowledge of the physiology and mechanism of labor, how should the nurse respond? 1 "I can't make a decision on that; I'll have to ask your primary healthcare provider." 2 "Please stay in bed; walking could interfere with effective uterine contractions." 3 "It's all right for you to walk as long as you feel comfortable and your membranes are intact." 4 "You may sit in a chair, because your contractions cannot be timed when you walk and I won't be able to listen to the baby's heart."

3 "It's all right for you to walk as long as you feel comfortable and your membranes are intact." Contractions become stronger and more regular when the woman is standing; also, as the woman walks, the diameter of the pelvic inlet increases, allowing easier entrance of the head into the pelvis.

A female client who has been sexually active for 5 years is diagnosed with gonorrhea. The client is upset and asks the nurse, "What can I do to keep from getting another infection in the future?" Which statement by the client indicates that the teaching by the nurse was effective? 1 "I'll douche after each time I have sex." 2 "Having sex is a thing of the past for me." 3 "My partner has to use a condom all the time." 4 "I'll be using a spermicidal cream from now on."

3 "My partner has to use a condom all the time."

Which teaching point regarding safety should the nurse include in the instructions for the parents of a school-age client who is a latchkey child? 1 "Consider getting a pet for your child." 2 "Plan play dates for your child to attend on afternoons you are not home." 3 "Teach your child not to display the keys used to enter the home after school." 4 "Provide structured activities for your child to complete while they are home alone."

3 "Teach your child not to display the keys used to enter the home after school."

The mother of an 18-month-old child with a cleft palate asks the nurse why the pediatrician has recommended that closure of the palate be performed before the child is 2 years old. How should the nurse respond? 1 "As the child gets older, the palate gets wider and more difficult to repair." 2 "Eruption of the 2-year molars often complicates the surgical procedure." 3 "You need to have the surgery performed before your child starts to use faulty speech patterns." 4 "After a child is 2 years old, surgery is frightening, so you need to avoid it if at all possible."

3 "You need to have the surgery performed before your child starts to use faulty speech patterns."

A client asks the nurse at the family planning clinic whether contraception is needed while she is breastfeeding. How should the nurse reply? 1 "As long as you aren't having periods, you won't need a contraceptive." 2 "It would be best to delay sexual relations until you have your first period." 3 "You should use contraceptives, because ovulation may occur at any time without a period." 4 "Breastfeeding suppresses ovulation, so you don't need to worry about pregnancy."

3 "You should use contraceptives, because ovulation may occur at any time without a period."

Based on Erikson's theory of development, what should the nurse suggest to a mother of a toddler who harshly disciplines her child? 1 "Your behavior may induce a feeling of isolation in your child." 2 "You need to establish a sense of trust or the child may lose trust in you." 3 "Your child needs support and love or may develop feelings of shame and doubt." 4 "You need to control the child's impulses or the child may suffer guilt and frustration."

3 "Your child needs support and love or may develop feelings of shame and doubt." According to Erikson, a child of 2 years old is in the autonomy versus sense of shame and doubt stage. The nurse should counsel the mother that harshly disciplining her child may lead to a feeling of shame and doubt.

A nurse is teaching a family planning class about ovulation and conception. The nurse should instruct the class that the ovum is thought to be viable for what period of time after ovulation? 1 1 to 6 hours 2 12 to 18 hours 3 24 to 36 hours 4 48 to 72 hours

3 24 to 36 hours

A prescription for an isotonic enema is written for a 2-year-old child. What is the maximal amount of fluid the nurse should administer without a specific prescription from the healthcare provider? 1 100 to 150 mL 2 155 to 250 mL 3 255 to 360 mL 4 365 to 500 mL

3 255 to 360 mL

The nurse is assigned to care for an infant in the newborn nursery who is 24 hours old. During assessment the nurse becomes concerned that the baby is jaundiced. The nurse knows that jaundice first becomes visible in a newborn when serum bilirubin reaches what level? 1 1 to 3 mg/dL (17.1 to 51.3 µmol/L) 2 2 to 4 mg/dL (34.2 to 68.4 µmol/L) 3 5 to 7 mg/dL (85.5 to 119.7 µmol/L) 4 8 to 10 mg/dL (136.8 to 171 µmol/L)

3 5 to 7 mg/dL (85.5 to 119.7 µmol/L) Jaundice in a newborn first becomes visible when the serum bilirubin level reaches 5 to 7 mg/dL (85.5 to 119.7 µmol/L). Jaundice will not be visible at a serum bilirubin level of less than 5 mg/dL (85.5 µmol/L).

The nurse instructs the unlicensed assistive personnel (UAP) to obtain vital signs from four clients. From which client can the nurse instruct the UAP to obtain a radial pulse? 1 A 1-year-old child 2 An 18-month-old child 3 A 30-month-old child 4 A 6-month-old child

3 A 30-month-old child A satisfactory pulse can be taken radially in children older than 2 years of age, hence, the nurse can instruct the UAP to obtain a radial pulse from a 30-month-old child.

What is the most common cause of death among adolescents? 1 Suicide 2 Homicide 3 Accidents 4 Substance abuse

3 Accidents

What is the optimal nursing action for a client in active labor whose cervix is dilated 4 cm and 100% effaced with the fetal head at 0 station? 1 Document the fetal heart rate every 5 minutes. 2 Call the anesthesia department to alert the staff there of an imminent birth. 3 Assist the client's coach in helping her with the use of breathing techniques. 4 Suggest that the client accept the as-needed (PRN) medication for pain that has been prescribed.

3 Assist the client's coach in helping her with the use of breathing techniques. The client is in the early part of the first stage of labor, and it is important to help the partner with the role of coach. It is not necessary to check the fetal heart rate every 5 minutes until the second stage of labor.

When does the anterior fontanel of an infant close? 1 At 4 to 10 months 2 At 8 to 12 months 3 At 12 to 18 months 4 At 18 to 26 months

3 At 12 to 18 months

Which nursing intervention is specific to clients in active labor who present with a history of cardiac disease? 1 Encouraging frequent voiding 2 Checking the blood pressure hourly 3 Auscultating the lungs for crackles every 30 minutes 4 Helping turn the client from side to side at 15-minute intervals

3 Auscultating the lungs for crackles every 30 minutes Clients with cardiac problems are prone to heart failure during active labor; crackles indicate the presence of pulmonary edema

A 25-year-old woman comes to the clinic complaining of increased vaginal discharge, milky gray in color with a "fishy" odor that both she and her husband have noticed. A wet smear is performed and the presence of "clue cells" confirmed. Which type of infection does the nurse suspect? 1 Candidiasis 2 Trichomoniasis 3 Bacterial vaginosis 4 Group B Streptococcus

3 Bacterial vaginosis

Which nursing assessment is most important for a large-for-gestational-age (LGA) infant of a diabetic mother (IDM)? 1 Temperature less than 98° F (36.6° C) 2 Heart rate of 110 beats/min 3 Blood glucose level less than 40 mg/dL (2.2 mmol/L) 4 Increasing bilirubin during the first 24 hours

3 Blood glucose level less than 40 mg/dL (2.2 mmol/L) At birth, circulating maternal glucose is removed; however, the IDM still has a high level of insulin, and rebound hypoglycemia may develop.

Which information in a postpartum client's health history should alert the nurse to monitor the client for signs of infection? 1 Three spontaneous abortions 2 B-negative maternal blood type 3 Blood loss of 850 mL after a vaginal birth 4 Temperature of 99.9° F (37.7° C) during the first postpartum day

3 Blood loss of 850 mL after a vaginal birth Excessive blood loss predisposes the client to infection because of decreased maternal resistance; the expected blood loss is 350 to 500 mL.

Which clinical finding does the nurse expect when assessing a client with abruptio placentae? 1 Flaccid uterus 2 Painless bleeding 3 Boardlike abdomen 4 Bright red bleeding

3 Boardlike abdomen Extravasation of blood at the placental separation site into the myometrium causes a tetanic boardlike uterus. The uterus is rigid because it is filled with blood and clots.

During assessment, the nurse asks a client about developmental milestones such as the age at which thelarche and menarche occurred. The nurse determines that the client experienced pubertal delay. Which finding in the client's history supports the nurse's conclusion? 1 Weight increased by 8 to 12 kg. 2 Menarche occurred 2 years after thelarche. 3 Breast development occurred by 15 years of age. 4 Growth in height stopped 2 years after menarche.

3 Breast development occurred by 15 years of age. When the development of breasts has not occurred by 13 years of age in girls, it is considered pubertal delay.

A 24-year-old client is admitted at 40 weeks' gestation. The cervix is dilated 5 cm and is 100% effaced, and the presenting part is at station 0. The nurse assesses that the fetal heart tones are just above the umbilicus. Which fetal presentation does the nurse document? 1 Face 2 Brow 3 Breech 4 Shoulder

3 Breech

How should a nurse assess a 4-year-old child with abdominal pain? 1 By asking the child to point to where it hurts 2 By auscultating the child's abdomen for bowel sounds 3 By observing position and behavior while the child is moving 4 By questioning the parents about their child's eating and bowel habits

3 By observing position and behavior while the child is moving

After several episodes of abdominal pain and vomiting, a 5-month-old infant is admitted with a tentative diagnosis of intussusception. What assessment should the nurse document that will aid confirmation of the diagnosis? 1 Frequency of crying 2 Amount of oral intake 3 Characteristics of stools 4 Absence of bowel sounds

3 Characteristics of stools Because intussusception creates intestinal obstruction in which the intestine "telescopes" and becomes trapped, passage of intestinal contents is lessened; stools are red and look like currant jelly because of the mixing of stool with blood and mucus.

A parent brings a 2-month-old infant with Down syndrome to the pediatric clinic for a physical and administration of immunizations. Which clinical finding should prompt the nurse to perform further assessment? 1 Flat occiput 2 Small, low-set ears 3 Circumoral cyanosis 4 Protruding furrowed tongue

3 Circumoral cyanosis

A nurse in the birthing unit is admitting a client whose membranes ruptured at home. How does the nurse know whether the client is in true labor? 1 Contractions occur every 10 minutes with no change in frequency over 2 hours, and the cervix is closed. 2 Contractions are not evident; the cervix is dilated 3 cm and 50% effaced, and there is no change after 4 hours of staying out of bed. 3 Contractions occur every 5 to 10 minutes, the cervix is dilated 2 cm and 75% effaced, and dilation has increased to 3 cm in 2 hours. 4 Contractions are irregular, occurring every 10 to 15 minutes, the cervix is dilated one fingertip and is 50% effaced, and there is no change with 4 hours of bed rest.

3 Contractions occur every 5 to 10 minutes, the cervix is dilated 2 cm and 75% effaced, and dilation has increased to 3 cm in 2 hours.

Which should the nurse anticipate for a 5-year-old client related to play? Select all that apply. 1 Solitary play 2 Parallel play 3 Dress-up play 4 Team sport play 5 Cooperative play

3 Dress-up play 5 Cooperative play The 5-year-old preschool-age client would be expected to participate in dress-up play due to the preoccupation with magical thinking and the beginnings of cooperative play. Solitary play is expected for the infant and toddler. Parallel play is expected for the toddler and young preschool-age client. Team sport play is expected for the school-age client.

A client had a rubella infection (German measles) during the fourth month of pregnancy. At the time of the infant's birth, the nurse places the newborn in the isolation nursery. Which type of infection control precautions should the nurse institute? 1 Enteric 2 Contact 3 Droplet 4 Standard

3 Droplet

A 24-year-old woman wants to use her basal body temperature (BBT) in natural family planning but is unsure when to take her temperature. When should the nurse explain is the best time for accurate BBT assessment? 1 Each night right before bed 2 On the first day of her next menstrual cycle 3 Each morning before getting out of bed or increasing her activity 4 At bedtime beginning on day 14 of her menstrual cycle and continuing until her next period

3 Each morning before getting out of bed or increasing her activity The most accurate BBT is taken before a woman gets out of bed and begins any type of activity that could increase the body's temperature even slightly.

After abdominal surgery, a 5-year-old child is experiencing pain, and an opioid analgesic is prescribed. What should the nurse consider about children in pain and their response to opioid analgesics when an opioid analgesic is prescribed? 1 Addiction to opioids is more of a risk for children than adults. 2 Analgesics are not needed as frequently because pain is not as strongly felt by children as it is by adults. 3 Even though children do not like taking medicines, analgesics will make them more comfortable. 4 Children do not need analgesics because they are easily distracted and will quickly return to play or sleep.

3 Even though children do not like taking medicines, analgesics will make them more comfortable.

A nurse is planning to teach the parents of a preschool child with recently diagnosed cystic fibrosis why the child has respiratory problems. What should the nurse remember about the underlying pathophysiology? 1 Airway irritability causes spasms. 2 Lung parenchyma becomes inflamed. 3 Excessively thick mucus obstructs airways. 4 Endocrine glands secrete surplus hormones.

3 Excessively thick mucus obstructs airways.

A nurse who is observing a sleeping newborn at 2 hours of age identifies periods of irregular breathing and occasional twitching movements of the arms and legs. The neonate's heart rate is 150 beats/min; the respiratory rate is 50 breaths/min; and the glucose strip reading is 60 mg/dL (3.3 mmol/L). What does the nurse conclude that these findings indicate? 1 Hypoglycemia 2 Seizure activity 3 Expected adaptations 4 Respiratory distress syndrome

3 Expected adaptations During periods of active or irregular sleep, healthy newborns have some twitching movements and irregular respirations; the heart rate, respirations, and blood glucose level are within expected limits. Hypoglycemia in newborns is characterized by a blood glucose level below 30 mg/dL The newborn respiratory rate ranges between 30 and 60 breaths/min; irregular breathing is expected.

The mother of an infant with Down syndrome asks the nurse what causes the disorder. Before responding, the nurse recalls that the genetic factor of Down syndrome results from what? 1 An intrauterine infection 2 An X-linked genetic disorder 3 Extra chromosomal material 4 An autosomal recessive gene

3 Extra chromosomal material

A nurse is caring for a school-aged child with nephrotic syndrome who has massive edema. The nurse teaches the parents about the low-sodium diet that has been ordered. Which food group has the lowest level of sodium compared with the other food groups? 1 Meat 2 Dairy 3 Fresh fruit 4 Fresh vegetables

3 Fresh fruit

A pregnant client is making her first antepartum visit. She has a 2-year-old son born at 40 weeks, a 5-year-old daughter born at 38 weeks, and 7-year-old twin daughters born at 35 weeks. She had a spontaneous abortion 3 years ago at 10 weeks. How does the nurse, using the GTPAL format, document the client's obstetric history? 1 G4 T3 P2 A1 L4 2 G5 T2 P2 A1 L4 3 G5 T2 P1 A1 L4 4 G4 T3 P1 A1 L4

3 G5 T2 P1 A1 L4 The acronym GTPAL represents gravidity, term births, preterm births, abortions, and living children; G5 T2 P1 A1 L4 indicates that the client has had five pregnancies (twins count as one pregnancy and the current pregnancy counts as one); two term births; one preterm birth (the twins); one abortion; and four living children.

A 13-month-old toddler has a respiratory tract infection with a low-grade fever. When teaching the parents, which intervention should the nurse emphasize? 1 Encouraging high-calorie snacks to prevent weight loss 2 Keeping the toddler wrapped in blankets to prevent shivering 3 Giving small amounts of clear liquids frequently to prevent dehydration 4 Using cool-water baths to prevent the toddler's fever from increasing further

3 Giving small amounts of clear liquids frequently to prevent dehydration

The four essential components of labor are passenger, powers, passageway, and position. Passageway refers to the bony pelvis. Which type of pelvis is considered the most favorable for a vaginal delivery? 1 Android 2 Anthropoid 3 Gynecoid 4 Platypelloid

3 Gynecoid

A nurse is assessing a client at 16 weeks' gestation. Where does the nurse expect the fundal height to be located? 1 Above the umbilicus 2 At the level of the umbilicus 3 Half the distance to the umbilicus 4 Slightly above the symphysis pubis

3 Half the distance to the umbilicus

The nurse is caring for the newborn of a drug-addicted mother with suspected cytomegalovirus disease. What does the nurse suspect was the cause of the disease? 1 Handling a cat litter box 2 Drinking contaminated water 3 Having sex with many partners 4 Eating inadequately cooked meat

3 Having sex with many partners Cytomegalovirus has been recovered from semen, vaginal secretions, urine, feces, and blood; it is commonly found in clients who have human immunodeficiency virus

A 9-year-old child is admitted to the pediatric unit with a diagnosis of acute glomerulonephritis. What does the nurse expect the admission urinalysis to reveal? 1 Polyuria 2 Ketonuria 3 Hematuria 4 Bacteriuria

3 Hematuria The urine is cloudy, smoky, or the color of tea because of the presence of erythrocytes and casts from the affected kidney tissue.

A nurse anticipates that dialysis will be necessary for a 12-year-old child with chronic kidney disease when the child begins to exhibit which symptom? 1 Hypotension 2 Hypokalemia 3 Hypervolemia 4 Hypercalcemia

3 Hypervolemia Hypervolemia results when the kidneys have failed and are no longer able to maintain homeostasis, the blood pressure is high, and cardiac overload is imminent.

The nurse is caring for a preterm neonate who is receiving gastric feedings. Which neonatal clinical finding unique to necrotizing enterocolitis (NEC) leads the nurse to suspect that the neonate is experiencing this complication? 1 Persistent diarrhea 2 Decreased abdominal circumference 3 Increased amount of residual gastric aspirates 4 Small amount of vomitus after each gastric feeding

3 Increased amount of residual gastric aspirates An increasing residual volume from earlier feedings without increasing intake indicates that absorption is decreasing, a sign of NEC

A 16-year-old client has a steady boyfriend with whom she is having sexual relations. She asks the nurse how she can protect herself from contracting human immunodeficiency virus (HIV). Which guidance is most appropriate for the nurse to provide? 1 Ask her partner to withdraw before ejaculating. 2 Make certain their relationship is monogamous. 3 Insist that her partner use a condom when having sex. 4 Seek counseling about various contraceptive methods.

3 Insist that her partner use a condom when having sex.

What is an important nursing intervention when a client is receiving intravenous (IV) magnesium sulfate for preeclampsia? 1 Limiting IV fluid intake 2 Preparing for a possible precipitous birth 3 Maintaining a quiet, darkened environment 4 Obtaining magnesium gluconate as an antagonist

3 Maintaining a quiet, darkened environment Calcium gluconate is the antagonist for mag sulfate toxicity not mag gluconate. Infusions are not limited. Reducing environmental stimuli is essential for limiting or preventing seizures.

A nurse in the newborn nursery is monitoring an infant for jaundice related to ABO incompatibility. What blood type does the mother usually have to cause this incompatibility? 1 A 2 B 3 O 4 AB

3 O Mothers with type O blood have anti-A and anti-B antibodies that are transferred across the placenta. This is the most common incompatibility, because the mother is type O in 20% of all pregnancies.

The mother of a 2-year-old child tells the nurse that she is concerned about her child's vision. What behavior when the child is tired leads the nurse to suspect strabismus? 1 One eyelid droops. 2 Both eyes look cloudy. 3 One eye moves inward. 4 Both eyes blink excessively.

3 One eye moves inward.

A nurse is assessing a client for the potential for osteoporosis. Which factor in the client's history increases the risk for this disorder? 1 Estrogen therapy 2 Hypoparathyroidism 3 Prolonged immobility 4 Excessive calcium intake

3 Prolonged immobility

The nurse is assessing a client with worsening preeclampsia. What is the most significant clinical manifestation of severe preeclampsia? 1 Polyuria 2 Vaginal spotting 3 Proteinuria of 3+ 4 Blood pressure of 130/80 mm Hg

3 Proteinuria of 3+ As preeclampsia worsens, blood pressure and edema increase and degenerative changes of the kidney cause increasing proteinuria (3+). With worsening preeclampsia, oliguria, not polyuria, is expected.

A primigravida who is at 40 weeks' gestation arrives at the birthing center with abdominal cramping and a bloody show. Her membranes ruptured 30 minutes before arrival. A vaginal examination reveals 1 cm of dilation and the presenting part at -1 station. After obtaining the fetal heart rate and maternal vital signs, what should the nurse's priority intervention be? 1 Teach the client how to push with each contraction. 2 Encourage the client to perform patterned, paced breathing. 3 Provide the client with comfort measures used for women in labor. 4 Prepare to have the client's blood typed and crossmatched in the event of the need for a transfusion.

3 Provide the client with comfort measures used for women in labor. The client is experiencing the expected discomforts of labor; the nurse should initiate measures that will promote relaxation. The client is in early first-stage labor; pushing commences during the second stage. Patterned, paced breathing should be used in the transition phase, not the early phase of the first stage of labor

Elbow restraints are prescribed for an 18-month-old toddler who just had surgery for a cleft palate. The nurse explains to the parents that the restraints are used to keep the child from doing what? 1 Playing with unsterile toys 2 Rolling to a supine position 3 Putting fingers into the mouth 4 Removing the nasogastric tube

3 Putting fingers into the mouth

A client who is at risk for seizures as a result of severe preeclampsia is receiving an intravenous infusion of magnesium sulfate. What findings cause the nurse to determine that the client is showing signs of magnesium sulfate toxicity? Select all that apply. 1 Proteinuria 2 Epigastric pain 3 Respirations of 10 breaths/min 4 Loss of patellar reflexes 5 Urine output of 40 mL/hr

3 Respirations of 10 breaths/min 4 Loss of patellar reflexes

An emergency tracheotomy is performed on a toddler in acute respiratory distress from laryngotracheobronchitis (viral croup). What early signs of respiratory distress indicate that it is necessary for the nurse to suction the tracheotomy? Select all that apply. 1 Stridor 2 Cyanosis 3 Restlessness 4 Increased pulse rate 5 Substernal retractions

3 Restlessness 4 Increased pulse rate Restlessness and increased pulse rate are early signs of hypoxia; suctioning is required to keep the airway patent. Stridor, cyanosis, and substernal retractions are late signs of hypoxia; suctioning should be performed before substernal retractions occur.

A newborn has just been admitted to the pediatric surgical unit from the birth hospital with a diagnosis of tracheoesophageal fistula. In what position should this child be maintained? 1 Prone, to reduce risk of aspiration 2 Trendelenburg, to drain stomach contents 3 Semi-Fowler, to reduce the risk of chemical pneumonia 4 Supine, to reduce the risk of sudden infant death syndrome

3 Semi-Fowler, to reduce the risk of chemical pneumonia

The most appropriate method for a nurse to evaluate the effects of the maternal blood glucose level in the infant of a diabetic mother is by performing a heel stick blood test on the newborn. What specifically does this test determine? 1 Blood acidity 2 Glucose tolerance 3 Serum glucose level 4 Glycosylated hemoglobin level

3 Serum glucose level

When obtaining informed consent for sterilization from a developmentally challenged adult client, what must the nurse ensure? 1 That a parent or guardian signs the consent 2 That the client is able to explain what the procedure entails 3 That the client is able to comprehend the outcome of the procedure 4 That a parent or guardian has encouraged the client to make the decision

3 That the client is able to comprehend the outcome of the procedure

The nurse is performing a gestational age assessment using the New Ballard Scale. The infant's total neuromuscular score is 16, and total physical maturity score is 20. According to the graph, at how many weeks' gestation is the newborn?

38 weeks The fetus is at 38 weeks' gestation. Add the total neuromuscular score of 16 to the total physical maturity score of 20 for a total of 36. Look under the score total score column and you will find that the closest number to the total score of 36 is 35. Staying on the same row, move right to the Gestational Age column, where you will find the gestational age of 38 weeks. highest score of 35 = 38 weeks

A nursing student is learning about expected postpartum anatomic and physiologic changes. Which statement made by the nursing student indicates a need for further learning? 1 "The capacity of the bladder increases postpartum." 2 "The uterus involutes to approximately 350 g by two weeks after birth." 3 "The cervical dilation decreases to 2 to 3 cm by the second or third postpartum day." 4 "After birth, the vagina gradually decreases in size and returns to its pre-pregnancy state."

4 "After birth, the vagina gradually decreases in size and returns to its pre-pregnancy state." After a birth, the vagina gradually decreases in size; however, does not return to its pre-pregnancy state.

A mother asks the neonatal nurse why her infant must be monitored so closely for hypoglycemia when her type 1 diabetes was in excellent control during the entire pregnancy. How should the nurse best respond? 1 "A newborn's glucose level drops after birth, so we're being especially cautious with your baby because of your diabetes." 2 "A newborn's pancreas produces an increased amount of insulin during the first day of birth, so we're checking to see whether hypoglycemia has occurred." 3 "Babies of mothers with diabetes do not have large stores of glucose at birth, so it's difficult for them to maintain the blood glucose level within an acceptable range." 4 "Babies of mothers with diabetes have a higher-than-average insulin level because of the excess glucose received from the mothers during pregnancy, so the glucose level may drop."

4 "Babies of mothers with diabetes have a higher-than-average insulin level because of the excess glucose received from the mothers during pregnancy, so the glucose level may drop."

A nurse educates a mother about the proper administration of oral medication to her 4-year-old child. What statement made by the mother indicates effective learning? 1 "I should administer the medication with a cup or spoon." 2 "I should mix the medicine in a large amount of food." 3 "I should avoid giving a straw to my child to take pills." 4 "I should use a disposable oral syringe to prepare liquid doses."

4 "I should use a disposable oral syringe to prepare liquid doses."

What statement by a breast-feeding mother indicates that the nurse's teaching regarding stimulating the let-down reflex has been successful? 1 "I will take a cool shower before each feeding." 2 "I will drink a couple of quarts of fat-free milk a day." 3 "I will wear a snug-fitting breast binder day and night." 4 "I will apply warm packs and massage my breasts before each feeding."

4 "I will apply warm packs and massage my breasts before each feeding."

A 24-year-old client who has had type 1 diabetes for 6 years is concerned about how her pregnancy will affect both diet and insulin needs. How should the nurse respond? 1 "Insulin needs will decrease; the excess glucose will be used for fetal growth." 2 "Diet and insulin needs won't change, and maternal and fetal needs will be met." 3 "Protein needs will increase, and adjustments to insulin dosage will be necessary." 4 "Insulin dosage and dietary needs will be adjusted in accordance with the results of blood glucose monitoring."

4 "Insulin dosage and dietary needs will be adjusted in accordance with the results of blood glucose monitoring."

At a client's first prenatal visit, the nurse-midwife performs a pelvic examination. The nurse states that the client's cervix is bluish purple, which is known as the Chadwick sign. The client becomes concerned and asks whether something is wrong. What does the nurse respond with about this expected finding? 1 "It helps confirm your pregnancy." 2 "It is not unusual, even in women who are not pregnant." 3 "It occurs because the blood is trapped by the pregnant uterus." 4 "It is caused by increased blood flow to the uterus during pregnancy."

4 "It is caused by increased blood flow to the uterus during pregnancy." Stating that the Chadwick sign is caused by increased blood flow to the uterus during pregnancy underscores the normalcy of Chadwick sign and provides a simple explanation of the cause; women often need reassurance that the physical changes associated with pregnancy are expected. Stating that the Chadwick sign helps confirm pregnancy answers part of the question, but fails to explain why it occurs.

A postpartum client who was receiving an intravenous infusion of oxytocin to stimulate labor asks the nurse why it is not being discontinued now that the baby is born. How should the nurse respond? 1 "The oxytocin promotes the flow of lochia." 2 "The oxytocin eases the discomfort of involution." 3 "The oxytocin enhances the healing of tissue in the uterus." 4 "The contractions prevent excessive bleeding."

4 "The contractions prevent excessive bleeding." Oxytocin intensifies contractions of the uterus and promotes return of the uterus to its prepregnant state. It is sometimes used after delivery to prevent hemorrhage caused by uterine atony. As the uterus contracts, the flow of lochia decreases. Oxytocin tends to worsen, not ease, discomfort; the client may experience pain as the uterus contracts.

A mother is inspecting her newborn girl for the first time. The infant's breasts are edematous, and she has a pink vaginal discharge. How should the nurse respond when the mother asks what is wrong? 1 "You seem very concerned. I don't see anything unusual." 2 "Your baby appears to have a problem. I'll notify the pediatrician." 3 "The swelling and discharge will go away. It's nothing to worry about." 4 "The swelling and discharge are expected. They're a response to your hormones."

4 "The swelling and discharge are expected. They're a response to your hormones."

A nurse is making room assignments on the pediatric unit. Who is the best choice of roommate for a 10-year-old boy with juvenile idiopathic arthritis? 1 An 11-year-old girl with colitis 2 A 10-year-old boy with asthma 3 A 10-year-old girl with a fractured femur 4 An 11-year-old boy who has undergone splenectomy

4 An 11-year-old boy who has undergone splenectomy

A 30-month-old boy with cystic fibrosis is admitted to the pediatric unit with a severe upper respiratory infection. The toddler is small for his age. What pathologic process does the nurse know is the cause of his small stature? 1 Increased salt retention 2 An atrioventricular defect 3 Retention of carbon dioxide 4 An absence of pancreatic enzymes

4 An absence of pancreatic enzymes Fats, proteins, and carbohydrates are not digested because of a deficiency of pancreatic enzymes and therefore physical growth is hampered.

A teacher's aide in a kindergarten class informs the school nurse that a male student said that his mother beat him and that he has bruises on the back and shoulders. What is the priority nursing action? 1 Notifying Child Protective Services 2 Reporting this information to the principal 3 Calling the parents to arrange a conference 4 Assessing the child for the presence of bruises

4 Assessing the child for the presence of bruises

A toddler who lacks toilet training is admitted to a hospital. What does the nurse need to do when collecting urine samples from the toddler? Select all that apply. 1 Squeeze urine from the diaper. 2 Place a hat under the toilet seat. 3 Convince the child to void in the unfamiliar receptacle. 4 Attach single-use bags over the child's urethral meatus. 5 Use the terms for urination that the child can understand.

4 Attach single-use bags over the child's urethral meatus. 5 Use the terms for urination that the child can understand.

An infant with a diaphragmatic hernia undergoes corrective surgery. What nursing assessment indicates that the infant's respiratory condition has improved? 1 Cessation of crying 2 Retention of 1 oz (30 mL) of formula 3 Reduction of arterial blood pH to 7.31 4 Auscultation of breath sounds bilaterally

4 Auscultation of breath sounds bilaterally

Which finding in a newborn whose temperature over the last 4 hours has fluctuated between 98.0 °F (36.7 °C) and 97.4 °F (36.3 °C) would be considered critical? 1 Respiratory rate of 60 breaths/min 2 White blood count greater than 15,000 mm3 3 Serum calcium level of 8 mg/dL (2 mmol/L) 4 Blood glucose level of 26 mg/dL (1.4 mmol/L)

4 Blood glucose level of 26 mg/dL (1.4 mmol/L) Instability of the newborn's temperature is an indication of hypoglycemia

A few hours after being admitted to the hospital with a diagnosis of inevitable abortion, a client at 16 weeks' gestation begins to experience a bearing-down sensation and suddenly expels the products of conception in the bed. What should the nurse do first? 1 Notify the primary healthcare provider 2 Administer the prescribed sedative 3 Take the client to the operating room 4 Check the client's fundus for firmness

4 Check the client's fundus for firmness After a spontaneous abortion the uterine fundus should be palpated for firmness, which indicates effective uterine tone. If the uterus is not firm or appears to be hypotonic, hemorrhage may occur; a soft or boggy uterus also may indicate retained placental tissue. The nurse would notify the primary healthcare provider if necessary after checking for fundal firmness.

A couple interested in family planning asks the nurse about the cervical mucus method of preventing pregnancy. The nurse explains that with this method the couple must avoid intercourse when and a few days after the cervical mucus is what? 1 White and thick 2 Yellow and thin 3 Cloudy and viscid 4 Clear and stretchable

4 Clear and stretchable The cervical mucus is clear and stretchable (spinnbarkeit) at ovulation because of maximal estrogen stimulation.

A 26-year-old woman whose sister recently had a lumpectomy for breast cancer calls the local women's health center for an appointment for a mammogram. What guidance should the nurse provide the client in preparation for the test? 1 Do not eat for 6 hours before the test. 2 The room will be darkened throughout the procedure. 3 The first mammogram is usually performed at 50 years of age. 4 During the procedure, each breast will be compressed firmly between two plates.

4 During the procedure, each breast will be compressed firmly between two plates.

When changing her newborn's diaper a new mother notes a reddened area on the infant's buttock and reports it to the nurse. How should the nurse best address this mother's concern? 1 Have nursery staff members change the infant's diaper. 2 Use both lotion and powder to protect the involved area. 3 Request that the health care provider prescribe a topical ointment. 4 Encourage the mother to cleanse the area and change the diaper more often.

4 Encourage the mother to cleanse the area and change the diaper more often. Frequent cleansing and diaper changes will limit the presence of irritating substances.

After a spontaneous vaginal delivery, the client expresses concern when the newborn is brought to her. Although the newborn was just cleaned and examined, the mother notes a red rash consisting of small papules on the face, chest, and back of the newborn. What condition does the nurse recognize? 1 Harlequin sign 2 Vernix caseosa 3 Nevus flammeus 4 Erythema toxicum

4 Erythema toxicum Erythema toxicum is a benign, generalized, transient rash that is a reaction to the new environment in which a neonate finds itself. It disappears a short time after birth.

What should the plan of care for a client with a tentative diagnosis of partial abruptio placentae include? 1 Bed rest with sedation 2 Trendelenburg position and hydration 3 Preparation for emergency cesarean birth 4 External fetal monitoring and oxygenation

4 External fetal monitoring and oxygenation Fetal monitoring and oxygen administration should be instituted to protect the fetus. Some placental separation has occurred, and it may progress further. Sedation is contraindicated; it may further stress an already compromised fetus. The Trendelenburg position may shift the heavy uterus against the diaphragm and lead to compromised maternal respiratory function, further depriving the fetus of oxygen. Hydration is not a priority at this time. Further assessment of fetal status and progression of abruption placentae is needed before a cesarean birth is considered.

A newborn has just begun to breast-feed for the first time. Although the neonate has latched on to the mother's nipple, soon after beginning to suck the infant begins to choke, has an excessive quantity of frothy secretions, and exhibits unexplained episodes of cyanosis. How should the nurse best intervene at this time? 1 Tell the client to use the other breast and continue breast-feeding 2 Delay the feeding to allow more time for the infant to recover from the birthing process 3 Contact the lactation consultant to help the client learn a more successful breast-feeding technique 4 Halt the feeding and notify the healthcare provider to evaluate the infant for a tracheoesophageal fistula

4 Halt the feeding and notify the healthcare provider to evaluate the infant for a tracheoesophageal fistula

A client at term is admitted in active labor. She has tested positive for human immunodeficiency virus (HIV). Which intervention in the standard prescriptions should the nurse question? 1 Sonogram 2 Nonstress test 3 Sterile vaginal examination 4 Internal fetal scalp electrode

4 Internal fetal scalp electrode The electrode used for internal fetal monitoring pierces the fetal scalp; fetal exposure to maternal blood increases the risk of the fetus contracting HIV.

A client exhibits oligohydramnios at 36 weeks' gestation. What newborn complication should the nurse anticipate? 1 Spina bifida 2 Imperforate anus 3 Tracheoesophageal fistula 4 Intrauterine growth restriction (IUGR)

4 Intrauterine growth restriction (IUGR) Oligohydramnios is associated with IUGR; risk factors for IUGR include inadequate maternal nutrition and other high-risk conditions such as diabetes and preeclampsia.

A 23-year-old woman comes to the clinic for a Pap smear. After the examination, the client confides that her mother died of endometrial cancer 1 year ago and says that she is afraid that she will die of the same cancer. Which risk factor stated by the client after an education session on risk factors indicates that further teaching is needed? 1 Obesity 2 High-fat diet 3 Hypertension 4 Late-onset menarche

4 Late-onset menarche

A 1-year-old infant with a distended abdomen is admitted to the pediatric unit with the diagnosis of Hirschsprung disease. In which position should the nurse place the infant? 1 Prone 2 Sitting 3 Supine 4 Lateral

4 Lateral In the lateral position the distended abdomen does not press against the diaphragm, facilitating lung expansion.

The nurse is caring for preterm infants with respiratory distress in the neonatal intensive care unit. What is the priority nursing action? 1 Limiting caloric intake to decrease metabolic rate 2 Maintaining the prone position to prevent aspiration 3 Limiting oxygen concentration to prevent eye damage 4 Maintaining a high-humidity environment to promote gas exchange

4 Maintaining a high-humidity environment to promote gas exchange

What type of lochia should the visiting nurse expect to observe on a client's pad on the fourth day after a vaginal delivery? 1 Scant alba 2 Scant rubra 3 Moderate rubra 4 Moderate serosa

4 Moderate serosa The uterus sloughs off the blood, tissue, and mucus of the endometrium post-delivery. This happens in three stages that will vary in length and represent the normal healing of the endometrium. Lochia rubra is the first and heaviest stage of lochia. The blood that's expelled during lochia rubra will be bright red and may contain blood clots. The lochia rubra phase typically lasts for the first three days following delivery. Lochia serosa is the second stage of postpartum bleeding and is thinner in consistency and brownish or pink in color. Lochia serosa typically lasts from day four through day 10, following delivery. Lochia alba is the final stage of lochia. Rather than blood, there will be a white or yellowish discharge that's generated during the healing process and the initial reconstruction of the endometrium. Expect this discharge to continue for around six weeks after birth, but keep in mind that it may extend beyond that if the second phase of lochia lasted longer than ten days.

A client asks about the difference between cow's milk and breast milk. The nurse should respond that cow's milk differs from human milk in that it contains what? 1 Less protein, less calcium, and more carbohydrates 2 More protein, less calcium, and fewer carbohydrates 3 Less protein, more calcium, and more carbohydrates 4 More protein, more calcium, and fewer carbohydrates

4 More protein, more calcium, and fewer carbohydrates

The nurse is assessing the rate of involution of a client's uterus on the second postpartum day. Where does the nurse expect the fundus to be located? 1 At the level of the umbilicus 2 One fingerbreadth above the umbilicus 3 Above and to the right of the umbilicus 4 One or two fingerbreadths below the umbilicus

4 One or two fingerbreadths below the umbilicus

A postpartum client is being prepared for discharge. The laboratory report indicates that she has a white blood cell (WBC) count of 16,000/mm3. (16 X 109/L) What is the next nursing action? 1 Checking with the nurse manager to see whether the client may go home 2 Reassessing the client for signs of infection by taking her vital signs 3 Delaying the client's discharge until the practitioner has conducted a complete examination 4 Placing the report in the client's record because this is an expected postpartum finding

4 Placing the report in the client's record because this is an expected postpartum finding

A nurse is caring for a 6-year-old child with a diagnosis of glomerulonephritis. The child's urine output decreases to less than 100 mL/24 hr, the creatinine clearance is 60 mL/min, and there is an irregular apical pulse. A diagnosis of acute renal failure is made. Blood is drawn for testing. Which serum level requires immediate intervention? 1 Sodium 126 mEq/L (126 mmol/L) 2 Bilirubin 0.3 mg/dL (5.1 mcmol/L) 3 Creatinine 1.3 mg/dL (114.4 mcmol/L) 4 Potassium 6.1 mEq/L (6.1 mmol/L)

4 Potassium 6.1 mEq/L (6.1 mmol/L)

A newborn is experiencing cold stress while being admitted to the nursery. Which nursing goal has the highest immediate priority? 1 Minimize shivering 2 Prevent hyperglycemia 3 Limit oxygen consumption 4 Prevent metabolism of fat stores

4 Prevent metabolism of fat stores Newborns do not shiver. If the newborn is cold, there is increased brown fat metabolism (nonshivering thermogenesis), which increases fatty acid blood levels and predisposes the infant to acidosis. Hypoglycemia and not hyperglycemia will occur because the newborn's glycogen reserves deplete rapidly while under cold stress.

The nurse is providing education to the parents of a preschool-age client who is experiencing a severe fear of the dark. Which treatment option should the nurse share with the parents during the teaching session? 1 Prescription medication 2 Electroconvulsive therapy 3 Intensive therapy sessions 4 Repetition of brave statements

4 Repetition of brave statements

While assessing a newborn the nurse notes the following findings: arms and legs slightly flexed; smooth, transparent skin; abundant lanugo on the back; slow recoil of the pinnae; and few sole creases. What complication does the nurse anticipate in light of these findings? 1 Polycythemia 2 Hyperglycemia 3 Postmaturity syndrome 4 Respiratory distress syndrome

4 Respiratory distress syndrome The assessment findings are indicative of a preterm infant; therefore the nurse should monitor the infant for signs of respiratory distress syndrome.

The nurse should assess an infant with gastroesophageal reflux for what complication? 1 Bowel obstruction 2 Abdominal distention 3 Increased hematocrit 4 Respiratory problems

4 Respiratory problems

A 15-year-old adolescent with Down syndrome is scheduled for surgery. The parents inform the nurse that their child has a mental age of 8 years. At what age level should the nurse prepare the child's preoperative teaching plan? 1 Adult, for the parents to understand 2 Specific age, as ordered by the healthcare provider 3 Adolescent, because this is the child's chronologic age 4 School-age, because this is the child's developmental age

4 School-age, because this is the child's developmental age

A client at 42 weeks' gestation is scheduled for induction of labor. The nurse begins the induction with a piggyback infusion of 15 units of oxytocin. Which clinical finding requires the nurse to discontinue the oxytocin infusion? 1 Contractions that occur every 3 minutes and lasting 60 seconds 2 Elevation of blood pressure from 110/70 to 135/85 mm Hg during the last 30 minutes 3 Rupture of membranes with amniotic fluid that contains threads of blood and mucus 4 Several late fetal heart rate decelerations that return to baseline after the contraction is over

4 Several late fetal heart rate decelerations that return to baseline after the contraction is over Late decelerations suggest uteroplacental insufficiency, which is an indication that the oxytocin infusion should be stopped.

The parents of a boy with hypospadias with chordee ask a nurse why their child should undergo corrective surgery. What problem that may develop eventually should the nurse discuss with the parents? 1 Renal failure 2 Testicular cancer 3 Testicular torsion 4 Sexual difficulties

4 Sexual difficulties

A client asks for and receives instruction regarding birth control methods. She elects to use a diaphragm with a spermicide. Which disadvantage of using a diaphragm should be emphasized to the client? 1 Its failure rate is 50% when it is used alone. 2 It is physically uncomfortable when in place. 3 Thrombus formation and pulmonary emboli may occur. 4 Some women find its insertion and removal inconvenient.

4 Some women find its insertion and removal inconvenient.

A mother brings her 6-year-old child to the pediatric clinic, stating that the child has not been feeling well, is weak and lethargic, and has a poor appetite, headaches, and smoky-colored urine. What additional information should the nurse obtain that will aid diagnosis? 1 Rash on palms and feet 2 Shoulder and knee pain 3 Recent weight loss of 2 lb (0.9 kg) 4 Strep throat in the past two weeks

4 Strep throat in the past two weeks The smoky urine and the stated symptoms should lead the nurse to suspect glomerulonephritis, which usually occurs after a recent streptococcal infection.

The nurse is providing discharge instructions to the parents of a child who has undergone surgical correction of hypospadias. What is the priority information for the nurse to include? 1 Ensuring that the child's privacy is maintained 2 Increasing the time that the catheter is clamped 3 Maintaining the surgically implanted tension device 4 Teaching parents how to care for the catheterization system

4 Teaching parents how to care for the catheterization system Parents should know how to empty the urine bag and how to prevent kinking of the tubing.

How should the nurse screen the newborn of a diabetic mother for hypoglycemia? 1 Testing for glucose tolerance 2 Drawing blood for a serum glucose determination 3 Arranging for a fasting blood glucose determination 4 Testing heel blood with the use of a glucose-oxidase strip

4 Testing heel blood with the use of a glucose-oxidase strip Glucose-oxidase strips are used by nurses to screen infants for hypoglycemia.

A nurse assesses the frequency of a client's contractions by timing them from the beginning of a contraction until when? 1 The uterus starts to relax 2 The end of a second contraction 3 The uterus has relaxed completely 4 The beginning of the next contraction

4 The beginning of the next contraction

A toddler undergoes the implantation of a low-profile (skin-level) device (button) for a gastrostomy. The gastrostomy is now healed, and the parents are being taught to care for the stoma. What parental behavior indicates to the nurse that additional teaching is needed? 1 A parent is cleaning the stoma with soapy water. 2 Gastric contents are aspirated before the start of a feeding. 3 A parent inserts an adapter into the button to initiate a feeding. 4 The button is being maintained in the same position within the stoma.

4 The button is being maintained in the same position within the stoma.

On the third postpartum day a woman who is breastfeeding calls the nurse at the clinic and asks why her breasts are tight and swollen. What should the nurse consider before explaining engorgement to the client? 1 There is an overabundance of milk. 2 Breastfeeding is probably ineffective. 3 The breasts have been inadequately supported. 4 The lymphatic system in the breasts is congested.

4 The lymphatic system in the breasts is congested. An exaggeration of venous and lymphatic circulation caused by prolactin occurs before lactation. Engorgement occurs before lactation or milk production.

An adolescent at 10 weeks' gestation visits the prenatal clinic for the first time. The nutrition interview indicates that her dietary intake consists mainly of soft drinks, candy, French fries, and potato chips. Why does the nurse consider this diet inadequate? 1 The caloric content will result in too great a weight gain. 2 The ingredients in soft drinks and candy can be teratogenic in early pregnancy. 3 The salt in this diet will contribute to the development of gestational hypertension. 4 The nutritional composition of the diet places her at risk for a low-birth-weight infant.

4 The nutritional composition of the diet places her at risk for a low-birth-weight infant.

A nurse is evaluating a 3-year-old child's developmental progress. The inability to perform which task indicates to the nurse that there is a developmental delay? 1 Copying a square 2 Hopping on one foot 3 Catching a ball reliably 4 Using a spoon effectively

4 Using a spoon effectively

While preparing a client for her first routine Papanicolaou (Pap) smear, a nurse determines that she appears anxious. What should the nurse include as part of the teaching plan? 1 Past statistics on the incidence of cervical cancer 2 Description of the early symptoms of cervical cancer 3 Explanation of why there is a small risk for cervical cancer 4 Verbal instructions that a Papanicolaou smear is effective in detecting precancerous and cancerous cells within the cervix

4 Verbal instructions that a Papanicolaou smear is effective in detecting precancerous and cancerous cells within the cervix

The nurse is teaching a group of new mothers regarding the benefits of breastfeeding. Which factor has a significant influence on the availability of milk in the lactating woman and should be included in the teaching? 1 Age of the woman at the time of delivery 2 Distribution of erectile tissue in the nipples 3 Amount of milk products consumed during pregnancy 4 Viewpoint of the woman's family toward breast-feeding

4 Viewpoint of the woman's family toward breast-feeding If the woman perceives that significant others in her life hold a negative view of breast-feeding, she may be tense, and the let-down reflex may not occur; a positive attitude on the part of significant others toward breastfeeding promotes relaxation and the let-down reflex.

While assessing a newborn, the nurse notes that the infant's skin is mottled. What should the nurse's primary intervention be? 1 Administer oxygen 2 Offer an oral feeding 3 Notify the practitioner 4 Warm the environment

4 Warm the environment Mottling results from hypothermia; the newborn should be wrapped, placed under a radiant warmer, or given to the mother for skin-to-skin contact. Mottling is a phenomenon that usually indicates a decreasing temperature; the newborn requires warming, not oxygenation or medical attention.

A woman arrives at the women's health clinic complaining of frequency and burning pain when voiding. The diagnosis is a urinary tract infection. What is important for the nurse to encourage the client to do? 1 Void every 2 hours. 2 Record fluid intake and urinary output. 3 Pour warm water over the vulva after voiding. 4 Wash the hands thoroughly after urinating and defecating.

4 Wash the hands thoroughly after urinating and defecating.

A nurse receives an order to give amoxicillin/clavulanate 50 mg/kg q4h to a 4-year-old child who weighs 35 pounds (16 kg). The medication label says that there is 200 mg of amoxicillin/clavulanate in 1 mL of normal saline. How much amoxicillin/clavulanate should the nurse give? Record your answer using a whole number.

4 mL

Children's patterns of play change as they grow from infancy through school age. Rank the order of appearance of each type of play, starting with infant play. 1. Associative 2. Cooperative 3. Parallel 4. Solitary

4, 3, 1, 2

What are indications that a child with acute epiglottis may be experiencing an airway obstruction? A) nasal flaring and bradycardia B) leaning forward with the chin thrust out C) leaning back and supporting self on hands and arms D) sore throat

B) leaning forward with the chin thrust out This is describing the tripod position.

Which of the following would most likely be contraindicated in the care of a child with CF? A) forced expiration B) supplemental oxygen as needed C) aerobic exercise D) percussion and postural drainage

B) supplemental oxygen as needed Oxygen shouldn't be delivered as desired. Don't give extra oxygen just because they want it because it will further promote decreased respiratory drive. In general, they should be participating in the other listed activities.

One minute after birth a nurse assesses a newborn and auscultates a heart rate of 90 beats/min. The newborn has a strong, loud cry, moves all extremities well, and has acrocyanosis but is otherwise pink. What is this neonate's Apgar score? A. 9 B. 8 C. 7 D. 6

B. 8 A heart rate slower than 100 beats/min receives 1 point, and color (acrocyanosis—body pink, extremities blue) receives 1 point; the respiratory rate (strong, loud cry), muscle tone, and reflex irritability each get a score of 2, for a total of 8. A score of 9 is too high. An Apgar score of 7 is too low, as is a score of 6.

A 7-year-old child is brought to the Emergency Department for an acute asthma attack. He is wheezing, tachypneic, diaphoretic, and looks frightened. The nurse should prepare to administer: A. IV methylprednisolone (Solu-medrol) B. Albuterol (Proventil) C. Cromolyn sodium (Intal) D. Ipratropium bromide (Atrovent)

B. Albuterol (Proventil) If this question was SATA, you would also give A and D BUT B would always be first. Always give albuterol FIRST to help open the lungs and allow for the other medications to work faster.

You are caring for a child with a pulse oximeter. The low O2 sat alarm goes off. What should you do first? A. Count the child's respiratory rate. B. Look for circumoral cyanosis. C. Observe the child's level of consciousness. D. See if the photodector is placed correctly.

C. Observe the child's level of consciousness. If the pulse ox alarm is sounding, your first task is to physically look at the patient and see if there is a logical explanation for why their O2 sats are dropping.

Which of the following represent early subtle signs of hypoxia? A) peripheral cyanosis B) hypotension C) central cyanosis D) change in level of consciousness

D) change in level of consciousness LOC is always one of the first indicators of hypoxia.

A 12-year-old with cystic fibrosis is to receive three Pancrease capsules five times a day. The nurse is aware that this medication is given: A. Promote excretion of fats B. Promote adequate oxygenation C. Prevent iron-deficiency anemia D. Facilitate utilization of nutrients

D. Facilitate utilization of nutrients Pancrease (krion) brings pancreatic enzymes so food can be broken down to absorb it and get the nutrients Goal is to get the nutrients not excrete

A child is admitted with epiglottitis, and needs an x-ray. The nurse orders the x-ray to be done by: A. portable x-ray in the unit's waiting room. B. transporting the child to the x-ray department by a cart. C. transporting the child to the x-ray department in a parent's arms. D. portable x-ray in the child's hospital room.

D. portable x-ray in the child's hospital room. Epiglottitis is a form of croup.

A nurse in the pediatric clinic is evaluating a 6-year-old child with sickle cell anemia whose spleen autoinfarcted by age 4. What is the priority nursing care at this time? 1 Monitoring for signs of jaundice 2 Assessing the abdomen frequently 3 Monitoring serial hematocrit readings 4 Determining parental knowledge about infection

Determining parental knowledge about infection

Where should the fundus be 1 day after birth in a client whose bladder is not distended?

One day after birth, the fundus is expected to be at the level of the umbilicus. In the first 12 hours after birth, the uterus is expected to be one fingerbreadth above the umbilicus. It is then expected to descend by approximately one fingerbreadth per day until it descends under the pubic bone, usually around day 10.

A 12-year-old child with sickle cell anemia is admitted during a vaso-occlusive crisis. What is the priority of care for this child? 1 Relieving pain 2 Exercising joints 3 Increasing urine output 4 Improving respirations

Relieving pain

Why does feeding position affect the risk of developing an ear infection in infants?

Short eustachian tubes makes it easier for fluid to collect in ears if they're laying down and creates a good medium for bacteria growth. You want to feed them when they are sitting upright.

One minute after birth a nurse notes that a newborn is crying, has a heart rate of 140 beats/min, is acrocyanotic, resists the suction catheter, and keeps the arms extended. What Apgar score should the nurse assign to the newborn?

The Apgar score is 8. A perfect score is 10; 1 point is deducted for lessened muscle tone (the baby's arms do not flex) and 1 point for acrocyanosis, which is manifested by bluish hands and feet.

A newborn of 30 weeks' gestation has a heart rate of 86 beats/min and slow, irregular respirations. The infant grimaces in response to suctioning, is cyanotic, and has flaccid muscle tone. What Apgar score should the nurse assign to this neonate?

This infant's Apgar score is 3. A heart rate of less than 100 beats/min receives 1 point; slow and irregular respirations receive 1 point; grimaces in response to suctioning receive 1 point; flaccid muscle tone receives 0 points; and cyanosis receives 0 points. A score of 2 is too low. A score of 4 is too high, as is a score of 5.

A 2-year-old child with previously diagnosed hemophilia is admitted to the pediatric unit for observation after a motor vehicle collision. The toddler has several bruises but no other apparent injuries. What is the nurse's specific concern regarding this child? 1 Risk for falls 2 Undetected injury 3 Deficient fluid volume 4 Development of infection

Undetected injury

When caring for a child with suspected epiglottis, the nurse should: a) have intubation equipment available b) visually inspect the child's oropharynx with a tongue blade c) obtain a throat culture d) prepare to administer immunizations

a) have intubation equipment available

The nurse concludes that a parent of an otherwise healthy child with varicella chickenpox) has an accurate understanding of the disease when the parent states: a. "I will send my child back to school when all of the lesions are dry and crusted over." b. "I will apply calamine lotion every hour to decrease itching." c. "I will give my child 80 mg of aspirin three times a day until the fever is gone." d. "I will take my child to our primary care provide to request antibiotics."

a. "I will send my child back to school when all of the lesions are dry and crusted over."

A mother brings 6-month-old Eric to the clinic for a well-baby checkup. She comments, "I want to go back to work, but I don't want Eric to suffer because I'll have less time with him." The nurse's most appropriate answer is: a. "Let's talk about the child care options that will be best for Eric." b. "You will need to stay home until Eric starts school." c. "I'm sure he'll be fine if you get a good baby-sitter." d. "You should go back to work so Eric will get used to being with others."

a. "Let's talk about the child care options that will be best for Eric."

The nurse is teaching the parent about the diet of a child experiencing severe edema associated with acute glomerulonephritis. Which information should the nurse include in the teaching? a. "You will need to avoid adding salt to your child's food." b. "You will need to decrease the number of calories in your child's diet." c. "Your child's diet will need an increased amount of protein." d. "Your child's diet will consist of low-fat, low-carbohydrate foods."

a. "You will need to avoid adding salt to your child's food."

At what age do most children have an adult concept of death as being inevitable, universal, and irreversible? Select one: a. 9 to 11 years b. 2 to 3 years c. 4 to 5years d. 6 to 8 years

a. 9 to 11 years

A primigravida is being monitored in her prenatal clinic for preeclampsia. What finding should concern her nurse? a. A dipstick value of 3+ for protein in her urine. b. Blood pressure (BP) increase to 138/86 mm Hg. c. Weight gain of 0.5 kg during the past 2 weeks. d. Pitting pedal edema at the end of the day.

a. A dipstick value of 3+ for protein in her urine.

Which clinical manifestation would most suggest acute appendicitis? a. Abdominal pain that is most intense at McBurney point b. Abdominal pain that is relieved by eating c. Bright red or dark red rectal bleeding d. Rebound tenderness

a. Abdominal pain that is most intense at McBurney point

A client at 40 weeks' gestation is admitted to the birthing unit, and an amniotomy is performed to facilitate labor. Once the nurse determines that the umbilical cord has not prolapsed, what is her next action? 1 Assessing the fetal heart rate 2 Obtaining the maternal vital signs 3 Turning the client on her left side 4 Monitoring the frequency of contractions

1 Assessing the fetal heart rate Once cord prolapse and consequent cord compression have been ruled out, it is imperative to evaluate the effect of the amniotomy on the fetus.

The urinary output of a 9-year-old child with acute glomerulonephritis decreases to 250 mL/24 hr. A diet low in sodium and potassium is prescribed. What should the nurse encourage the child to have for lunch? 1 Baked chicken, green beans, and lemonade 2 Cream of tomato soup, salami sandwich, and cola 3 Grilled cheese sandwich, sliced tomatoes, and milk 4 Peanut butter and jelly sandwich, celery, and orangeade

1 Baked chicken, green beans, and lemonade The foods in this grouping have the least sodium and potassium.

Which foods should the nurse include when teaching a group of school-age clients regarding appropriate nutritional intake? Select all that apply. 1 Bananas 2 Fried chicken 3 Low-fat yogurt 4 Whole-wheat dinner rolls 5 Sugary, carbonated beverages

1 Bananas 3 Low-fat yogurt 4 Whole-wheat dinner rolls

A client seeking family planning information asks the nurse during which phase of the menstrual cycle an intrauterine device (IUD) should be inserted. Before responding the nurse recalls that the insertion is usually performed when? 1 Between the first and fourth days of the cycle 2 Between the 5th and 11th days 3 Between the 14th and 16th days 4 Between the 25th and 28th days

1 Between the first and fourth days of the cycle An IUD should be inserted during menstruation because the cervical os is slightly dilated at this time; also, there is little chance of the woman's being pregnant.

A nurse suspects that a newborn has toxoplasmosis, one of the TORCH infections. How and when may it have been transmitted to the newborn? 1 In utero through the placenta 2 In the postpartum period through breast milk 3 During birth through contact with the maternal vagina 4 After the birth through a blood transfusion given to the mother

1 In utero through the placenta

During a home visit the nurse obtains information regarding a postpartum client's behavior and suspects that she is experiencing postpartum depression. Which assessments support this conclusion? Select all that apply. 1 Lethargy 2 Ambivalence 3 Emotional lability 4 Increased appetite 5 Long periods of sleep

1 Lethargy 2 Ambivalence 3 Emotional lability Insomnia, rather than long periods of sleep, is also associated with postpartum depression.

One hour postpartum a nurse assesses the amount of vaginal bleeding and determines that a client's uterus has become relaxed and boggy. Which intervention is a priority for the nurse to take in this situation? 1 Massage the uterus until firm. 2 Check the client's blood pressure. 3 Obtain a prescription for oxytocin. 4 Notify the primary healthcare provider immediately.

1 Massage the uterus until firm.

A 3-year-old boy in respiratory distress is treated in the emergency department. A diagnosis of acute spasmodic laryngitis (spasmodic croup) is made. At the time of discharge, the mother asks how to handle another attack at home. What should the nurse recommend? 1 Placing him near a cool-mist humidifier 2 Bringing him to the emergency department 3 Giving him an over-the-counter cough syrup 4 Offering him warm tea sweetened with honey

1 Placing him near a cool-mist humidifier During a spasmodic croup attack, cool humidified air to decrease inflammation is a fast home remedy.

The nurse is reevaluating a newborn who had an axillary temperature of 97° F (36.1° C) and was placed skin to skin with the mother. The newborn's axillary temperature is still 97° F (36.1° C) after 1 hour of skin-to-skin contact. Which intervention should the nurse implement next? 1 Placing the newborn under a radiant warmer in the nursery 2 Checking the newborn for a wet diaper and then continue the skin-to-skin contact 3 Leaving the newborn in skin-to-skin contact and rechecking the temperature in 1 hour 4 Double-wrapping the newborn in warm blankets and returning the newborn to a crib by the mother's bedside

1 Placing the newborn under a radiant warmer in the nursery The newborn's temperature should be kept in the normal range of 97.7° F to 99.5° F (36.5° C to 37.5° C). A hypothermic temperature that has not improved in 1 hour with the use of skin-to-skin contact requires additional measures. The infant should be placed under a radiant warmer for a short time until the temperature returns to the normal range.

Because preterm infants are at risk for respiratory distress syndrome, immediate nursing intervention is required when a preterm infant exhibits what sign? 1 Supraventricular retractions 2 Tachycardia of 160 beats/min 3 Respirations of 50 to 60 breaths/min 4 Neonatal Infant Pain Scale (NIPS) score of three

1 Supraventricular retractions Supraventricular retractions are a prominent feature of respiratory problems in preterm infants because of their compliant chest walls.

A pregnant client is concerned that she may have been infected with human immunodeficiency virus (HIV). Which information should a nurse include when counseling this client regarding HIV testing? Select all that apply. 1 The risks of passing the virus to the fetus 2 What positive or negative test results indicate 3 The risk factors for contracting HIV 4 The need for pregnant women to be tested for HIV 5 The emotional, legal, and medical implications of test results

1 The risks of passing the virus to the fetus 2 What positive or negative test results indicate 5 The emotional, legal, and medical implications of test results

A nurse caring for a pregnant client at 28 weeks' gestation and her partner suspects intimate partner violence. Which assessments support this suspicion? Select all that apply. 1 The woman has injuries to the breasts and abdomen. 2 The partner refuses to come into the examination room. 3 The partner answers questions that are asked of the woman. 4 The woman has visited the clinic several times in the last month. 5 The partner is excessively attentive while the health history is being taken.

1 The woman has injuries to the breasts and abdomen. 3 The partner answers questions that are asked of the woman. 4 The woman has visited the clinic several times in the last month.

Which toddler behaviors should the nurse identify as ritualism during the health history portion of the assessment? Select all that apply. 1 Using the same cup with each meal 2 Being able to use a spoon efficiently 3 Eating the same foods as other family members 4 Refusing to eat if the different foods are touching 5 Rejecting a meal because it is served in a different bowl

1 Using the same cup with each meal 4 Refusing to eat if the different foods are touching 5 Rejecting a meal because it is served in a different bowl

Name three differences between pediatric and adult respiratory systems that put children at great risk with a respiratory illness.

1) Everything is smaller- easier for infection to spread 2) Everything is narrower- any swelling at all causes significant decreased surface area for breathing 3) Ability to take a deep breath when they are SOB is decreased because they don't have as developed of chest muscles- rib cage is more flexible and doesn't allow them to take a deep breath 4) Immune system function isn't fully functional- antibodies from mother drops off at 3months

The nurse is providing nutritional guidance to the parents of a preschool-age client. Which parental comment would prompt the nurse to provide further education? 1 "We allow our child to drink only pasteurized apple cider." 2 "We let our child sample cookie dough while making cookies." 3 "We always wash our hands well before any food preparation." 4 "We use separate utensils for food preparation and for eating."

2 "We let our child sample cookie dough while making cookies."

A nurse in the pediatric clinic is assessing an 8-year-old child who has had asthma since infancy. What clinical finding requires immediate intervention? 1 Barrel chest 2 Audible wheezing 3 Heart rate of 105 beats/min 4 Respiratory rate of 30 breaths/min

2 Audible wheezing

When a nurse brings a newborn to the new mother, the mother comments about the milia on her infant's face. What information should the nurse include when responding? 1 They are common and will disappear in 2 to 3 days. 2 Avoid squeezing them and don't try to wash them off. 3 They are birthmarks that will disappear in 3 to 4 months. 4 Proper handwashing technique is important because milia are infectious.

2 Avoid squeezing them and don't try to wash them off. Attempts to remove milia will irritate the infant's skin, and such attempts are not needed because the milia will disappear during the first month of life. Although milia are common, they do not disappear for several weeks after birth. Milia are not birthmarks; the tiny plugged sebaceous glands are the result of maternal hormonal influence. The white material is not purulent and is not infectious.

A neonate born at 35 weeks' gestation has Apgar scores of 8 and 9. At 4 hours of age the newborn begins to experience respiratory distress, has a below-normal temperature in a warm environment, and has a low blood glucose level. What problem does the nurse suspect? 1 Hypoglycemia 2 Bacterial sepsis 3 Cocaine withdrawal 4 Meconium aspiration

2 Bacterial sepsis Preterm neonates react to infection with respiratory distress and subnormal temperatures. Although hypothermia is one sign of hypoglycemia, the newborn is not exhibiting other signs, such as tremors and lethargy.

A nurse is caring for a 2½-year-old child who is expressing pain. What is the most reliable indicator of this child's pain? 1 Crying and sobbing 2 Changes in behavior 3 Verbal exclamations of pain 4 Changes in pulse and respiratory rate

2 Changes in behavior Although there are several indicators of pain in children, a change in behavior is the one that occurs most often. Crying is not a valid indicator of pain; there is more than one cause for crying, including pain, separation, fear, and unhappiness.

What step should a nurse take when preparing to administer Rho(D) immune globulin to a postpartum client? 1 Start a primary intravenous (IV) line so that the drug may be administered via IV piggyback. 2 Ensure that the client is Rh negative and the neonate is Rh positive. 3 Obtain a syringe and needle appropriate for the subcutaneous injection. 4 Determine that the client has not eaten since midnight of the previous night.

2 Ensure that the client is Rh negative and the neonate is Rh positive. Rho(D) immune globulin is given to Rh-negative mothers not previously sensitized who have Rh-positive neonates; it prevents Rh incompatibility in the next pregnancy.

The school nurse is working with a child with a hearing deficit. The child arrives at school today without hearing aids. When the nurse talks with the child about the reasons for not wearing the aids, the nurse will need to ensure that the child understands what is being said. What actions by the nurse will promote effective communication? Select all that apply. 1 Speaking slower, louder than normal, and excessively fast 2 Facing the child directly when talking to the child 3 Avoiding chewing gum while communicating with the child 4 Avoiding using hand expressions that could interfere with lip reading 5 Moving from side to side while talking to the child to keep the child looking at the nurse

2 Facing the child directly when talking to the child 3 Avoiding chewing gum while communicating with the child

When assessing a toddler with Autism Spectrum Disorder (ASD), what characteristic findings or behaviors should the nurse expect? Select all that apply. 1 The desire to hug the nurse 2 Flat, blank facial expression 3 Laughing when pulse is taken 4 Inability to maintain eye contact 5 Enjoys climbing on stairs and furniture

2 Flat, blank facial expression 3 Laughing when pulse is taken 4 Inability to maintain eye contact

After an uneventful pregnancy a client at term arrives at the birthing unit. The nurse determines that her contractions are 10 minutes apart and that her cervix is dilated 2 cm. What stage of labor should the nurse document in the client's medical record? 1 Second stage 2 Latent first stage 3 Active first stage 4 Transition stage

2 Latent first stage Regular contractions occurring 10 minutes apart with a cervix dilated 2 cm indicate that the client is in the latent phase of the first stage of labor. The second stage of labor begins with full dilation and ends with expulsion of the fetus. Contractions occur more regularly and more frequently and the cervix is more dilated in the active stage of labor. Contractions are intense and occur every 1 to 2 minutes in the transition phase of the first stage of labor.

A home health nurse is caring for school-aged children in a family that is economically deprived. Which characteristic is most common to those living in poverty? 1 Open expression of anger 2 Long-term feeling of powerlessness 3 Willingness to postpone gratification 4 Compliance with health recommendations

2 Long-term feeling of powerlessness

The nurse is planning care for a school-aged child with autism spectrum disorder (ASD) who has been hospitalized for some tests. Which intervention should the nurse plan to implement? 1 Providing adequate stimulation through play 2 Placing the child in a private room 3 Encouraging staff to visit the child frequently 4 Giving detailed explanations about the upcoming tests

2 Placing the child in a private room The child with ASD should be placed in a private room. Decreasing stimulation by placing the child in a private room may lessen the disruptiveness of hospitalization. Play should be carefully planned; overstimulation can precipitate behavioral outbursts. Children with ASD need to be introduced slowly to new situations, with visits from staff caregivers kept short whenever possible.

A toddler who was admitted in acute respiratory distress is now resting quietly. The parents tell the nurse that they must leave. What should the nurse suggest that the parents do? 1 Try to "room in" to decrease the child's anxiety. 2 Plan to visit the child as frequently as possible. 3 Tell the child that they are leaving but will be back tomorrow. 4 Leave while the child is distracted to reduce the upset it may cause.

2 Plan to visit the child as frequently as possible. Because the parents have stated that they must leave, advising them to visit as often as possible takes into consideration the effect that separation will have on the toddler.

A client with severe preeclampsia is receiving magnesium sulfate therapy. What is the priority nursing assessment as the nurse monitors this client's response to therapy? 1 Urine output 2 Respiratory rate 3 Deep tendon reflexes 4 Level of consciousness

2 Respiratory rate Respiratory depression occurs with toxic levels of magnesium sulfate; calcium gluconate should be readily available to counteract toxicity. Although the other assessments (urine output, deep tendon reflexes, and level of consciousness) are important, none is the priority.

A 30-month-old toddler is brought to the emergency department in acute respiratory distress, and a diagnosis of laryngotracheobronchitis (viral croup) is made. What is the most important equipment for the nurse to have available when the child is admitted to the pediatric unit? 1 Intravenous set 2 Tracheotomy set 3 Nasal cannula for oxygen 4 Crib with padded side rails

2 Tracheotomy set A patent airway is the priority. A tracheotomy set should be kept immediately available in case of complete obstruction of the airway.

A nurse from the pediatric clinic who is strongly opposed to any chemical or mechanical method of birth control is asked to work in the family planning clinic. What is the most professional response that this nurse could give to the requesting supervisor? 1 "I will go, but it is against my beliefs and values." 2 "I won't do it, because I do not believe in birth control at all." 3 "I would prefer another assignment that is not contrary to my beliefs." 4 "I will have to stress that the rhythm method is the method of choice."

3 "I would prefer another assignment that is not contrary to my beliefs." Expressing a preference for another assignment that is not contrary to the nurse's beliefs is a positive negotiation to be reassigned to an area where the nurse's personal values will not pose a problem.

A nurse is teaching a postpartum client the characteristics of lochia and any deviations that should be reported immediately. Which client statement indicates that the teaching was effective? 1 "If I pass any clots, I'll notify the clinic." 2 "I'll call the clinic if my lochia changes from red to pink." 3 "I'll notify the clinic if my lochia starts to smell bad." 4 "If my vaginal discharge continues for 3 weeks, I'll call the clinic."

3 "I'll notify the clinic if my lochia starts to smell bad."

The parents of a 2-year-old child are watching the nurse administer the Denver II Developmental Screening Test to their child. They ask, "Why did you make our child draw on paper? We don't let our child draw at home." What is the best response by the nurse? 1 "I should have asked you about drawing first." 2 "These drawings help us determine your child's intelligence." 3 "It lets us test the child's ability to perform tasks requiring the hands." 4 "I don't understand why drawing is forbidden in your home."

3 "It lets us test the child's ability to perform tasks requiring the hands."

After an incomplete abortion, a client tells the nurse that although her primary healthcare provider explained what an incomplete abortion was, she did not understand. What is the best response by the nurse? 1 "I don't think you should focus on this anymore." 2 "It's when the fetus dies but is retained in the uterus for at least 2 months." 3 "It's when the fetus is expelled but other parts of the pregnancy remain in the uterus." 4 "I think it's best for you to ask your primary healthcare provider for the answer to that question."

3 "It's when the fetus is expelled but other parts of the pregnancy remain in the uterus." A correct and simple definition answers the question and fulfills the client's need to know. Telling the client not to focus on the topic any more denies the client's right to know. The definition of a missed abortion is when the fetus dies but is retained in the uterus for at least 2 months.

A nurse withholds methylergonovine maleate from a postpartum client. What clinical finding supports the withholding of the medication? 1 Urine output of 50 mL/hr 2 Third-degree perineal laceration 3 Blood pressure of 160/90 mm Hg 4 Respiratory rate of 12 breaths/min

3 Blood pressure of 160/90 mm Hg Methylergonovine maleate can cause hypertension and should not be given to a client with an increased blood pressure

During the assessment of a hospitalized infant, the nurse notes dry mucous membranes, absence of tears when the infant cries, and poor skin turgor. Which parameter will help the nurse further evaluate these findings? 1 Daily serum electrolytes 2 Respiratory rate and rhythm 3 Intake and output over the past 24 hours 4 Alterations in heart sounds since admission

3 Intake and output over the past 24 hours

A pregnant client with severe preeclampsia is receiving an infusion of magnesium sulfate. What does the nurse identify as the main reason that this medication is administered? 1 It acts as a diuretic. 2 It has a sedative effect. 3 It acts as an anticonvulsant. 4 It has an antihypertensive effect.

3 It acts as an anticonvulsant.

A nurse is assessing a newborn for signs of hyperbilirubinemia (pathologic jaundice). Which clinical finding confirms this complication? 1 Muscle irritability within 1 hour of birth 2 Neurologic signs during the first 24 hours 3 Jaundice that develops in the first 12 to 24 hours 4 Jaundice that develops between 48 and 72 hours after birth

3 Jaundice that develops in the first 12 to 24 hours

A nurse is teaching a class for staff members working in a group home about the cognitive development of children with cognitive impairments. What concept can these children probably learn the fastest? 1 Love versus hate 2 Life versus death 3 Large versus small 4 Right versus wrong

3 Large versus small

A child who recently returned from a three-day camping trip over spring vacation is brought to the clinic after a rash, chills, and low-grade fever develop. What are the most important data for the nurse to assess when taking the child's history? Select all that apply. 1 Date of return to school 2 Sports played on camping trip 3 Tendency to allergic reactions 4 Duration of signs and symptoms 5 Recent exposure to poison oak or ivy

3 Tendency to allergic reactions 4 Duration of signs and symptoms 5 Recent exposure to poison oak or ivy

The nurse is assessing a newborn and anticipates that the newborn has renal impairment. Which finding supports the nurse's conclusion? 1 The newborn has odorless urine. 2 The newborn has colorless urine. 3 The newborn first voids after 76 hours. 4 The newborn's urine has a specific gravity of 1.020.

3 The newborn first voids after 76 hours. A newborn should void within 24 hours. However, in this case, the newborn first voids after 76 hours, indicating renal impairment.

A primigravida client with type 1 diabetes is attending her first prenatal visit. While discussing changes in insulin needs during pregnancy and after birth, the nurse explains that in light of the client's blood glucose readings she should expect to increase her insulin dosage. Between which weeks of gestation is this expected to occur? 1 Tenth and twelfth weeks of gestation 2 Eighteenth and twenty-second weeks of gestation 3 Twenty-fourth and twenty-eighth weeks of gestation 4 Thirty-sixth and fortieth weeks of gestation

3 Twenty-fourth and twenty-eighth weeks of gestation At the end of the second trimester and the beginning of the third trimester, insulin needs increase because of an increase in maternal resistance to insulin.

The nurse is assessing a client with a tentative diagnosis of hydatidiform mole. Which clinical finding should the nurse anticipate? 1 Hypotension 2 Decreased fetal heart rate 3 Unusual uterine enlargement 4 Painless, heavy vaginal bleeding

3 Unusual uterine enlargement The proliferation of trophoblastic tissue filled with fluid causes the uterus to enlarge more quickly than if a fetus were in the uterus. Hypertension, not hypotension, often occurs with a molar pregnancy. There is no fetus within a hydatidiform mole. There may be slight painless vaginal bleeding.

According to Freud's theory, in what order do the stages of child development occur? 1. Anal 2. Latency 3. Oral 4. Genital 5. Oedipal

3, 1, 5, 2, 4 Freud's theory sets forth five stages of child development. The oral stage (stage 1) spans the period from birth to 12 to 18 months. The anal stage (stage 2) occurs between the ages of 18 months and 3 years. The Oedipal stage (stage 3) occurs between 3 and 6 years of age. The latency stage (stage 4) occurs between the ages of 6 and 12 years. Finally, the genital stage (stage 5) lasts from puberty through adulthood.

Which statement helps the nurse determine that a woman with genital herpes (HSV-2) understands her self-care in regards to this infection? 1 "When I have a baby, I don't want a cesarean." 2 "I can have sex as soon as the herpes sores have healed." 3 "When I finish the acyclovir prescription I will be cured." 4 "I must be careful when I have sex because herpes is a lifelong problem."

4 "I must be careful when I have sex because herpes is a lifelong problem."

A pregnant client has a positive group B Streptococcus (GBS) test at 36 weeks' gestation. What is the priority instruction that the nurse will include in the client's teaching plan? 1 "Go straight to the outpatient area of the maternity unit for a nonstress test." 2 "You'll need to schedule visits twice a week with your healthcare provider until you deliver." 3 "Your baby will have to spend at least 3 days in the neonatal intensive care unit because of this infection." 4 "This information will be in your prenatal record; however, please remind your labor and delivery nurse of this finding."

4 "This information will be in your prenatal record; however, please remind your labor and delivery nurse of this finding."

A client expresses a desire to breast-feed her preterm neonate, who is in the neonatal intensive care unit (NICU). The client states that she will pump her breasts until her baby is ready to breast-feed. The infant has been sucking on a pacifier for 1 week in accordance with protocol. How should the nurse respond to the mother's request? 1 By telling the client that this is unnecessary because the infant is being fed by gavage 2 By discouraging the client because of the time and effort it will take to pump her breasts 3 By instructing the client that breast milk is inadequate because it does not contain the necessary nutrients 4 By supporting the client's decision and explaining that the infant may lie close to her breast for nippling as desired

4 By supporting the client's decision and explaining that the infant may lie close to her breast for nippling as desired

Which neurologic manifestation should the nurse anticipate for a toddler-age client exposed to low doses of lead? 1 Coma 2 Paralysis 3 Convulsions 4 Learning difficulties

4 Learning difficulties

Which drug is administered to women after delivery to prevent postpartum uterine atony and hemorrhage but is not given to augment labor? 1 Dinopristone 2 Mifepristone 3 Indomethacine 4 Methylergonovine

4 Methylergonovine

A 42-year-old client undergoes amniocentesis during the 16th week of gestation because of concern about Down syndrome. Which additional information about the fetus will examination of the amniotic fluid reveal at this time? 1 Lung maturity 2 Type 1 diabetes 3 Cardiac anomaly 4 Neural tube defect

4 Neural tube defect

A grand multipara at 34 weeks' gestation is brought to the emergency department because of vaginal bleeding. The nurse suspects that the client has a placenta previa. Which characteristic typical of placenta previa supports the nurse's conclusion? 1 Painful vaginal bleeding in the first trimester 2 Painful vaginal bleeding in the third trimester 3 Painless vaginal bleeding in the first trimester 4 Painless vaginal bleeding in the third trimester

4 Painless vaginal bleeding in the third trimester Placenta previa is typically associated with painless uterine bleeding in the latter half of pregnancy

The parents of a preschooler tell the nurse that they try to inculcate good eating habits by asking the child to be at the table until the "plate is clean." What condition is the child at risk for? 1 Anorexia 2 Depression 3 Aggression 4 Poor eating habits

4 Poor eating habits

A woman who is 28 weeks pregnant calls the clinic to report that she is frightened because she has begun leaking breast milk. What is the best response? 1 She needs to come in for a calcium level. 2 She needs to come in for a nonstress test. 3 She needs to get off her feet and rest more. 4 This can be a normal occurrence during pregnancy.

4 This can be a normal occurrence during pregnancy.

A 17-year-old high school student with a history of asthma is brought to the emergency department. The nurse performs a physical assessment and identifies that the adolescent is experiencing an acute asthma exacerbation. Identify the assessments that support this conclusion. (Select all that apply.) A. Temperature 99.6 oF oral B. Dry, non-productive cough C. Wheezing on expiration D. Oxygen saturation 91% on room air E. Respiratory rate 40

B. Dry, non-productive cough C. Wheezing on expiration D. Oxygen saturation 91% on room air E. Respiratory rate 40 17 year old should have normal adult RR of 12-18. Medications that can be anticipated are a duoneb and steroids Duoneb = albuterol and Atrovent

What intervention is appropriate for a child with bronchiolitis? A. Place the infant upright with the neck flexed B. Ensure that the assigned RN does not care for other high-risk children C. Administered pancreatic supplements with meals and snacks. D. Place the child on airborne precautions.

B. Ensure that the assigned RN does not care for other high-risk children Bronciolitis is highly contagious so you don't want the RN caring for a child with this disease and also a high risk patient. A is wrong because with the neck flexed it would be harder to breathe.

The nurse encourages the mother of a toddler with acute laryngotracheobronchitis (LTB) who is in the ER to stay at the bedside as much as possible. The nurse's rationale for this action is primarily that: A. Mothers of hospitalized toddlers often experience guilt. B. The mother's presence will reduce anxiety and ease child's respiratory efforts. C. Separation from mother is a major developmental threat at this age. D. The mother can provide constant observations of the child's respiratory efforts.

B. The mother's presence will reduce anxiety and ease child's respiratory efforts.

A child is admitted to the hospital with the diagnosis of laryngotracheobronchitis (LTB). The nurse should be prepared to: A. administer antibiotics and assist with possible intubation. B. administer nebulized epinephrine and oral or IM dexamethasone. C. swab the throat for a throat culture. D. obtain a sputum specimen.

B. administer nebulized epinephrine and oral or IM dexamethasone. laryngotracheobronchitis is a form of croup. Typically croup is VIRAL so would not give ATBs. Swabbing for throat culture can cause irritation. Sputum specimen would only be if we suspected bacterial. Racemic epinephrine = nebulized ephinephrine Dexamethasone = steroid; want systemic steroids to rapidly decrease inflammation

What should the plan of care include to minimize the potential for a sickling episode in a child with sickle cell anemia? 1 Providing an iron-rich diet 2 Ensuring hemoconcentration 3 Enforcing periods of quiet play 4 Promoting adequate oxygenation

Promoting adequate oxygenation

An 18-month-old child is seen in the clinic is diagnosed with acute otitis media (AOM). Oral amoxicillin is prescribed. Which statement made by the parent indicates a correct understanding of the instructions? a. "I should administer all the prescribed medication." b. "I will immediately stop giving medication if I notice a change in hearing." c. "I will stop giving medication if fever is still present in 24 hours." d. "I should continue medication until the symptoms subside."

a. "I should administer all the prescribed medication."

A nurse is teaching nursing students the physiology of congenital heart defects. Which defect results in decreased pulmonary blood flow? Select one: a. Tetralogy of Fallot b. Patent ductus arteriosus c. Atrial septal defect d. Ventricular septal defect

a. Tetralogy of Fallot

The cheese-like, whitish substance that fuses with the epidermis and serves as a protective coating is called: Select one: a. vernix caseosa. b. acrocyanosis. c. caput succedaneum. d. surfactant.

a. vernix caseosa.

The recommended treatment for the prevention of human immunodeficiency virus (HIV) transmission to the fetus during pregnancy is: Select one: a. zidovudine. b. ofloxacin. c. podophyllin. d. acyclovir.

a. zidovudine.

Why does the rate of respiratory infections increase from 3-6mos of age? a) the exposure to pathogens is decreased b) maternal antibodies have decreased c) the diameter of the airway is larger in the infant than that of an older child d) viral agents that are milder in older children are more severe in infants

b) maternal antibodies have decreased

A patient arrives at the clinic seeking emergency contraception 24 hours after she had unprotected intercourse. The nurse correctly responds by saying: a. " the pills must be started the morning after unprotected intercourse." b. " Come right in so we can get you started." c. "You need to wait until you miss your period" d. "you must wait 72 hours before the pill will work"

b. " Come right in so we can get you started."

Which pain assessment tool should be used with a neonate? a. Wong-Baker Faces Scale b. NIPS c. Numeric Scale d. Visual analog scale

b. NIPS

A laboring woman received an opioid agonist (meperidine) intravenously 90 minutes before she gave birth. Which medication should be available to reduce the postnatal effects of Demerol on the neonate? a. Nalbuphine (Nubain) b. Naloxone (Narcan) c. Fentanyl (Sublimaze) d. Promethazine (Phenergan)

b. Naloxone (Narcan)

A new mother recalls from prenatal class that she should try to feed her newborn daughter when she exhibits feeding readiness cues rather than waiting until her infant is crying frantically. On the basis of this information, this woman should feed her infant about every 2.5 to 3 hours when she: Select one: a. waves her arms in the air. b. makes sucking motions. c. stretches her legs out straight. d. has hiccups.

b. makes sucking motions.

On examining a woman who gave birth 5 hours ago, the nurse finds that the woman has completely saturated a perineal pad within 15 minutes. The nurse's first action is to: a. begin an intravenous (IV) infusion of Ringer's lactate solution. b. massage the woman's fundus. c. assess the woman's vital signs. d. call the woman's primary health care provider.

b. massage the woman's fundus.

On examining a woman who gave birth 5 hours ago, the nurse finds that the woman has completely saturated a perineal pad within 15 minutes. The nurse's first action is to: Select one: a. begin an intravenous (IV) infusion of Ringer's lactate solution. b. massage the woman's fundus. c. assess the woman's vital signs. d. call the woman's primary health care provider.

b. massage the woman's fundus.

The priority nursing care associated with an oxytocin (Pitocin) infusion is: a. evaluating cervical dilation. b. monitoring uterine response. c. measuring urinary output. d. increasing infusion rate every 30 minutes.

b. monitoring uterine response.

The priority nursing care associated with an oxytocin (Pitocin) infusion is: a. measuring urinary output. b. monitoring uterine response. c. increasing infusion rate every 30 minutes. d. evaluating cervical dilation.

b. monitoring uterine response.

A first-time father is changing the diaper of his 1-day-old daughter. He asks the nurse, "What is this black, sticky stuff in her diaper?" The nurse's best response is: Select one: a. "Oh, don't worry about that. It's okay." b. "That means your baby is bleeding internally." c. "That's meconium, which is your baby's first stool. It's normal." d. "That's transitional stool."

c. "That's meconium, which is your baby's first stool. It's normal."

The nurse inserts and IV in the arm of a 2-year-old. What would be an expected response from the child? a. Facial grimacing and decreased po intake b. Tries to be brave and not cry if that's what he/she believes the nurse expects it. c. Crying and trying to hit the nurse d. Begging the nurse to put the IV in at a later time

c. Crying and trying to hit the nurse

A neonate is born with mild clubfeet. When the parents ask the nurse how this will be corrected, the nurse should base the explanation on what fact? Select one: a. Traction is tried first. b. Surgical intervention is needed. c. Frequent, serial casting is tried first. d. Children outgrow this condition when they learn to walk.

c. Frequent, serial casting is tried first.

Which major neonatal complication is carefully monitored after the birth of the infant of a diabetic mother? a. Hypoinsulinemia b. Hypobilirubinemia c. Hypoglycemia d. Hypercalcemia

c. Hypoglycemia

What is the priority nursing intervention for a child hospitalized with hemarthrosis resulting from hemophilia? a. Administration of acetaminophen for pain relief b. Assessment of the impact of hospitalization on the family system c. Immobilization and elevation of the affected joint d. Assessment of the child's response to hospitalization

c. Immobilization and elevation of the affected joint

What is a common side effect of corticosteroid therapy? a. Fever b. Hypertension c. Increased appetite d. Weight loss

c. Increased appetite

Which statement correctly describes β-thalassemia major (Cooley's anemia)? Select one: a. All formed elements of the blood are depressed. b. Inadequate numbers of red blood cells are present. c. Increased incidence occurs in families of Mediterranean extraction. d. Increased incidence occurs in persons of West African descent.

c. Increased incidence occurs in families of Mediterranean extraction.

Which minerals and vitamins usually are recommended to supplement a pregnant woman's diet? a. Calcium and zinc b. Fat-soluble vitamins A and D c. Iron and folate d. Water-soluble vitamins C and B6

c. Iron and folate

The nurse is assessing a child who was just admitted to the hospital for observation after a head injury. What is the most essential part of the nursing assessment to detect early signs of a worsening condition? Select one: a. Vital signs b. Posturing c. Level of consciousness d. Focal neurologic signs

c. Level of consciousness

An infant diagnosed with pyloric stenosis experiences excessive vomiting that can result in which condition? a. Hyperchloremia b. Hypernatremia c. Metabolic alkalosis d. Metabolic acidosis

c. Metabolic alkalosis

What is an advantage of peritoneal dialysis? Select one: a. Dietary limitations are not necessary. b. Treatments are done in hospitals. c. Parents and older children can perform treatments. d. Protein loss is less extensive.

c. Parents and older children can perform treatments.

The head-to-tail direction of growth is referred to as: a. sequential. b. proximodistal. c. cephalocaudal. d. mass to specific.

c. cephalocaudal.

The nurse providing newborn stabilization must be aware that the primary side effect of maternal narcotic analgesia in the newborn is: a. tachypnea. b. acrocyanosis. c. respiratory depression. d. bradycardia.

c. respiratory depression.

What is the primary result of anemia? Select one: a. Presence of abnormal hemoglobin. b. Depressed hematopoietic system. c. Increased blood viscosity. d. Decreased oxygen-carrying capacity of blood.

d. Decreased oxygen-carrying capacity of blood.

Which of the following statements about the various forms of hepatitis is accurate? Select one: a. A vaccine exists for hepatitis C but not for hepatitis B. b. Hepatitis B is less contagious than human immunodeficiency virus (HIV). c. The incidence of hepatitis C is decreasing. d. Hepatitis A is acquired by eating contaminated food or drinking polluted water.

d. Hepatitis A is acquired by eating contaminated food or drinking polluted water.

Which drug would be used to treat a child who has increased intracranial pressure (ICP) resulting from cerebral edema? a. Atropine sulfate b. Sodium bicarbonate c. Epinephrine hydrochloride d. Mannitol

d. Mannitol

The nurse is caring for a child diagnosed with an acute respiratory infection. What nursing intervention should be included in the plan of care? a. Asses respiratory rate once per shift b. Instruct patient to eat a high-protein diet c. Enocurage at least 4 ounces of fluid per hour d. Monitor pulse oximetry continously

d. Monitor pulse oximetry continously

What food choice by the parent of a 2-year-old child with celiac disease indicates a need for further teaching? a. Corn muffin b. Meat patty c. Rice cake d. Oatmeal

d. Oatmeal

A child with pulmonary atresia exhibits cyanosis with feeding. On reviewing this child's laboratory values, the nurse is not surprised to notice which abnormality? Select one: a. Dehydration b. Infection c. Anemia d. Polycythemia

d. Polycythemia

As part of the treatment for congestive heart failure, the child takes the diuretic furosemide. As part of teaching home care, the nurse encourages the family to give the child foods such as bananas, oranges, and leafy vegetables. These foods are recommended because they are high in what electrolyte? Select one: a. Chlorides b. Sodium c. Zinc d. Potassium

d. Potassium

What is the primary purpose of prescribing a histamine receptor antagonist for an infant diagnosed with gastroesophageal reflux? a. Prevent reflux b. Increase gastric acid production. c. Prevent hematemesis. d. Reduce gastric acid production.

d. Reduce gastric acid production.

Which action by the school nurse is important in the prevention of rheumatic fever? Select one: a. Recommend salicylates instead of acetaminophen for minor discomforts. b. Conduct routine blood pressure screenings. c. Encourage routine cholesterol screenings. d. Refer children with sore throats for throat cultures.

d. Refer children with sore throats for throat cultures.

Which nursing intervention is appropriate to assess for neurovascular competency in a child suspected of experiencing compartment syndrome? Select one: a. The degree of motion and ability to position the extremity. b. The amount of swelling noted in the extremity and pain intensity. c. The length, diameter, and shape of the extremity. d. The skin color, temperature, movement, sensation, and capillary refill of the extremity.

d. The skin color, temperature, movement, sensation, and capillary refill of the extremity.

The nurse understands that hypospadias refers to what urinary anomaly? a. Penis shorter than usual for age. b. Urethral opening along dorsal surface of penis. c. Absence of a urethral opening. d. Urethral opening along ventral surface of penis.

d. Urethral opening along ventral surface of penis.

What is a common sign of digoxin toxicity? Select one: a. Bradypnea b. Seizures c. Tachycardia d. Vomiting

d. Vomiting

Screening at 24 weeks of gestation reveals that a pregnant woman has gestational diabetes mellitus (GDM). In planning her care, the nurse and the woman mutually agree that an expected outcome is to prevent injury to the fetus as a result of GDM. The nurse identifies that the fetus is at greatest risk for: a. preterm birth. b. congenital anomalies of the central nervous system. c. low birth weight. d. macrosomia.

d. macrosomia.

What name is given to identify an acquired hemorrhagic disorder that is characterized by excessive destruction of platelets? Select one: a. Immune thrombocytopenia b. Thalassemia major c. Disseminated intravascular coagulation d. Aplastic anemia

i got this wrong :(

How is LMX or Emla used in pediatric pain management? Select one or more: a. It should be injected subcutaneously near the preferred injection site prior to the painful procedure. b. It should be rubbed into the skin prior to the painful procedure. c. It should be applied to skin in a thick layer and covered with an occlusive dressing before the painful procedure. d. It should be given on an empty stomach before a painful procedure.

i got this wrong lol

Which parental statement regarding the use of car safety seats for a toddler-age child indicates the need for further education by the nurse? 1 "I should follow the manufacturer instructions regarding use." 2 "I should allow my child to ride unrestrained for short car trips." 3 "I should allow my child to have a toy for quiet play while in the car." 4 "I should encourage my child to help attach the buckles."

2 "I should allow my child to ride unrestrained for short car trips."

A primigravida in her seventh week of gestation asks the nurse when she can expect to feel her baby move. The nurse replies that quickening usually occurs in which week? 1 24th week 2 20th week 3 16th week 4 12th week

2 20th week

A 2-year-old boy born with cryptorchidism is to undergo orchiopexy. What should the nurse tell the parents about the anticipated outcome of this surgery? 1 The urine stream will be directed downward. 2 Damage to the undescended testicle will be prevented. 3 Fluid that has collected in the scrotum will be removed. 4 The fibrous tissue that has caused the penile deformity will be released.

2 Damage to the undescended testicle will be prevented.

A 40-year-old primigravida is scheduled to have her first abdominal ultrasound. What should the nurse's instructions include? 1 Postpone breakfast until after the test. 2 Drink water until bladder is full. 3 Empty the bladder immediately before the test. 4 Insert a suppository after arising on the day of the test.

2 Drink water until bladder is full. A full bladder raises the uterus above the pelvis, providing better visualization of its contents.

The nurse plans to assess a postpartum client's uterine fundus. What should the nurse ask the client to do before this assessment? 1 Drink fluids 2 Empty her bladder 3 Perform the Valsalva maneuver 4 Assume the semi-Fowler position

2 Empty her bladder Having the client empty her bladder will help ensure accurate assessment of fundal height. A full bladder may promote a boggy uterus and may elevate the uterus upward and toward the client's right side.

A woman's pregnancy has been uneventful, and she has gained 25 lb (11.3 kg). At term her hemoglobin level is 10.6 g/dL (106 mmol/L) and her hematocrit is 31%. What is the physiologic reason for these hemoglobin and hematocrit levels? 1 Infection 2 Hemodilution 3 Nutritional deficits 4 Concealed bleeding

2 Hemodilution The increase in circulating blood volume during pregnancy is reflected in lower hemoglobin and hematocrit readings (physiological anemia of pregnancy)

The nurse is assessing a new mother at a healthcare facility. Which symptom does the nurse identify as a risk factor for postpartum blues? 1 Frantic energy 2 Mild irritability 3 Hallucinations 4 Unwillingness to sleep

2 Mild irritability Postpartum blues are transient symptoms that a client may experience after childbirth. About 85% of women experience postpartum blues with symptoms of mild irritability, tearfulness, rapid mood fluctuations, and anxiety.

A 1-year-old infant is brought to the pediatric clinic for the first time. During the assessment the nurse suspects a developmental delay. What developmental milestone should have been achieved by this age? 1 Saying six words 2 Responding to peek-a-boo 3 Building a tower of two cubes 4 Pointing to things when they are named

2 Responding to peek-a-boo

The nurse is helping a mother breast-feed her newborn. What is the best indication that the newborn has achieved an effective attachment to the breast? 1 The tongue is securely on top of the nipple. 2 The mouth covers most of the areolar surface. 3 Loud sucking sounds are heard during the 15 minutes spent at each breast. 4 Vigorous suckling occurs for the 5 minutes the infant spends at each breast before falling asleep.

2 The mouth covers most of the areolar surface.

A new mother who is learning about infant feedings asks the nurse how anyone who is breast-feeding gets anything done with a baby feeding on demand. What is the best response by the nurse? 1 "Most mothers find that feeding whenever the baby cries works out fine." 2 "Perhaps a schedule would be better because the baby is already accustomed to the hospital routine." 3 "Babies on demand feedings eventually set a schedule, so there should be time for you to do other things." 4 "Most breast-feeding mothers find that their babies do better on demand because the amount of milk ingested varies from feeding to feeding."

3 "Babies on demand feedings eventually set a schedule, so there should be time for you to do other things."

A 6-year-old child is admitted to the pediatric unit with a diagnosis of nephrotic syndrome. What should the plan of care include during the acute phase? 1 Offering a low-protein diet 2 Encouraging fluids every hour 3 Promoting frequent position changes 4 Providing time for active play periods

3 Promoting frequent position changes Severe edema is usually present, and changes of position are necessary to prevent skin breakdown. A high-protein diet should be offered, although there is no evidence that it alters the outcome of the disorder. A low-protein diet is used for children with azotemia resulting from renal failure.

Which heart sound is normally heard in a toddler that is considered abnormal in an adult over 30-years-old? 1 S1 2 S2 3 S3 4 S4

3 S3

A client seeking advice regarding contraception asks a nurse to explain how an intrauterine device (IUD) prevents pregnancy. How should the nurse respond? 1 "It covers the entrance to the cervical os." 2 "The openings to the fallopian tubes are blocked." 3 "The sperm are kept from reaching the vagina." 4 "It produces a spermicidal intrauterine environment."

4 "It produces a spermicidal intrauterine environment."

At 39 weeks' gestation a client asks the nurse about the difference between true and false labor. Which information regarding true labor contractions should the nurse include in a response to the client's question? 1 Usually fluctuate in length 2 Continuous, without relaxation 3 Related to time of membrane rupture 4 Accompanied by progressive cervical dilation

4 Accompanied by progressive cervical dilation Progressive cervical dilation is the only positive sign of true labor; the cervix dilates in response to regular, coordinated uterine contractions. The contractions of true labor increase in length and intensity.

In which stage of development are food choices affected by such factors as peer acceptability and sociability, possibly resulting in malnutrition or chronic illness? 1 Midlife 2 Infancy 3 Childhood 4 Adolescence

4 Adolescence

The nurse is caring for a client who is in the first stage of labor. The fetal heart rate monitor displays an irregular baseline that was in the 150s and is now in the 130s with variability. What is the priority nursing intervention? 1 Administering oxygen 2 Notifying the primary healthcare provider 3 Changing the client's position 4 Continuing to monitor the client

4 Continuing to monitor the client This is an expected occurrence caused by the interplay of the sympathetic and parasympathetic nervous systems

When teaching a class about parenting, the nurse asks the participants what they do when their toddlers have a temper tantrum. Which statement demonstrates one father's understanding of the origin of temper tantrums? 1 After a temper tantrum he disciplines his child by restricting a favorite food or activity. 2 When a temper tantrum begins he isolates and ignores his child until the behavior improves. 3 During a temper tantrum he partially gives in to his child before the tantrum becomes excessive. 4 He tries to prevent a temper tantrum by allowing his child to choose between two reasonable alternatives.

4 He tries to prevent a temper tantrum by allowing his child to choose between two reasonable alternatives. This parent's action gives his child more control by allowing the child to make a decision. This demonstrates an understanding of what the toddler can and cannot do safely.

A nurse is caring for four postpartum clients, each with a different medical condition. Which condition will result in the primary healthcare provider advising the new mother not to breast-feed? 1 Mastitis 2 Inverted nipples 3 Herpes genitalis 4 Human immunodeficiency virus (HIV)

4 Human immunodeficiency virus (HIV)

What does the nurse state is the cause of frequent upper respiratory tract infections in toddlers? 1 Stress 2 Unhealthy diet 3 Lack of exercise 4 Immature immune system

4 Immature immune system

What is the best room assignment for a 5-year-old child admitted with injuries that may be related to abuse? 1 In an isolation room 2 With a friendly older child 3 With a child of the same age 4 In a room near the nurses' desk

4 In a room near the nurses' desk

The nurse is caring for a preterm infant who is receiving oxygen therapy. What should the nurse do to prevent retinopathy of prematurity (ROP)? 1 Cover the neonate's eyes with a shield 2 Place the neonate in an elevated side-lying position 3 Assess the neonate every hour with a pulse oximeter 4 Support the neonate's oxygen saturation while providing minimal FiO2

4 Support the neonate's oxygen saturation while providing minimal FiO2 ROP is a complex disease of the preterm infant; hyperoxemia is one of the numerous causes implicated. Oxygen therapy is maintained at the lowest level necessary to support respiratory status.

A male newborn has been exposed to human immunodeficiency virus (HIV) in utero. Which assessment supports the diagnosis of HIV infection in the newborn? 1 Delay in temperature regulation 2 Continued bleeding after circumcision 3 Hypoglycemia within the first day of birth 4 Thrush that does not respond readily to treatment

4 Thrush that does not respond readily to treatment

After her baby's birth a client wishes to begin breast-feeding as soon as possible. How can the nurse best assist the client at this time? 1 Giving the infant a bottle first to evaluate the sucking reflex 2 Positioning the infant to grasp the nipple to express colostrum 3 Leaving the infant and parents alone to promote attachment behaviors 4 Touching the infant's cheek adjacent to the nipple to elicit the rooting reflex

4 Touching the infant's cheek adjacent to the nipple to elicit the rooting reflex

A 25-year-old single woman comes to the gynecologist's office for a follow-up visit related to her abnormal Papanicolaou (Pap) smear. The test revealed that the patient has human papillomavirus (HPV). The patient asks, "What is that? Can you get rid of it?" Your best response is: Select one: a. "HPV stands for 'human papillomavirus.' It is a sexually transmitted infection (STI) that may lead to cervical cancer. There is no known cure but symptoms are treated." b. "You probably caught this from your current boyfriend. He should get tested for this." c. "It's just a little lump on your cervix. We can freeze it off." d. "HPV is a type of early human immunodeficiency virus (HIV). You will die from this."

a. "HPV stands for 'human papillomavirus.' It is a sexually transmitted infection (STI) that may lead to cervical cancer. There is no known cure but symptoms are treated."

The nurse is discussing storage of breast milk with a mother whose infant is preterm and in the special care unit. What statement would indicate that the mother needs additional teaching? Select one: a. "I can store my breast milk in the freezer for 3 months." b. "I can store my breast milk in the refrigerator for 3 to 5 days." c. "I can store my breast milk at room temperature for 8 hours." d. "I can store my breast milk in the refrigerator for 3 months."

a. "I can store my breast milk in the freezer for 3 months."

When it is generally recommended that a child being treated for acute streptococcal pharyngitis may return to school? a. After taking antibiotics for 24 hours b. If no complications develop c. When the sore throat is better d. After taking antibiotics for 3 days

a. After taking antibiotics for 24 hours

Approximately 10% to 15% of all clinically recognized pregnancies end in miscarriage. Which is the most common cause of spontaneous abortion? a. Chromosomal abnormalities b. Infections c. Immunologic factors d. Endocrine imbalance

a. Chromosomal abnormalities

A woman gave birth to a healthy infant boy 5 days ago. What type of lochia would the nurse expect to find when assessing this woman? a. Lochia serosa b. Lochia sangra c. Lochia alba d. Lochia rubra

a. Lochia serosa

While evaluating the reflexes of a newborn, the nurse notes that with a loud noise the newborn symmetrically abducts and extends his arms, his fingers fan out and form a "C" with the thumb and forefinger, and he has a slight tremor. The nurse would document this finding as a positive: Select one: a. Moro reflex. b. Babinski reflex. c. glabellar (Myerson) reflex. d. tonic neck reflex.

a. Moro reflex.

Which finding 12 hours after birth requires further assessment? a. The fundus is palpable two fingerbreadths above the umbilicus. b. The fundus is palpable at the level of the umbilicus. c. The fundus is palpable one fingerbreadth below the umbilicus. d. The fundus is palpable two fingerbreadths below the umbilicus.

a. The fundus is palpable two fingerbreadths above the umbilicus.

Prenatal testing for human immunodeficiency virus (HIV) is recommended for: a. all women, regardless of risk factors. b. a woman who has had a sexually transmitted infection. c. a woman who has had more than one sexual partner. d. a woman who is monogamous with her partner.

a. all women, regardless of risk factors.

As relates to the use of tocolytic therapy to suppress uterine activity, nurses should be aware that: a. its most important function is to afford the opportunity to administer antenatal glucocorticoids. b. if the patient develops pulmonary edema while receiving tocolytics, intravenous (IV) fluids should be given. c. there are no important maternal (as opposed to fetal) contraindications. d. the drugs can be given efficaciously up to the designated beginning of term at 37 weeks.

a. its most important function is to afford the opportunity to administer antenatal glucocorticoids.

A maternal indication for the use of forceps is: a. maternal exhaustion. b. a wide pelvic outlet. c. a history of rapid deliveries. d. failure to progress past 0 station.

a. maternal exhaustion.

It is important for the nurse to develop a realistic birth plan with the pregnant woman in her care. The nurse can explain that a major advantage of nonpharmacologic pain management is: a. no side effects or risks to the fetus are involved. b. a more rapid labor is likely. c. greater and more complete pain relief is possible. d. the woman remains fully alert at all times.

a. no side effects or risks to the fetus are involved.

A woman who is 14 weeks pregnant tells the nurse that she always had a glass of wine with dinner before she became pregnant. She has abstained during her first trimester and would like to know if it is safe for her to have a drink with dinner now. The nurse would tell her: a. "One drink every night is too much. One drink three times a week should be fine." b. "Because no one knows how much or how little alcohol it takes to cause fetal problems, the best course is to abstain throughout your pregnancy." c. "Since you're in your second trimester, you can drink as much as you like." d. "Since you're in your second trimester, there's no problem with having one drink with dinner."

b. "Because no one knows how much or how little alcohol it takes to cause fetal problems, the best course is to abstain throughout your pregnancy."

Which description of a stool is characteristic of intussusception? a. Loose, foul-smelling stools b. "Currant jelly" stools c. Hard stools positive for guaiac d. Ribbon-like stools

b. "Currant jelly" stools

What is the characteristic of the immune-mediated type 1 diabetes mellitus? Select one: a. Oral agents are often effective for treatment b. Age at onset is usually younger than 18 years c. Ketoacidosis is infrequent d. Onset is gradual

b. Age at onset is usually younger than 18 years

Which viral sexually transmitted infection is characterized by a primary infection followed by recurrent episodes? Select one: a. Cytomegalovirus (CMV) b. Herpes simplex virus (HSV)-2 c. Human immunodeficiency virus (HIV) d. Human papillomavirus (HPV)

b. Herpes simplex virus (HSV)-2

Which statement is true about the term contraceptive failure rate? a. It increases over time as couples become more careless. b. It varies from couple to couple, depending on the method and the users. c. It refers to the minimum level that must be achieved to receive a government license. d. It refers to the percentage of users expected to have an accidental pregnancy over a 5-year span.

b. It varies from couple to couple, depending on the method and the users.

In caring for the preterm infant, what complication is thought to be a result of high arterial blood oxygen level? Select one: a. Intraventricular hemorrhage (IVH) b. Retinopathy of prematurity (ROP) c. Necrotizing enterocolitis (NEC) d. Bronchopulmonary dysplasia (BPD)

b. Retinopathy of prematurity (ROP)

What is the major focus of the therapeutic management for a child with lactose intolerance? a. Administration of daily normal saline enemas b. Teaching dietary modifications c. Compliance with the medication regimen d. Providing emotional support to family members

b. Teaching dietary modifications

The priority nursing intervention after an amniotomy should be to: a. estimate the amount of amniotic fluid. b. assess the fetal heart rate. c. assess the color of the amniotic fluid. d. change the patient's gown.

b. assess the fetal heart rate.

The nurse caring for the after birth woman understands that breast engorgement is caused by: Select one: a. overproduction of colostrum. b. congestion of veins and lymphatics. c. hyperplasia of mammary tissue. d. accumulation of milk in the lactiferous ducts.

b. congestion of veins and lymphatics.

It is important for the nurse to develop a realistic birth plan with the pregnant woman in her care. The nurse can explain that a major advantage of nonpharmacologic pain management is: a. the woman remains fully alert at all times. b. no side effects or risks to the fetus are involved. c. greater and more complete pain relief is possible. d. a more rapid labor is likely.

b. no side effects or risks to the fetus are involved.

A couple comes to the family practice clinic and asks about sterilization procedures. Which question by the nurse should determine whether this method of family planning would be most appropriate? a. "Do either of you have diabetes mellitus?" b. " Do either of you have a problem with high blood pressure?" c. "Do you plan to have other children?" d. "Have you ever had surgery?"

c. "Do you plan to have other children?"

The nurse uses the palms of the hands when handling a wet cast to achieve what outcome? Select one: a. Keep the patient's limb balanced b. Facilitate easy turning c. Avoid indenting the cast d. Assess dryness of the cast

c. Avoid indenting the cast

A young child with human immunodeficiency virus is receiving several antiretroviral drugs. What is the expected outcome of these drug therapies? Select one: a. Prevent spread of disease b. Cure the disease c. Delay disease progression d. Treat Pneumocystis jiroveci pneumonia

c. Delay disease progression

An African-American woman noticed some bruises on her newborn girl's buttocks. She asks the nurse who spanked her daughter. The nurse explains that these marks are called: Select one: a. vascular nevi. b. nevus flammeus. c. Mongolian spots. d. lanugo.

c. Mongolian spots.

What is the most common mode of transmission of human immunodeficiency virus (HIV) in the pediatric population? a. Sexual abuse b. Blood transfusions c. Perinatal transmission d. Poor hand washing

c. Perinatal transmission

The nurse is assessing a child with herpetic (HSV - Type I) gingivostomatitis. The nurse wears gloves when examining the lesions. This action is: a. unnecessary because the virus is sexually transmitted. b. unnecessary because the virus is not easily spread. c. necessary because the virus can easily enter breaks in the skin. d. necessary only if the nurse touches his/her own mouth.

c. necessary because the virus can easily enter breaks in the skin.

The first and most important nursing intervention when a nurse observes profuse after birth bleeding is to: a. assess maternal blood pressure and pulse for signs of hypovolemic shock. b. administer the standing order for an oxytocic. c. palpate the uterus and massage it if it is boggy. d. call the woman's primary health care provider.

c. palpate the uterus and massage it if it is boggy.

Postpartal overdistention of the bladder and urinary retention can lead to which complications? Select one: a. Urinary tract infection and uterine rupture b. Fever and increased blood pressure c. After birth hemorrhage and eclampsia d. After birth hemorrhage and urinary tract infection

d. After birth hemorrhage and urinary tract infection

The nurse is explaining the benefits associated with breastfeeding to a new mother. Which statement by the nurse would be inaccurate and provide conflicting information to the patient? Select one: a. Breastfeeding may enhance after birth weight loss. b. Women who breastfeed have a decreased risk of breast cancer. c. Breastfeeding increases bone density. d. Breastfeeding is an effective method of birth control.

d. Breastfeeding is an effective method of birth control.

Which is the most common congenital anomaly associated with Down syndrome? a. Hypospadias b. Congenital hip dysplasia c. Pyloric stenosis d. Heart defects

d. Heart defects

Exophthalmos may occur in children with what diagnosis? Select one: a. Hyperparathyroidism b. Hypoparathyroidism c. Hypothyroidism d. Hyperthyroidism

d. Hyperthyroidism

An 8-month-old infant becomes hypercyanotic while blood is being drawn. What should be the nurse's first action? Select one: a. Assess for neurologic defects b. Prepare the family for imminent death c. Begin cardiopulmonary resuscitation d. Place the child in the knee-chest position

d. Place the child in the knee-chest position

Which condition would not be classified as a bleeding disorder in late pregnancy? a. Abruptio placentae b. Cord insertion c. Placenta previa d. Spontaneous abortion

d. Spontaneous abortion

Which term is used when a patient remains in a deep sleep, responsive only to vigorous and repeated stimulation? a. Obtundation b. Persistent vegetative state c. Coma d. Stupor

d. Stupor

A woman gave birth vaginally to a 9-lb, 12-ounce girl yesterday. Her primary health care provider has written orders for perineal ice packs, use of a sitz bath tid, and a stool softener. What information is most closely correlated with these orders? a. The woman is a gravida 2, para 2. b. The woman received epidural anesthesia. c. The woman had a vacuum-assisted birth. d. The woman has an episiotomy.

d. The woman has an episiotomy.

Necrotizing enterocolitis (NEC) is an inflammatory disease of the gastrointestinal mucosa. The signs of NEC are nonspecific. Some generalized signs include: Select one: a. scaphoid abdomen, no residual with feedings, and increased urinary output. b. hypertension, absence of apnea, and ruddy skin color. c. hypertonia, tachycardia, and metabolic alkalosis. d. abdominal distention, temperature instability, and grossly bloody stools.

d. abdominal distention, temperature instability, and grossly bloody stools.

Part of the health assessment of a newborn is observing the infant's breathing pattern. A full-term newborn's breathing pattern is predominantly: Select one: a. chest breathing with nasal flaring. b. diaphragmatic with chest retraction. c. deep with a regular rhythm. d. abdominal with synchronous chest movements.

d. abdominal with synchronous chest movements.

To prevent the abduction of newborns from the hospital, the nurse should: Select one: a. instruct the mother not to give her infant to anyone except the one nurse assigned to her that day. b. carry the infant when transporting him or her in the halls. c. restrict the amount of time infants are out of the nursery. d. apply an electronic and identification bracelet to mother and infant.

d. apply an electronic and identification bracelet to mother and infant.

A placenta previa in which the placental edge just reaches the internal os is more commonly known as: a. total. b. complete. c. partial. d. marginal.

d. marginal.

The nurse is providing teaching to the parents of a preschool-age client who is prescribed iron supplements for iron-deficiency anemia. Which parental statements indicate the need for further education? Select all that apply. 1 "We will mix the iron with milk to enhance absorption." 2 "We will mix the iron with black tea to enhance absorption." 3 "We will mix the iron with orange juice to enhance absorption." 4 "We will avoid giving our child green tea because it can decrease absorption." 5 "We will avoid feeding our child tomatoes because it can decrease absorption."

"We will mix the iron with milk to enhance absorption." "We will mix the iron with black tea to enhance absorption."

Which is often administered to prevent or control hemorrhage in a child diagnosed with cancer? Select one: a. Platelets b. Corticosteroids c. Nitrosoureas d. Whole blood

a. Platelets

Type 1 diabetes mellitus is suspected in an adolescent. Which clinical manifestation may be present? Select one: a. Poor wound healing b. Fluid overload c. Moist skin d. Weight gain

a. Poor wound healing

Absence or weakness of which reflexes tested during the newborn assessment should the nurse report to the health care provider? 1 Gag 2 Moro 3 Babinski 4 Tonic neck

1 Gag Absence or diminution of the gag reflex could be life threatening. The infant might aspirate mucus or a feeding.

After her baby undergoes corrective surgery for hypertrophic pyloric stenosis, the mother is asked to offer the first feeding. The infant sucks it eagerly and vomits immediately. What is the nurse's explanation to the mother? 1 "This often occurs after the first feeding." 2 "The baby is ridding postoperative mucus." 3 "Your feeding technique may need to be changed." 4 "Feedings will have to be stopped until peristalsis improves."

1 "This often occurs after the first feeding."

After a difficult vaginal birth, assessment of a full-term newborn reveals an unequal Moro reflex on one side and a flaccid arm in adduction. Which problem does the nurse suspect? 1 Brachial palsy 2 Supratentorial tear 3 Fracture of the clavicle 4 Developmental dysplasia of the hip

1 Brachial palsy Brachial palsy results from excessive stretching of the nerve fibers that run from the neck, through the shoulder, and down toward the arm; the muscles of the upper arm are involved, and the infant holds the arm at the side with the elbow extended and the hand rotated inward.

A nurse is reviewing previous education with the parents and a 12-year-old child with cystic fibrosis (CF). The disease was diagnosed when the child was 3 years old and has resulted in one hospitalization for a respiratory infection. For which potential complications of CF should the family be alert? Select all that apply. 1 Diabetes 2 Hematuria 3 Nasal polyps 4 Prolapsed rectum 5 Pulmonary infections 6 Urinary tract infections

1 Diabetes 3 Nasal polyps 4 Prolapsed rectum 5 Pulmonary infections By adolescence almost 50% of children with CF will have abnormal glucose tolerance test results as a result of the disease's effects on the pancreas

A child undergoes tonsillectomy and adenoidectomy for numerous recurrent respiratory tract infections. After the surgery, what should the nurse teach the parents to do? 1 Offer crushed ice chips. 2 Encourage the intake of ice cream. 3 Keep the child in the supine position. 4 Gargle with a diluted mouthwash solution.

1 Offer crushed ice chips. Ice chips are soothing and promote vasoconstriction.

The nurse is caring for a new mother who has a chlamydial infection. For which complications should the nurse assess the client's neonate? Select all that apply. 1 Pneumonia 2 Preterm birth 3 Microcephaly 4 Conjunctivitis 5 Congenital cataracts

1 Pneumonia 2 Preterm birth 4 Conjunctivitis

After an uneventful pregnancy a client gives birth to an infant with a meningocele. The neonate has 1-minute and 5-minute Apgar scores of 9 and 10, respectively. What is the priority nursing care for this newborn? 1 Protecting the sac with moist sterile gauze 2 Removing buccal mucus and administering oxygen 3 Placing name bracelets on both the mother and infant 4 Transferring the newborn to the neonatal intensive care unit

1 Protecting the sac with moist sterile gauze Preventing infection and trauma is the priority; rupture of the sac may lead to meningitis. The Apgar scores are 9 and 10 at 1 and 5 minutes, respectively; oxygen is not needed. Removing buccal mucus is not the priority. Placement of name bracelets on both mother and infant may be done before the infant leaves the birthing room; the priority is care of the infant's sac. The infant's sac must be protected before the infant is transferred to the neonatal intensive care unit.

Which finding in the urinalysis of a 5½-year-old child will alert the nurse to consider the possibility of lead-induced kidney damage? 1 Protein 2 Calcium 3 Potassium 4 Phosphate

1 Protein Protein is usually not excreted in urine because it is a large molecule. When found, it indicates kidney disease.

The nurse is caring for a newborn with caput succedaneum. The nurse is able to differentiate caput succedaneum from cephalhematoma because of what characteristic of the scalp edema in caput succedaneum? 1 Becomes ecchymotic 2 Crosses the suture line 3 Increases after several hours 4 Is tender in the surrounding area

2 Crosses the suture line Scalp edema that crosses the suture line is the clinical finding that differentiates between these two conditions. With caput succedaneum the swelling crosses the suture line, whereas in cephalhematoma it does not.

What strategy should the nurse employ to be effective when using play therapy with a 6-year-old child with autism? 1 Play music and dance with the child. 2 Use mechanical and inanimate objects for play. 3 Employing positive reinforcements such as hugging. 4 Provide brightly colored toys and blocks that can be held.

2 Use mechanical and inanimate objects for play. Self-isolation and disinterest in interpersonal relationships lead the autistic child to find security in nonthreatening, impersonal objects.

On a 6-week postpartum visit a new mother tells the nurse she wants to feed her baby whole milk after 2 months because she will be returning to work and can no longer breastfeed. The nurse plans to teach the mother that she should switch to formula feeding because whole milk does not meet the infant's nutritional requirements for what? 1 Fat and calcium 2 Vitamin C and iron 3 Thiamine and sodium 4 Protein and carbohydrates

2 Vitamin C and iron

A nurse is preparing to discharge a school-aged child who has undergone splenectomy for β-thalassemia (Cooley anemia). What information should the discharge teaching include? 1 The child should avoid participation in contact sports. 2 The hematologic problems have been cured by the splenectomy. 3 A high fever should be reported to the child's healthcare provider. 4 Blood transfusions will be needed more frequently than before the splenectomy

3 A high fever should be reported to the child's healthcare provider.

When picked up by a parent or the nurse, an 8-month-old infant screams and seems to be in pain. After observing this behavior, what should the nurse discuss with the parent? 1 Accidents and the importance of their prevention 2 Limiting playtime with other children in the family 3 Any other behaviors that the parent might have noticed 4 Food and specific vitamins that should be given to infants

3 Any other behaviors that the parent might have noticed

A 3-week-old infant has surgery for esophageal atresia. What is the immediate postoperative nursing care priority for this infant? 1 Giving the oral feedings slowly 2 Reporting vomiting to the practitioner 3 Checking the patency of the nasogastric tube 4 Monitoring the child for signs of infection at the incision site

3 Checking the patency of the nasogastric tube

A client in labor is admitted to the birthing unit 20 hours after her membranes have ruptured. Which complication should the nurse anticipate when assessing the character of the client's amniotic fluid? 1 Cord prolapse 2 Placenta previa 3 Maternal sepsis 4 Abruptio placentae

3 Maternal sepsis Prolonged rupture of membranes of more than 18 hours increases the risk of maternal and newborn sepsis.

At a routine monthly visit, while assessing a client who is in her 26th week of gestation, the nurse identifies the presence of striae gravidarum. The nurse describes this condition to the client as what? 1 Brownish blotches on the face 2 Purplish discoloration of the cervix 3 Reddish streaks on the abdomen and breasts 4 A black line running between the umbilicus and mons veneris

3 Reddish streaks on the abdomen and breasts Reddish streaks on the abdomen and breasts are striae gravidarum; they occur as a result of stretching of the breast and abdominal skin. These are known as "stretch marks." Chloasma refers to the condition where brownish blotches develop on the face. Purplish discoloration of the cervix is Chadwick sign. A black line running between the umbilicus and mons veneris is the linea nigra.

A 1-week-old infant has been in the pediatric unit for 18 hours after placement of a spica cast. The nurse obtains a respiratory rate slower than 24 breaths/min; no other changes are noted. Because the infant is apparently well, the nurse does not report or document the slow respiratory rate. Several hours later the infant experiences severe respiratory distress, and emergency care is necessary. What should be considered if legal action is taken? 1 Most infants' respirations are slow when they are uncomfortable. 2 The respirations of young infants are irregular, so a drop in rate is unimportant. 3 Vital signs that are outside the expected parameters are significant and should be documented. 4 The respiratory tracts of young infants are underdeveloped, and the respiratory rate is not significant.

3 Vital signs that are outside the expected parameters are significant and should be documented.

A 7-year-old child must remain quietly in bed while undergoing peritoneal dialysis. What activity is most appropriate for the nurse to plan for this child? 1 Learning to play chess 2 Constructing a model airplane 3 Working multiple-piece puzzles with another child 4 Using a large sponge ball to play catch with a roommate

3 Working multiple-piece puzzles with another child Working puzzles is a quiet activity that will not jeopardize placement of the peritoneal catheter and is appropriate for the child's cognitive level and allows social interaction with a peer. Chess requires cognitive abilities beyond the scope of a 7-year-old child. Although constructing a model airplane is a quiet activity, it is probably too difficult for a 7-year-old to do without help from an adult.

A new mother wishes to breast-feed her infant and asks the nurse whether she needs to alter her diet. How should the nurse respond? 1 "Eat as you have been during your pregnancy." 2 "Drink a lot of milk—the added calcium will help you make milk." 3 "Your body produces the milk your baby needs as a result of the vigorous suckling." 4 "You'll need greater amounts of the same foods you've been eating and more fluids."

4 "You'll need greater amounts of the same foods you've been eating and more fluids."

A nurse is teaching a group of women about the side effects of different types of contraceptives. What common side effect associated with the use of an intrauterine device (IUD) should the nurse discuss during the teaching session? 1 Tubal pregnancy 2 Rupture of the uterus 3 Expulsion of the device 4 Excessive menstrual flow

4 Excessive menstrual flow After IUD insertion there may be excessive menstrual flow for several cycles. Because the IUD is a foreign body, there is an increase in the blood supply, a result of the inflammatory process

A client who is at 38 weeks' gestation is admitted to the birthing unit because her membranes ruptured 24 hours ago and contractions have started. The fetus is in a breech presentation. The nurse observes that the amniotic fluid is green. What does the nurse conclude from these findings? 1 The fetus has a neural tube defect 2 Fetal well-being is compromised 3 Intrauterine infection has developed 4 Meconium is being expelled with contractions

4 Meconium is being expelled with contractions In a breech presentation, the pressure of the contractions on the fetus's lower abdomen causes meconium to be expelled into the amniotic fluid with each contraction.

What teaching must the nurse emphasize to the family when preparing a school-aged child with persistent asthma for discharge? 1 A cold, dry environment is desirable. 2 Limits should not be placed on the child's behavior. 3 The health problem is gone when symptoms subside. 4 Medications must be continued even when the child is asymptomatic.

4 Medications must be continued even when the child is asymptomatic.

During a well-child visit the parents tell a nurse, "Our 3-year-old doesn't listen to us when we speak and ignores us!" An auditory screening reveals that the child has a mild hearing loss. What should the nurse explain to the parents about this degree of hearing loss? 1 A severe hearing deficit may develop. 2 It will not interfere with progress in school. 3 An immediate follow-up visit is not necessary. 4 Speech therapy in addition to hearing aids may be required.

4 Speech therapy in addition to hearing aids may be required.

The nurse is conducting the admission assessment of a client who is positive for group B streptococcus (GBS). Which finding is of most concern to the nurse? 1 Continued bloody show 2 Cervical dilation of 4 cm 3 Contractions every 4 minutes 4 Spontaneous rupture of membranes 3 hours ago

4 Spontaneous rupture of membranes 3 hours ago

The nurse evaluates a new mother who is breastfeeding. The client asks how to care for her nipples. What should the nurse recommend? 1 Putting lanolin cream on the nipples after breastfeeding 2 Applying vitamin E gel to the nipples before breastfeeding 3 Using soap and water to clean the breasts and nipples at least once a day 4 Spreading breast milk on the nipples after the feeding and allowing them to air dry

4 Spreading breast milk on the nipples after the feeding and allowing them to air dry

A nurse is caring for an infant who has undergone surgery to repair a diaphragmatic hernia. What is the best position for the nurse to place the infant in? 1 Semi-Fowler in an infant seat 2 Side-lying on the unaffected side 3 Prone with the head turned to the side 4 Supine with the head of the bed elevated

4 Supine with the head of the bed elevated The supine position keeps pressure off the surgical site. Placing the infant on the unaffected side limits gas exchange in the lung on the unoperated side.

A nurse reviews with the parents of a young infant the principles of growth and development. Place the milestones in the order of their usual achievement. 1. Waves bye-bye and sits alone 2. Climbs stairs and drinks from a cup 3. Draws a vertical line and walks on tiptoe 4. Walks alone and builds a tower of two blocks 5. Sits momentarily without support and rolls over

5, 1, 4, 2, 3

A nurse working the 7 am to 3 pm shift is caring for a 14-year-old adolescent for whom intake and output are being monitored. The primary healthcare provider prescribes an intravenous infusion to be administered at a rate of 50 mL/hr. The adolescent had 4 oz (120 mL) of milk and a muffin for breakfast at 8:30 am. At 9 am the adolescent vomited 200 mL. At 10 am the adolescent had 60 mL of water with medications. At 11 am the adolescent voided 550 mL of urine. For lunch, at 12:30 pm, the adolescent ate 3 oz (90 mL) of soup and 4 oz (120 mL) of ice cream. The adolescent voided 450 mL at 2 pm. Calculate the adolescent's total intake for the 7 am to 3 pm shift. Record your answer using a whole number. _____ mL

790 mL

What is the characteristic of the preoperational stage of cognitive development? a. Generalizations can be made. b. Thinking is concrete. c. Thinking is logical. d. Reasoning is inductive.

???

A 3-year-old child, who has been diagnosed with asthma, is being prescribed albuterol (Ventolin) for acute episodes. The parents should be advised that the child may exhibit which of the following common side effects of the medication? A. Insomnia B. Lethargy C. Constipation D. Weight gain

A. Insomnia Other side effects include tachycardia, agitation, and excitability.

A child is admitted to the hospital with pneumonia. The child's oximetry reading upon admission to the pediatric floor is 88. The priority nursing activity for this child would be to: A. begin administration of intravenous fluids. B. begin oxygen per nasal cannula at 1 liter. C. obtain a blood sample to send to the lab for electrolyte analysis. D. medicate for pain.

B. begin oxygen per nasal cannula at 1 liter. Remember your ABCs when answering priority questions! 1 L is good start for peds patients as long as their O2 sat isn't too low

A nurse is assessing a preterm infant. The assessment that might indicate the infant's respiratory status is worsening is: A. an arterial CO2 of 40. B. grunting respirations with nasal flaring. C. acrocyanosis. D. periorbital edema.

B. grunting respirations with nasal flaring.

A client who is at 12 weeks' gestation tells the nurse at the prenatal clinic that she is experiencing severe nausea and frequent vomiting. The nurse suspects that the client has hyperemesis gravidarum. Which factor is frequently associated with this disorder? A. History of cholecystitis B. Large amount of amniotic fluid C. High level of chorionic gonadotropin D. Decreased secretion of hydrochloric acid

C. High level of chorionic gonadotropin A high level of chorionic gonadotropin is frequently associated with severe vomiting during pregnancy and may result in hyperemesis gravidarum. A high level may also occur in the presence of a hydatidiform mole or multiple pregnancy.

How long should a child with strep throat be on isolation?

For 24 hrs after starting ATBs. They can return to school when they feel well enough to once those 24hrs of ATBs are up.

Nurses must be alert for increased fluid requirements when a child presents with which possible concern? a. Mechanical ventilation b. Increased intracranial pressure (ICP) c. Fever d. Congestive heart failure

c. Fever

An infant's parents ask the nurse about preventing otitis media (OM). What intervention should the nurse recommend? Select one: a. Avoid tobacco smoke b. Bottle-feed or breastfeed in supine position c. Avoid children with OM d. Use nasal decongestant

a. Avoid tobacco smoke

What is the difference between asthma rescue/relief medications and controller medications?

Relief- is short acting, taken during exacerbation Control- is long acting, treats underlying things- maybe controls allergy response or decreases inflammation in general at a really low dose, effects mucous production, taken regardless of symptoms

Common clinical manifestations of vaccine reactions include: a. Intussusception b. Laryngeal edema c. Encephalopathy d. Urticaria

Urticaria

This disease has an incubation period of 14 to 21 days. Infected individuals are highly contagious. The disease presents with flu-like symptoms followed by an eruption of lesions that change from macules to papules to vesicles. a. Chickenpox b. Roseola c. Measles d. Mumps

a. Chickenpox

What name is given to identify an acquired hemorrhagic disorder that is characterized by excessive destruction of platelets? a. Immune thrombocytopenia b. Aplastic anemia c. Thalassemia major d. Disseminated intravascular coagulation

a. Immune thrombocytopenia

For clinical purposes, preterm and postterm infants are defined as: Select one: a. Preterm before 37 weeks, and postterm beyond 42 weeks, no matter the size for gestational age at birth. b. Preterm, SGA before 38 to 40 weeks, and postterm, LGA beyond 40 to 42 weeks. c. preterm before 34 weeks if appropriate for gestational age (AGA) and before 37 weeks if small for gestational age (SGA). d. Postterm after 40 weeks if large for gestational age (LGA) and beyond 42 weeks if AGA.

a. Preterm before 37 weeks, and postterm beyond 42 weeks, no matter the size for gestational age at birth.

This disease present with a high fever, and malaise followed by a maculopapular rash that starts on the face and moves downward. Affected children are often congested. Complications include blindness, encephalitis, otits media, seizures, and death a. Rubeola b. Varicella c. Roseola d. Erythema infectiousum

a. Rubeola

What is the priority nursing goal for a 14 year old diagnosed with Graves' disease? Select one: a. Verbalizing the importance of monitoring for medication side effects b. Developing alternative educational goals c. Allowing the adolescent to make decisions about whether or not to take medication d. Relieving constipation

a. Verbalizing the importance of monitoring for medication side effects

Which vitamin supplements are necessary for children with cystic fibrosis? Select one: a. Vitamins A, D, E, and K b. Vitamins B6 and B12 c. Magnesium d. Vitamin C and calcium

a. Vitamins A, D, E, and K

Which intervention is appropriate when examining a male infant for cryptorchidism? a. Warming the room b. Cooling the examiner's hands c. Taking a rectal temperature d. Eliciting the cremasteric reflex

a. Warming the room

Which intervention should be included in the plan of care for an infant with the nursing diagnosis of Excess Fluid Volume related to congestive heart failure? Select one: a. Weigh the infant every day on the same scale at the same time. b. Administer digoxin as ordered by the physician. c. Put the infant in a car seat to minimize movement. d. Notify the physician when weight gain exceeds more than 20 g/day.

a. Weigh the infant every day on the same scale at the same time.

Which pain assessment tool is most appropriate for a 6-year-old Select one or more: a. Wong-Baker Faces Scale b. rFLACC c. NIPS d. Numeric Scale

a. Wong-Baker Faces Scale

The nurse practicing in a labor setting knows that the woman most at risk for uterine rupture is: a. a gravida 4 who has had all cesarean births. b. a gravida 3 who has had two low-segment transverse cesarean births. c. a gravida 2 who had a low-segment vertical incision for delivery of a 10-lb infant. d. a gravida 5 who had two vaginal births and two cesarean births.

a. a gravida 4 who has had all cesarean births.

Injectable progestins (DMPA, Depo-Provera) are a good contraceptive choice for women who: a. have difficulty remembering to take oral contraceptives daily. b. are homeless or mobile and rarely receive health care. c. want menstrual regularity and predictability. d. have a history of thrombotic problems or breast cancer.

a. have difficulty remembering to take oral contraceptives daily.

The viral sexually transmitted infection (STI) that affects most people in the United States today is: Select one: a. human papillomavirus (HPV). b. cytomegalovirus (CMV). c. herpes simplex virus type 2 (HSV-2). d. human immunodeficiency virus (HIV).

a. human papillomavirus (HPV).

The nurse providing newborn stabilization must be aware that the primary side effect of maternal narcotic analgesia in the newborn is: a. respiratory depression. b. bradycardia. c. tachypnea. d. acrocyanosis.

a. respiratory depression.

Prepidil (prostaglandin gel) has been ordered for a pregnant woman at 43 weeks of gestation. The nurse recognizes that this medication will be administered to: a. ripen the cervix in preparation for labor induction. b. increase amniotic fluid volume. c. stimulate the amniotic membranes to rupture. d. enhance uteroplacental perfusion in an aging placenta.

a. ripen the cervix in preparation for labor induction.

Which "expected outcome" would be developmentally appropriate for a hospitalized 4-year-old child? a. The child will be dressed and fed by the parents. b. The child will independently ask for play materials or other personal needs. c. The child will be able to verbalize an understanding of the reason for the hospitalization. d. The child will have a parent stay in the room at all times.

b. The child will independently ask for play materials or other personal needs.

A 4-month-old infant diagnosed with gastroesophageal reflux disease (GERD) is thriving without other complications. What should the nurse suggest to minimize reflux? a. Place in Trendelenburg position after eating. b. Thicken formula with rice cereal. c. Give continuous nasogastric tube feedings. d. Give larger, less frequent feedings.

b. Thicken formula with rice cereal.

A 5-month-old has a white patches on the tongue and buccal mucosa. He was recently treated with an antibiotic for a strep infection. He has decreased oral intake and cries when the lesions are touched. a. Corona virus b. Thrush c. Coxsackievirus d. Lyme disease

b. Thrush

A meconium stool can be differentiated from a transitional stool in the newborn because the meconium stool is: Select one: a. the residue of a milk curd. b. passed in the first 12 hours of life. c. seen at age 3 days. d. lighter in color and looser in consistency.

b. passed in the first 12 hours of life.

To prevent nipple trauma, the nurse should instruct the new mother to: Select one: a. wash the nipples daily with mild soap and water. b. position the infant so the nipple is far back in the mouth. c. assess the nipples before each feeding. d. limit the feeding time to less than 5 minutes.

b. position the infant so the nipple is far back in the mouth.

The goal of treatment of the infant with phenylketonuria (PKU) is to: Select one: a. prevent gastrointestinal symptoms. b. prevent central nervous system (CNS) damage, which leads to mental retardation. c. cure the urinary tract infection. d. cure mental retardation.

b. prevent central nervous system (CNS) damage, which leads to mental retardation.

The nurse is taking a health history on an adolescent. Which best describes how the chief complaint should be determined? a. Ask for a detailed listing of symptoms. b. Use what the adolescent says to determine, in correct medical terminology, what the problem is. c. Ask the adolescent, "Why did you come here today?" d. Interview the parent away from the adolescent to determine the chief complaint.

c. Ask the adolescent, "Why did you come here today?"

What is the appropriate way to use the rFLACC tool to assess pain? Select one or more: a. Vital signs must be assessed to score pain with this tool b. Use this tool on a sleeping child and the Numeric Scale when the child is awake c. Assess an awake child for 1 to 5 minutes or longer to determine rFLACC scores d. A pain intervention is not needed with tool for a score less than 6

c. Assess an awake child for 1 to 5 minutes or longer to determine rFLACC scores

In evaluating the effectiveness of magnesium sulfate for the treatment of preterm labor, what finding would alert the nurse to possible side effects? a. Deep tendon reflexes 2+ and no clonus b. Respiratory rate of 16 breaths/min c. Serum magnesium level of 10 mg/dL d. Urine output of 160 mL in 4 hours

c. Serum magnesium level of 10 mg/dL

The leading cause of death from unintentional injuries in children is: a. poisoning. b. motor vehicle related fatalities. c. drowning. d. fire- and burn-related fatalities.

c. drowning.

With regard to a pregnant woman's anxiety and pain experience, nurses should be aware that: a. women who have had a painful labor will have learned from the experience and have less anxiety the second time because of increased familiarity. b. even mild anxiety must be treated. c. severe anxiety increases tension, which increases pain, which in turn increases fear and anxiety, and so on. d. anxiety may increase the perception of pain, but it does not affect the mechanism of labor.

c. severe anxiety increases tension, which increases pain, which in turn increases fear and anxiety, and so on.

A woman in preterm labor at 30 weeks of gestation receives two 12-mg doses of betamethasone intramuscularly. The purpose of this pharmacologic treatment is to: a. reduce maternal and fetal tachycardia associated with ritodrine administration. b. maintain adequate maternal respiratory effort and ventilation during magnesium sulfate therapy. c. stimulate fetal surfactant production. d. suppress uterine contractions.

c. stimulate fetal surfactant production.

What is a common clinical manifestation of juvenile hypothyroidism? Select one: a. Accelerated growth b. Diarrhea c. Dry skin d. Insomnia

c. Dry skin

The school nurse is assessing a 10-year-old boy with hemophilia who has fallen while playing in the schoolyard. At which site does the nurse expect to find internal bleeding? 1 Joints 2 Abdomen 3 Cerebrum 4 Epiphyses

Joints

What in students is being assessed when a school nurse conducts audiometric screenings? 1 Hearing acuity 2 Sensorineural hearing loss 3 Auditory processing deficits 4 Hearing problems caused by wax

1 Hearing acuity

What is a common, serious complication of rheumatic fever? Select one: a. Cardiac valve damage b. Cardiac arrhythmias c. Seizures d. Pulmonary hypertension

a. Cardiac valve damage

Which age-group is most concerned with body integrity? a. Toddler b. School-age child c. Adolescent d. Preschooler

b. School-age child

Which diagnostic finding is present when a child has primary nephrotic syndrome? Select one: a. Positive ASO titer b. Leukocytosis c. Proteinuria d. Hyperalbuminemia

c. Proteinuria

An abortion in which the fetus dies but is retained within the uterus is called a(n): a. threatened abortion. b. incomplete abortion. c. missed abortion. d. inevitable abortion.

c. missed abortion.

A woman who has completed one pregnancy with a fetus (or fetuses) reaching the stage of fetal viability is called a: a. primigravida. b. multipara. c. nulligravida. d. primipara.

d. primipara.

Jaundice develops in a newborn 72 hours after birth. How should the nurse best explain the probable cause of this jaundice to the parents? 1 An allergic response to the feedings 2 The physiologic destruction of fetal red blood cells 3 A temporary bile duct obstruction commonly found in newborns 4 The seepage of maternal Rh-negative blood into the neonate's bloodstream

2 The physiologic destruction of fetal red blood cells After birth, fetal erythrocytes hemolyze, releasing bilirubin into the circulation; the immature liver cannot metabolize the bilirubin as rapidly as it is produced, resulting in physiologic jaundice.

A client at 7 weeks' gestation tells the nurse in the prenatal clinic that she is sick every morning with nausea and vomiting and adds that she does not think she can tolerate it throughout her pregnancy. The nurse assures her that this is a common occurrence in early pregnancy and will probably disappear by the end of which month? 1 Fifth month 2 Third month 3 Fourth month 4 Second month

2 Third month Morning sickness rarely persists beyond the first trimester.

Which strategy needs to be employed while interviewing the adolescent as a part of her health-screening? 1 To start with more sensitive issues 2 To explain the limits of confidentiality 3 To ask more of close-ended questions 4 To interview the adolescent along with her parents

2 To explain the limits of confidentiality

A first-time mother at 18 weeks of gestation comes for her regularly scheduled prenatal visit. The patient tells the nurse that she is afraid that she is going into premature labor because she is beginning to have regular contractions. The nurse explains that this is the Braxton Hicks sign and teaches the patient that this type of contraction: a. causes cervical dilation. b. impedes oxygen flow to the fetus. c. is painless. d. increases with walking.

c. is painless.

In which condition are all the formed elements of the blood simultaneously depressed? Select one: a. Thalassemia major b. Iron deficiency anemia c. Sickle cell anemia d. Aplastic anemia

d. Aplastic anemia


Kaugnay na mga set ng pag-aaral

Roaring Economy to Great Depression QUIZ 100%

View Set

Virginia Real Estate License Exam

View Set

Service Titan Essential System Practice

View Set

Chapter 12 - Individual Policy Provisions

View Set

Ancient civilizations: Greece: dark ages

View Set

Chapter 7 - Nutrition Through the Lifespan

View Set